BRS comprehensive exam (Embryo, physio, path, pharm, micro)

अब Quizwiz के साथ अपने होमवर्क और परीक्षाओं को एस करें!

Which phase of the cardiac cycle is absent if there is no P wave on the electrocardiogram (ECG)? (A) Atrial systole (B) Isovolumetric ventricular contraction (C) Rapid ventricular ejection (D) Reduced ventricular ejection (E) Isovolumetric ventricular relaxation (F) Rapid ventricular filling (G) Reduced ventricular filling

A [Chapter 3, V; Figure 3.15]. The P wave represents electrical activation (depolarization) of the atria. Atrial contraction is always preceded by electrical activation.

Which of the following hormones causes contraction of vascular smooth muscle? (A) Antidiuretic hormone (ADH) (B) Aldosterone (C) Atrial natriuretic peptide (ANP) (D) 1,25-Dihydroxycholecalciferol (E) Parathyroid hormone (PTH)

A [Chapter 7, III C 1 b]. Antidiuretic hormone (ADH) not only produces increased water reabsorption in the renal collecting ducts (V2 receptors) but also causes constriction of vascular smooth muscle (V1 receptors).

A 56-year-old man was recently diagnosed with early-stage colon cancer. He has no known family history of colon cancer. Which of the following known risk factors is most likely to have contributed to the development of this form of cancer? (A) A diet low in fiber and high in fat (B) Aflatoxin B1 ingestion (C) Helicobacter pylori infection (D) Hepatitis B infection (E) Tobacco and alcohol abuse

A. A diet low in fiber and high in fat is believed to be a risk factor for the development of colon cancer. Both aflatoxin B1 ingestion and hepatitis B infection are risk factors for hepatocellular carcinoma. Helicobacter pylori infection is associated with stomach cancer. Cancers of the mouth, tongue, and esophagus have a marked association with the combined abuse of tobacco and alcohol, as well as HPV infection in a subset of cases

In addition to IL-1, which of the following is an inflammation inducing cytokine? (A) IL-2 (B) IL-5 (C) TNF-a (D) TGF-b

A. Both IL-1 and TNF-a are released following tissue injury and induce inflammation.

A 3-year-old girl presents with dark precipitates along gingival margins, radiopaque deposits in the epiphyses of her bones, and urinary excretion of delta-aminolevulinic acid (delta-ALA). Her father states that they live in an old house that has chipped paint. The patient is diagnosed with lead toxicity. The child's blood would likely also have which of the following? (A) Basophilic stippling of erythrocytes (B) Schistocytes and helmet cells (C) Increased osmotic fragility of erythrocytes (D) Clumping of erythrocytes at temperatures below 30°C (E) Macrocytes

A. Classic features of lead poisoning include dark precipitates forming a gingival lead line (composed of precipitated lead sulfide), radiopaque deposits in epiphyses, basophilic stippling of erythrocytes, increased urinary delta-ALA, and peripheral neuropathy and other CNS changes.

If a series of drug resistance genes are transferred together routinely, what is the most likely method of transfer? (A) Plasmid-mediated conjugation (B) Generalized transduction (C) Specialized transduction (D) Transformation

A. Genes for drug resistance may reside individually on the bacterial chromosome or on plasmids. Clusters of multiple drug resistance genes are occasionally on chromosomes but are more commonly on plasmids. Also, in general, plasmid DNA is entirely transferred in a conjugal cross of F+ cells with F- cells. R-factors are like those F+ cells with just a few extra, closely linked drug resistance genes. So the conjugal transfer of a R- factor is the process that is most likely to transfer multiple drug resistance.

When IgG is cleaved by papain, which of the following fragments appear? (A) Two monovalent antigen binding fragments (Fab) (B) Two Fab fragments that contain only the variable section of the heavy chain (C) Two Fab fragments that contain only the variable section of the light chain (D) Two Fc (crystallizable) fragments and one Fab fragment

A. Papain cleaves IgG into two monovalent antigen binding fragments containing both a heavy and light chain.

Organophosphate poisoning is treated with (A) Pralidoxime (B) Parathion (C) Amyl nitrate (D) Bethanechol (E) Nicotine

A. Pralidoxime reactivates acetylcholinesterase to reverse the effects of exposure to organophosphates, of which parathion is actually an example. Amyl nitrate can be used in cases of ingestion of the cytochrome oxidase inhibitor cyanide. Bethanechol is a direct-acting muscarinic cholinoceptor agonist used to treat urinary retention and overdose and can result in symptoms similar to organophosphate poisoning. Nicotine is sometimes found in insecticides and can cause vomiting, weakness, seizures, and respiratory arrest.

A newborn baby has redness associated with the conjunctiva of both eyes. She keeps rubbing them as if irritated and a watery discharge is present. Visual examination shows a ''dendritic ulcer'' present. Appropriate topical treatment is with (A) Acyclovir (B) Foscarnet (C) Ribavirin (D) Zanamivir

A. The ''dendritic ulcer'' indicates that the conjunctivitis is caused by HSV, which should be treated with the antiherpes drug acyclovir.

A 60-year-old man undergoes upper endoscopy for dyspepsia. The endoscopist notes a well-circumscribed submucosal tumor. The tumor is resected and demonstrates a spindled cell population on microscopy. Immunostaining for CD117 is positive. The diagnosis is (A) Gastrointestinal stromal tumor (GIST). (B) Melanoma. (C) Leiomyoma. (D) Adenocarcinoma. (E) Lymphoma.

A. The clinical and microscopic picture is that of GIST. These tumors most commonly arise in the submucosa of the stomach but can arise anywhere along the GI tract and occasionally occur outside the GI tract. Microscopically they are comprised of spindled cells which stain for CD117. On the molecular level, GISTs show mutations in the oncogene c-kit. A minority of tumors also show PDGFR-á mutations. Although most cases are indolent and cured by resection, a subset shows aggressive behavior. Size, mitotic count, and location are the most important predictors of prognosis, with small size, low mitotic activity, and gastric location suggesting relatively benign behavior.

A 24-year-old nonsmoker who does not consume alcohol is found to have both pulmonary emphysema and cirrhosis of the liver. A sister and several close relatives also have had similar findings. This history suggests that this patient's illness may be caused by deficiency of (A) á1-antitrypsin. (B) galactokinase. (C) glucose-6-phosphatase. (D) glucocerebrosidase. (E) phenylalanine hydroxylase.

A. The combination of emphysema and hepatic cirrhosis in a young person with a family history of similarly affected family members is strongly suggestive of homozygous á1-antitrypsin deficiency. In these instances the emphysema is usually panacinar in type.

A patient presents with an inflamed, itchy, expanding cutaneous lesion on her side. She suspects she might have picked it up from her dog as it is where the dog sleeps next to her and the dog is losing fur. You treat with topical drug from one of the major drug families commonly used to treat the infection. What is the mechanism of action of that drug class? (A) Inhibition of ergosterol synthesis (B) Nicking 60S ribosomes (C) Inhibition of formation of the peptide bond and peptide chain elongation on 70S ribosomes (D) Inhibition of chitin synthesis (E) Inhibition of mycolic acid synthesis (F) Inhibition of microtubule formation

A. The infection is most likely tinea corporis transferred by direct contact from the dog to the human. The drug most likely used was a topical imidazole, which inhibits the synthesis of ergosterol needed for fungal membranes. The other drugs commonly used include terbinafine, for which the answer would be the same, or griseofulvin, which inhibits cell division by disrupting microtubules.

The v-src oncogene of Rous sarcoma virus is a (A) Protein kinase (B) Growth factor (C) DNA-binding protein (D) G protein (E) GTP binding protein

A. The v-src oncogene associated with the Rous sarcoma virus codes for a tyrosine protein kinase with a biologic activity that results in cellular transformation.

Chronic inflammatory disease confined to the rectum characterizes (A) Ulcerative colitis (B) Myasthenia gravis (C) Wiskott-Aldrich syndrome (D) Systemic lupus erythematosus (SLE) (E) Graves disease

A. Ulcerative colitis is the only syndrome listed whose chronic inflammation is confined to the rectum.

If an artery is partially occluded by an embolism such that its radius becomes one-half the preocclusion value, which of the following parameters will increase by a factor of 16? (A) Blood flow (B) Resistance (C) Pressure gradient (D) Capacitance

B [Chapter 3, II C, D]. A decrease in radius causes an increase in resistance, as described by the Poiseuille relationship (resistance is inversely proportional to r4). Thus, if radius decreases twofold, the resistance will increase by (2)4 or 16-fold.

Fat absorption occurs primarily in the (A) stomach (B) jejunum (C) terminal ileum (D) cecum (E) sigmoid colon

B [Chapter 6, V C 1, 2]. First, fat absorption requires the breakdown of dietary lipids to fatty acids, monoglycerides, and cholesterol in the duodenum by pancreatic lipases. Second, fat absorption requires the presence of bile acids, which are secreted into the small intestine by the gallbladder. These bile acids form micelles around the products of lipid digestion and deliver them to the absorbing surface of the small intestinal cells. Because the bile acids are recirculated to the liver from the ileum, fat absorption must be complete before the chyme reaches the terminal ileum.

Which gastrointestinal secretion has a component that is required for the intestinal absorption of vitamin B12? (A) Saliva (B) Gastric secretion (C) Pancreatic secretion (D) Bile

B [Chapter 6, V E 1 c; Table 6.3]. Gastric parietal cells secrete intrinsic factor, which is required for the intestinal absorption of vitamin B12.

Which step in the steroid hormone synthetic pathway is required for the development of female secondary sex characteristics, but not male secondary sex characteristics? (A) Aldosterone synthase (B) Aromatase (C) Cholesterol desmolase (D) 17,20-Lyase (E) 5á-Reductase

B [Chapter 7, X A]. Aromatase catalyzes the conversion of testosterone to estradiol in the ovarian granulosa cells. Estradiol is required for the development of female secondary sex characteristics.

Which immunoglobulin has the highest level in a normal 1 day old infant? (A) IgA (B) IgG (C) IgM (D) IgD (E) IgE

B. A 1 day old infant has a maternal level of IgG due to cross-placental transfer of the mother's IgG. The other Igs are slowly being synthesized by the infant.

A 48-year-old woman who presents with muscle weakness and a thymoma is suspected of having myasthenia gravis. If correct, lab tests should show autoantibody against (A) Thyroid-stimulating hormone (B) Acetylcholine receptor (C) Intrinsic factor (D) Rheumatoid factor

B. Autoantibodies against acetylcholine receptor are characteristic of myasthenia gravis and are not present in the other conditions.

An antibiotic that has a b-lactam ring in its structure is (A) Tetracycline (B) Cephalosporin (C) Streptomycin (D) Erythromycin (E) Griseofulvin (F) Bacitracin

B. Cephalosporin drugs have the b-lactam ring as do the penicillins. They also inhibit cell-wall biosynthesis and are inactivated by some b-lactamases.

Since the patient in the previous question was treated successfully in terms of his blood pressure, you elect to keep the same overall pharmacologic strategy. Which of the following would allow you to do this? (A) Aliskirin (B) Epleronone (C) Amiloride (D) Enalapril (E) Metazolone

B. Epleronone is a much more specific mineralocorticoid receptor antagonist than sprinolactone and is not associated with gynocomastia. Amiloride is another potassium-sparing diuretic but acts to block renal ENa channels.

In which of the following phases of growth is a Gram-positive bacterium most susceptible to the action of penicillin? (A) Lag (B) Exponential (C) Stationary (D) Decline (E) Death

B. Gram-positive bacteria would be most susceptible to penicillin in the exponential phase, because this is the phase in which cell-wall synthesis is greatest.

A 50-year-old man presents with a neurologic disorder that began with choreoathetoid movements and has progressed to dementia. Other family members have been similarly affected. A CT scan reveals atrophy of the caudate nucleus, resulting in the appearance of enlarged lateral ventricles. Which of the following is the most likely diagnosis? (A) Alzheimer disease (B) Huntington disease (C) Idiopathic Parkinson disease (D) Myasthenia gravis (E) Wernicke-Korsakoff syndrome

B. Huntington disease, a disorder caused by an increased number of trinucleotide repeats, is manifested anatomically by progressive degeneration and atrophy of the caudate nucleus, putamen, and frontal cortex. The caudate nucleus normally bulges convexly into the lateral ventricles; thus atrophy of the caudate nucleus results in the appearance of "bat-wing-shaped" or enlarged lateral ventricles.

An individual is known to have chronic hepatitis disease. Which serological marker should be monitored to determine if the blood contains the infectious virus? (A) HBcAg (B) HBeAg (C) HBsAg (D) IgG HBs (E) IgM HBc

B. Levels of HBeAg are monitored to determine if blood contains infectious HBV.

O antigens are found in (A) Capsule (B) Lipopolysaccharide (C) Lipoprotein (D) Mesosome (E) Peptidoglycan (F) Teichoic acid

B. O antigen or O-specific side chains are major surface antigens in the polysaccharide component of lipopolysaccharide.

What autoimmune disease is characterized by antibodies against intrinsic factor? (A) Congenital agammaglobulinemia (B) Pernicious anemia (C) Wiskott-Aldrich syndrome (D) Dysgammaglobulinemia (E) Graves disease

B. Pernicious anemia patients give rise to antibodies against intrinsic factor that inhibit the transfer of vitamin B12 from the stomach to the bloodstream.

Which of the following drugs is a selective á-adrenergic receptor agonist that is available over the counter? (A) Epinephrine (B) Phenylephrine (C) Isoproterenol (D) Norepinephrine (E) Phentolamine

B. Phenylephrine is a selective á1-adrenoreceptor agonist that causes nasal vasoconstriction, which results in decreased nasal secretion. Epinephrine is the most potent of the adrenergic receptor agonists, followed by norepinephrine. Isoproterenol is the weakest antagonist. But the previous three agents also bind â-adrenergic receptors and are not available over the counter. Phentolamine is just the opposite, an á1-adrenergic antagonist.

A 45-year-old woman presents with an insidious and progressive syndrome characterized by pain and tenderness in multiple joints, with joint stiffness on rising in the morning, and early afternoon fatigue and malaise. Joint involvement is symmetric, with the proximal interphalangeal and metacarpophalangeal joints especially involved. Physical examination reveals tenderness in nearly all inflamed joints. Which of the following laboratory abnormalities is most likely? (A) Antibodies to double-stranded DNA (B) IgM anti-IgG antibodies (C) Urate crystals and neutrophils in synovial fluid (D) Anti-DNAase B (E) HLA-B27 antigen

B. Symmetric polyarthritis with involvement of the proximal interphalangeal and metacarpophalangeal joints in a female patient are characteristics of rheumatoid arthritis. Rheumatoid factor, an IgM antibody directed against the Fc portion of IgG, is found in about 80% of affected individuals.

In March, a nursery school reports an outbreak among the children of a disease characterized by a high grade (38.8C to 40C) fever that starts suddenly and rapidly falls. The children develop a short-lived (1 to 2 days) maculopapular rash that begins on the trunk and spreads to extremities; the face is spared. The children have mild malaise, but otherwise do not appear ill. The most likely causative agent is (A) Coxsackie A16 virus (B) HHV-6 (C) Parvovirus B19 (D) Rubella virus (E) VZV

B. The disease is most likely roseola caused by HHV-6 since it has a short duration and the face, palms and soles are not affected.

A 5-year-old boy is seen because he has recurrent hemarthroses and a large painful hematoma involving the soft tissues of his right thigh. Given the following choices, which is the most likely cause of the bleeding? (A) Ascorbic acid deficiency (B) Christmas disease (C) DIC (D) Rocky Mountain spotted fever (E) Primary thrombocytopenia

B. The patient has secondary hemostatic bleeding, which is characteristic of disorders of the coagulation pathway. Relatively common coagulation pathway disorders include classic hemophilia (factor VIII deficiency) and Christmas disease (factor IX deficiency). Both are disorders of the intrinsic pathway of coagulation, and are clinically indistinguishable from one another except by specific factor assays. DIC is characterized by widespread thrombosis and hemorrhage with both primary and secondary hemostatic bleeding. The other choices listed are characterized by primary hemostatic bleeding, which is manifested by punctate cutaneous hemorrhages and oozing from mucosal surfaces.

What organism is noted for its antigenic variation that leads to its ability to repeatedly reinfect and cause disease and has led to difficulty in vaccine development? Estimates of number of different serotypes are over 1 million. (A) Chlamydia trachomatis (B) Neisseria gonorrhoeae (C) Neisseria meningitidis (D) Streptococcus pneumoniae (E) Treponema pallidum

B. The phenomenon described is antigenic variation. It is a particular problem in Neisseria gonorrhoeae.

A patient with a cough of several weeks' duration but no fever has radiological signs of a coin lesion. The needle biopsy shows large, broad-based budding yeasts with a very thick cell wall, most of which are extracellular. What is the most likely diagnosis? (A) Aspergillosis (B) Blastomycosis (C) Coccidioidomycosis (D) Histoplasmosis (E) Cancer

B. The picture given is classical for the pulmonary lesions of blastomycosis, which cannot be differentiated from carcinoma without biopsy.

A 10-year-old boy presents with an itchy scalp with the underlying hair becoming lighter. The patch has expanded to 2 inches. What would you expect to see in a KOH mount of skin scrapings from this patient with tinea capitis? (A) Yeast cells, pseudohyphae, and true hyphae (B) Regular septate hyphae with little branching (C) Broad aseptate hyphae (D) Septate hyphae regularly branching dichotomously at an acute angle (E) Yeasts only

B. The tineas are caused by dermatophytes that generally appear in the tissue as septate hyphae with little branching. Sometimes arthroconidia are formed where the entire filament breaks up into spores.

A 65-year-old man is found to have the combination of increased skin pigmentation, cirrhosis of the liver, and diabetes mellitus. Which pattern of serum iron and total iron-binding capacity (TIBC) is most consistent with the familial illness suggested by these findings? (A) Serum iron increased, TIBC increased (B) Serum iron increased, TIBC decreased (C) Serum iron normal, TIBC increased (D) Serum iron decreased, TIBC increased (E) Serum iron decreased, TIBC decreased

B. The vignette is suggestive of long-standing, late-stage hereditary hemochromatosis, and the expected findings would include markedly increased serum iron and moderately reduced TIBC. This combination often results in almost 100% saturation of iron-binding capacity.

A 59-year-old man presents at the emergency department with labored breathing and a sharp stabbing pain behind the breast bone. He complains of being weak and tired. He has a low-grade fever and an electrocardiogram shows elevated ST segments. The most likely infectious cause of these symptoms is (A) BK virus (B) Coxsackie B virus (C) Moraxella catarrhalis (D) Nocardia asteroides (E) Rotavirus

B. Viruses are the major infectious agents that produce symptoms similar to a heart attack. The most common of these viruses is Coxsackie B virus.

What should be given to correct coagulopathy due to an overdose of warfarin in a 73-year-old man? (A) Aminocaproic acid (B) Vitamin K (C) Heparin (D) Vitamin D (E) Oprelveldn

B. Warfarin is an orally active inhibitor of vitamin K-dependent carboxylation of various clotting factors. In the event of supratherapeutic doses of warfarin, the anticoagulation can be reversed by giving vitamin K. Heparin is an intravenous preparation that is also an anticoagulation agent. Aminocaproic acid inhibits plasminogen activation and is used in the treatment of hemophilia. Vitamin D is used in cases of its deficiency or in the treatment of osteoporosis. Oprelvekin is a recombinant form of interleukin-11 that stimulates platelet production and does not affect the clotting factors.

Albuterol is useful in the treatment of asthma because it acts as an agonist at which of the following receptors? (A) á1 Receptor (B) â1 Receptor (C) â2 Receptor (D) Muscarinic receptor (E) Nicotinic receptor

C [Chapter 2, I C 1 d]. Albuterol is an adrenergic â2 agonist. When activated, the b2 receptors in the bronchioles produce bronchodilation.

A 5-year-old boy has a severe sore throat, high fever, and cervical adenopathy. Before antibiotic therapy is initiated, the patient is given aspirin to reduce his fever. The mechanism of fever reduction by aspirin is (A) shivering (B) stimulation of cyclooxygenase (C) inhibition of prostaglandin synthesis (D) shunting of blood from the surface of the skin (E) increasing the hypothalamic set-point temperature

C [Chapter 2, VI C 2]. By inhibiting cyclooxygenase, aspirin inhibits the production of prostaglandins and lowers the hypothalamic set-point temperature to its original value. After aspirin treatment, the hypothalamus "reads" the body temperature as being higher than the set-point temperature and activates heat-loss mechanisms, including sweating and vasodilation of skin blood vessels. This vasodilation shunts blood toward the surface skin. When heat is lost from the body by these mechanisms, body temperature is reduced.

A patient's electrocardiogram (ECG) shows periodic QRS complexes that are not preceded by P waves and that have a bizarre shape. These QRS complexes originated in the (A) sinoatrial (SA) node (B) atrioventricular (AV) node (C) His-Purkinje system (D) ventricular muscle

C [Chapter 3, III A]. Because there are no P waves associated with the bizarre QRS complex, activation could not have begun in the sinoatrial (SA) node. If the beat had originated in the atrioventricular (AV) node, the QRS complex would have had a "normal" shape because the ventricles would activate in their normal sequence. Therefore, the beat must have originated in the His-Purkinje system, and the bizarre shape of the QRS complex reflects an improper activation sequence of the ventricles. Ventricular muscle does not have pacemaker properties.

Compared with the base of the lung, in a person who is standing, the apex of the lung has (A) a higher ventilation rate (B) a higher perfusion rate (C) a higher ventilation/perfusion (V/Q) ratio (D) the same V/Q ratio (E) a lower pulmonary capillary PO2

C [Chapter 4, VII C; Table 4.5]. In a person who is standing, both ventilation and perfusion are greater at the base of the lung than at the apex. However, because the regional differences for perfusion are greater than those for ventilation, the ventilation/ perfusion (V/Q) ratio is higher at the apex than at the base. The pulmonary capillary Po2 therefore is higher at the apex than at the base because the higher V/Q ratio makes gas exchange more efficient.

Which of the following transport mechanisms is inhibited by furosemide in the thick ascending limb? (A) Na+ diffusion via Na+ channels (B) Na+-glucose cotransport (symport) (C) Na+-K+-2Cl- cotransport (symport) (D) Na+-H+ exchange (antiport) (E) Na+,K+-adenosine triphosphatase (ATPase)

C [Chapter 5, IV C 2]. Na+-K+-2Cl- cotransport is the mechanism in the luminal membrane of the thick ascending limb cells that is inhibited by loop diuretics such as furosemide. Other loop diuretics that inhibit this transporter are bumetanide and ethacrynic acid.

A 62-year-old male alcoholic being treated for non-insulin-dependent diabetes mellitus comes to the emergency department with a 1-hour history of nausea, vomiting, headache, hypotension, and profuse sweating. What is the most likely causative agent? (A) Clomiphene (B) Glyburide (C) Chlorpropamide (D) Nandrolone (E) Vasopressin

C. A disulfiram-like reaction may be seen in noninsulin-dependent diabetics treated with chlorpropamide, an oral hypoglycemic, when used in combination with alcohol.

In a 44-year-old man with hemoptysis and hematuria, a linear pattern of glomerular immunofluorescence for IgG is observed in a renal biopsy. The most likely associated laboratory finding is a positive test for antibodies directed to (A) streptolysin O. (B) C3 convertase. (C) glomerular basement membranes. (D) hepatitis B virus. (E) Sm (Smith) nuclear antigen.

C. A linear pattern of glomerular immunofluorescence for IgG is found in Goodpasture syndrome, which is caused by antibodies that react with both glomerular and alveolar basement membranes.

The mother of a 9 year old who is in the same school as another child just diagnosed with meningitis (but who did not know nor had any direct contact with the infected child), called the pediatrician demanding her son be prophylactically treated. The physician suggests vaccinating the boy and doing surveillance cultures and watching the child for symptoms. The medium that the lab will use has antibiotics in it and therefore is called a(n) (A) Minimal medium (B) Differential medium (C) Selective medium (D) Inhibitory medium

C. A selective medium permits growth in the presence of agents that inhibit other bacteria, in this case the normal oropharyngeal flora. A minimal medium contains the minimum quantity and number of nutrients capable of sustaining growth of the organism. A differential medium differentiates among organisms on the basis of color due to different fermentation or pH. Although you are inhibiting some bacteria, this name is not used as you are setting conditions to selectively grow the Neisseria.

An infant with combined immunodeficiency disease (SCID) exhibits (A) Presence of the thymus but absence of the bursal equivalent (B) Deficiency in the nicotinamide adenine dinucleotide phosphate (NADPH) oxidase system (C) Deficiency in adenosine deaminase, and loss of this enzyme activity (D) An absence of neutrophils

C. Absence of adenosine deaminase results in the accumulation of adenosine interfering with DNA synthesis. Although lymphocytes are severely depressed, myeloid cells are present in normal numbers. The thymus does not develop.

A 21-year-old man sustains multiple blunt traumas after being beaten with a baseball bat by a gang. Aside from his fractures, a serum creatine kinase measurement is dramatically elevated and the trauma team is worried as the myoglobinuria caused by the trauma can cause kidney failure. Although controversial because no randomized trials have been conducted to show efficacy, they immediately begin to administer bicarbonate to alkalinize the urine. How might this serve to decrease myoglobin levels? (A) Increasing glomerular filtration (B) Promoting renal tubular secretion (C) Inhibiting renal tubular reabsorption (D) Increasing hepatic first-pass metabolism (E) Inducing the P-450 system

C. Alterations in urinary pH alters renal reabsorption of substances. In this case, alkalinization traps filtered myoglobin in the urine so that it cannot be reabsorbed, which leads to decreased levels in the serum. The other mechanisms such as increasing glomerular filtration and promoting tubular secretion are other potential ways to alter plasma drug/metabolite levels. Myoglobin is not hepatically metabolized; therefore, hepatic or P-450 metabolism would not alter myoglobin levels.

A child presents with impetigo with bullae. A Gram-positive, b-hemolytic, catalase-positive, coagulase-positive coccus is isolated. Which of the following is the most likely organism? (A) Group A streptococcus (B) Group B streptococcus (C) Staphylococcus aureus (D) Staphylococcus epidermidis

C. Although groups A and B streptococci and Staphylococcus aureus are bhemolytic, only the streptococci are catalase negative. Only S. aureus is both b-hemolytic and coagulase positive.

A 30-year-old woman dies following a prolonged period in a vegetative state that resulted from injuries sustained in a headon auto collision 5 years earlier. An expected autopsy finding would be (A) a cherry-red spot on the macula. (B) asbestos bodies in the lung. (C) autophagic granules in skeletal muscles. (D) Lewy bodies within neurons. (E) Mallory bodies within hepatocytes.

C. Autophagic granules are intracytoplasmic vacuoles containing debris from degraded organelles, such as mitochondria. They are especially prominent in cells that have become atrophic, such as skeletal muscle cells after prolonged immobilization.

Which of the following statements characterizes idiotypic determinants? (A) They are found in the crystallizable fragment of immunoglobulins (B) They are found on protein antigens (C) They can be antigenic (D) They are responsible for rejection of transplants

C. Because idiotypic determinants on antibodies contain amino acid sequences in the (Fab')2 variable regions that are unique to the respondent, they can be antigenic.

Which of the following is a good choice to treat newly diagnosed generalized anxiety disorder (GAD) in a patient who is a truck driver? (A) Alprazolam (B) Triazolam (C) Buspirone (D) Trazodone (E) Thiopental

C. Buspirone is a partial serotonin 5-HT1A-receptor agonist that has efficacy comparable to that of benzodiazepines for the treatment of anxiety, but is significantly less sedating. Alprazolam is an intermediate-acting benzodiazepine used in the treatment of generalized anxiety disorder (GAD) but still has some sedation, which would be undesirable in this situation. Triazolam is a short-acting benzodiazepine, and trazodone is a heterocyclic antidepressant, both used to induce sleep. Thiopental is a barbiturate sometimes used to induce anesthesia.

A receptor for the human immunodeficiency virus (HIV) is (A) CD2 (B) CD3 (C) CD4 (D) CD8 (E) CD25

C. CD4 is a prominent receptor for Gp 160.

A 65-year-old man is evaluated for abdominal pain radiating through to the back, jaundice, anorexia, and recent weight loss. An additional likely finding is (A) history of thorium dioxide (Thorotrast) exposure. (B) increased AFP. (C) migratory venous thrombosis. (D) pancreatic calcification and pseudocyst formation. (E) urine test negative for bilirubin.

C. Carcinoma of the pancreas with common bile duct obstruction is strongly suggested by the clinical findings. Spontaneous migratory venous thrombosis with visceral neoplasms is known as the Trousseau sign or syndrome.

A woman develops cervical carcinoma. What viral protein played a role in the development of the carcinoma? (A) Large T antigen (B) E1A protein (C) E6 protein (D) TAX protein

C. Cervical carcinoma is caused by oncogenic strains of the human papillomavirus (most commonly 16, 18, and 31). The early protein E6 is associated with the oncogenic potential of human papillomavirus.

A 50-year-old man is seen in the emergency room. He sustained a head injury after falling from a ladder. He had a brief period of loss of consciousness after the fall, which then resolved. He now has headache, vomiting, and borderline state of consciousness. Emergency CT demonstrates a fracture in the temporoparietal area of the skull, as well as a lens-shaped homogeneous density contained within the suture lines. Which of the following is the most likely diagnosis? (A) Bruising of the brain substance of the cerebral hemisphere (B) Enlargement of the cerebral ventricles (C) Epidural hematoma (D) Subdural hematoma (E) Subarachnoid hemorrhage

C. Epidural hematoma is most often caused by skull fracture with laceration of the branches of the middle meningeal artery. The classic CT scan finding is formation of a "lens-shaped" density contained within the suture lines of the skull.

A nurse comes into your office informing you that the child you delivered yesterday failed to pass meconium. The nurse remarks that the child also cries on palpation of the abdominal area. Relevant physical exam finding include abdominal distention, megacolon on palpation, and gushing of fecal material on a rectal digital exam. Relevant laboratory findings include barium enema showing a dilated proximal segment and a narrow distal segment of the sigmoid colon. Which of the following is the most likely diagnosis? (A) Rectal atresia (B) Rectovesical fistula (C) Hirschsprung disease (D) Anorectal agenesis (E) Intussusception

C. Hirschsprung disease (colonic aganglionosis). Hirschsprung disease is caused by the arrest of the caudal migration of neural crest cells. The hallmark is the absence of ganglionic cells in the myenteric and submucosal plexuses, most commonly in the sigmoid colon and rectum, resulting in a narrow segment of colon (i.e., the colon fails to relax). Although the ganglionic cells are absent, there is a proliferation of hypertrophied nerve fiber bundles. The most characteristic functional finding is the failure of internal anal sphincter to relax following rectal distention (i.e., abnormal rectoanal reflex). Mutations of the RET protooncogene (chromosome 10q.11.2) have been associated with Hirschsprung disease. It is associated clinically with a distended abdomen, inability to pass meconium, gushing of fecal material on a rectal digital exam, and a loss of peristalsis in the colon segment distal to the normal innervated colon.

An RNA tumor virus associated with the neurologic disease topical spastic paraparesis is (A) AKR leukemia virus (B) Human immunodeficiency virus (C) Human T lymphotrophic virus type 1 (D) Rous sarcoma virus

C. Human T lymphotrophic virus type 1 causes adult acute T-cell leukemia, but is also associated with tropical spastic paraparesis, a slowly progressive (10 or more years) neurologic disease endemic in some areas of the Caribbean.

A 16-year-old girl presents on May 10 in obvious emotional distress. On questioning, she relates that on May 1 she experienced sexual intercourse for the first time, without using any means of birth control. Most of her anxiety stems from her fear of pregnancy. What should the physician do to alleviate her fear? (A) Prescribe diazepam and wait to see if she misses her next menstrual period (B) Use ultrasonography to document pregnancy (C) Order a laboratory assay for serum hCG (D) Order a laboratory assay for serum progesterone (E) Prescribe diethylstilbestrol ("morningafter pill")

C. Human chorionic gonadotropin (hCG) can be assayed in maternal serum at day 8 of development and in urine at day 10. If this teenager is pregnant, the blastocyst would be in week 2 of development (day 10). Laboratory assay of hCG in either the serum or urine can be completed; however, serum hCG might be more reliable. It is important to note that if she is pregnant, she will not miss a menstrual period until May 15, at which time the embryo will be entering week 3 of development.

Which of the following is a short-acting acetylcholinesterase inhibitor? (A) Pyridostigmine (B) Bethanechol (C) Edrophonium (D) Scopolamine (E) Methantheline

C. In myasthenia gravis, autoantibodies develop to nicotinic acetylcholine receptors, causing impaired neuromuscular dysfunction, which results in muscular fatigue. This fatigue can be treated with acetylcholinesterase inhibitors. Edrophonium is the shortest-acting agent in this class and used to diagnosis this disorder, with such weakness immediately corrected with its use. Pyridostigmine is a longer acting agent used in the treatment of the disease. Bethanecol is a direct-acting muscarinic cholinergic agonist, whereas both scopolamine and methantheline are both muscarinic-receptor antagonists.

A prediabetic female 52-year-old patient with COPD, hypertension, and 2+ proteinurea is being treated with losartan. She complains that she has had a rash and a mild itching since she started losartan. Which of the following drugs would inhibit RAAS and be most suitable for this patient? (A) Captopril (B) Enalapril (C) Aliskirin (D) Epleronone (E) Spironolactone

C. Inhibition of RAAS has been shown to slow the progression of kidney damage in Type 2 diabetes. Since the patient has COPD, ACE inhibitors would be contraindicated because of the potential to worsen cough. She has demonstrated a sensitivity to an ARB; she is likely to have a sensitivity to other ARBs. Aliskerin is a small-molecule inhibitor of renin.

A neonate with very low Apgar scores dies 2 hours after birth. Autopsy reveals disseminated granulomatous lesions throughout; some are caseating, but they are not calcified. During her pregnancy, the mother most likely had a septicemia caused by (A) Escherichia coli (B) Group B streptococci (C) Listeria monocytogenes (D) Parvovirus B19 (E) Toxoplasma gondii

C. Listeria may cause in utero infections or may infect the baby during delivery. In utero Listeria infections are generally severe and are characterized by caseating, granulomatous lesions. Except for Toxoplasma and parvovirus, the other organisms listed cause infections acquired during birth. Toxoplasma usually manifests with calcified central nervous system lesions in the baby. Parvovirus B19 can cause anemia and hydrops fetalis when it crosses the placenta.

The role of macrophages in the immune response includes all of the following functions except (A) Antigen engulfment (B) Production of interleukin-1 (IL-1) (C) Production of IL-2 (D) Production of endogenous pyrogen (E) Presentation of antigen in context of class II histocompatibility antigens

C. Macrophages do not produce IL-2.

Which of the following virulence factors is produced by several genera of bacteria that are notable mucosal colonizers? (A) Elastase (B) Hemagglutinin (C) Immunoglobulin A protease (D) Mucinase

C. Organisms that colonize mucosal surfaces such as the Neisseria meningitidis and Streptococcus pneumoniae generally produce immunoglobulin A proteases.

Six 18-year-old women return from a recent Canadian camping trip with abdominal cramping, gas, pain, and diarrhea that is pale, greasy, and malodorous. They drank untreated stream water on the last 2 days of the trip after losing their water filter. What is the most likely causative agent? (A) Baylisascaris procyonis (B) Entamoeba histolytica (C) Giardia lamblia (D) Norwalk agent (E) Salmonella enteritidis (F) Vibrio parahaemolyticus

C. Pale, greasy, malodorous stools with malabsorption after drinking untreated stream or lake water strongly suggests a Giardia lamblia infection. The organisms can be detected most reliably by a fecal antigen test because they attach to the intestinal mucosa.

What respiratory infection is known to increase susceptibility to pneumonia caused by Streptococcus pneumoniae? (A) Epstein-Barr virus infection (B) Haemophilus influenzae type b or d infection (C) Influenza virus infection (D) Mycobacterium tuberculosis infection (E) Mycoplasma pneumoniae infection

C. Pneumococcal pneumonia is most frequent in patients with some damage to mucociliary elevators in the upper respiratory tracts. Antecedent measles, influenza virus infections, and alcoholism predispose patients to pneumococcal pneumonia.

In which one of the following fungal scalp infections is hair loss most likely to be permanent? (A) Anthropophilic tinea capitis (B) Black-dot tinea capitis of adults (C) Favus (tinea favosa) (D) Zoophilic tinea capitis

C. Scarring and permanent hair loss are most likely to occur with favus (tinea favosa).

A mother brings in her 2-year-old son to the clinic, stating that she "thinks her son can't hear her when she calls to him." She also indicates that he seems "slower mentally than the other kids" and he isn't "saying any works like Mommy." Her son has been in and out of the hospital a lot due to congenital heart defects and recently had his cataracts removed. She remarks that while she was pregnant toward the beginning she was little sick and "broke out in a rash," but she thinks that "was due to a new lotion she was using." Relevant physical exam findings include microcephaly, deafness, hepatosplenomegaly, blueberry muffin spots, and a hint of jaundice. Which of the following is the most likely diagnosis? (A) HIV infection (B) Herpes simplex virus infection (C) Rubella virus infection (D) Patau syndrome (E) Down syndrome

C. Rubella virus infection. Rubella virus (German measles; member of TORCH) belongs to the Togaviridae family, which are enveloped, icosahedral, positive, single-stranded RNA viruses. The rubella virus is transmitted to the fetus transplacentally. The risk of fetal rubella infection is greatest during the first month of pregnancy and apparently declines thereafter. Fetal rubella infection results in the classic triad of cardiac defects (e.g., patent ductus arteriosus, pulmonary artery stenosis, atrioventricular [AV] septal defects), cataracts, and low birth weight. With the pandemic of rubella in 1964, the complexity of this syndrome became apparent, and the term expanded rubella syndrome became standard. The clinical manifestations include intrauterine growth retardation (most common manifestation), hepatosplenomegaly, generalized adenopathy, hemolytic anemia, hepatitis, jaundice, meningoencephalitis, eye involvement (e.g., cataracts, glaucoma, retinopathy), bluish-purple lesions on a yellow, jaundiced skin ("blueberry muffin spots"), osteitis (celery stalk appearance of long bones), and sensorineural deafness. Exposure of pregnant women requires immediate assessment of their immune status. If the exposed pregnant woman is known to be immune (i.e., antibodies present), the woman can be assured of no risk. Postexposure prophylaxis of pregnant women with immune globulin (IG) is not recommended and should be considered only if abortion is not an option. Control measures for rubella prevention should be placed on immunization of children. Other members of TORCH include Toxoplasma gondii (a protozoan parasite), cytomegalovirus (CMV), herpes simplex virus, varicella zoster virus, Treponema pallidum (a spirochete), and hepatitis B virus. TORCH infections are caused by Toxoplasma (T), rubella (R), cytomegalovirus (C), herpes virus (H), and other (O) bacterial and viral infections that are grouped together because they cause similar clinical and pathological manifestations.

A patient developed an infection following catheterization. A biofilm is present on the catheter. What is the most likely causative agent? (A) Enterococcus faecalis (B) Staphylococcus aureus (C) Staphylococcus epidermidis (D) Streptococcus agalactiae (E) Streptococcus pneumoniae (F) Streptococcus viridans

C. Staphylococcus epidermidis is noted for its ability to secrete biofilms and adhere to intravenous lines. Streptococcus mutans (a Viridans streptococci) is also noted for the production of a dextran biofilm that adheres these organisms to dental surfaces and causes dental plaque.

A 10-day-old infant has high fever, nuchal rigidity, and photophobia. A lumbar puncture is performed to obtain cerebrospinal fluid (CSF) for analysis. Which of the following is the most likely CSF finding? (A) Decreased protein, decreased glucose (B) Decreased protein, increased glucose (C) Increased protein, decreased glucose (D) Increased protein, increased glucose (E) Normal protein, decreased glucose

C. The clinical history is strongly suggestive of pyogenic meningitis, the most common form of meningitis in the newborn. The diagnosis is confirmed by abnormalities in the CSF. These findings include increased pressure, cloudy appearance, markedly increased cell count with numerous neutrophils, increased protein, and decreased glucose (a useful rule is that the normal CSF glucose concentration is about two-thirds that of the serum glucose).

A 5 year old Somali immigrant girl presented to the local Family Medicine Clinic with a short (2 day) history of headache, earache, and swallowing difficulty. Physical examination showed bilateral swelling of the parotid glands. She had been in this country for 4 months and had started her childhood vaccinations 2 weeks ago. The genetic nature of the pathogen causing her disease is (A) Double-stranded DNA (B) Double-stranded RNA (C) Negative-sense RNA (D) Positive-sense RNA (E) Single-stranded DNA

C. The girl is infected with the mumps virus, a single-stranded negative-sense RNA virus.

An 18-year-old New York dirt bike racer, who recently raced for the first time in races along the Mississippi and also in the desert southwest, presents in September with cough, malaise, low-grade fever, myalgias, and chest pain. Rales are heard and respiratory infiltrates are noted on radiograph. Sputum stained with calcofluor white and viewed with an ultraviolet microscope shows hundreds of tiny yeast cells inside white cells. If cultured at room temperature, hyphae with small microconidia and tuberculate macroconidia would grow. What is the most likely causative agent? (A) Candida albicans (B) Coccidioides immitis (C) Histoplasma capsulatum (D) Aspergillus fumigatus (E) Mycoplasma pneumoniae (F) Streptococcus pneumoniae

C. This one was probably picked up in the Midwest, probably along the Mississippi. (They may have stirred up dried bird excreta dust from starlings or chickens or bats that contained Histoplasma capsulatum.) It can only be Histoplasma capsulatum from the description.

A 4-year-old girl has had multiple hospitalizations for pneumonia. Additionally, she is small for her age and has had symptoms of fat malabsorption. Her father refers to her as his "little potato chip" because she tastes salty when he kisses her. Chest radiograph demonstrates pulmonary infiltrates indicative of pneumonia. Which of the following is the most likely pathogen causing pulmonary infection in this patient? (A) Legionella pneumophila (B) Haemophilus influenzae (C) Pseudomonas aeruginosa (D) Staphylococcus aureus (E) Streptococcus pneumoniae

C. This patient has cystic fibrosis. Cystic fibrosis is an autosomal recessive disease caused by mutations in the CFTR gene, which encodes a protein that functions as a chloride channel. Pseudomonas aeruginosa is the most likely pathogen causing chronic pulmonary infection and pulmonary failure, and is the leading cause of death in patients with cystic fibrosis. P. aeruginosa is also (after S. aureus and E. coli) the leading cause of nosocomial (hospital-acquired) infections and a frequent cause of death from burns.

A 3-month-old, apparently female infant is evaluated for ambiguous genitalia. The clitoris appears large, and there are palpable masses in the inguinal region. Further investigation reveals the presence of both ovarian and testicular tissue. Which of the following terms is used to describe these findings? (A) Male pseudohermaphroditism (B) Female pseudohermaphroditism (C) True hermaphroditism (D) Turner syndrome (E) Klinefelter syndrome

C. True hermaphroditism requires the presence of both ovarian and testicular tissue, as in this case. The karyotype is either XX (with translocation of at least part of the Y chromosome to an X chromosome or to an autosome) or a mosaicism, such as XX/XXY.

Which of the following cells is capable of attacking a certain tumor cell spontaneously (i.e., without prior sensitization)? (A) Monocyte-macrophage (B) CD8+ cell (C) Mature B cell (D) Natural killer (NK) cell (E) Null (K) cell

D

Which of the following is an example of a primary active transport process? (A) Na+-glucose transport in small intestinal epithelial cells (B) Na+-alanine transport in renal proximal tubular cells (C) Insulin-dependent glucose transport in muscle cells (D) H+-K+ transport in gastric parietal cells (E) Na+-Ca2+ exchange in nerve cells

D [Chapter 1, II]. H+-K+ transport occurs via H+, K+-adenosine triphosphatase (ATPase) in the luminal membrane of gastric parietal cells, a primary active transport process that is energized directly by ATP. Na+-glucose and Na+-alanine transport are examples of cotransport (symport) that are secondary active transport processes and do not use ATP directly. Glucose uptake into muscle cells occurs via facilitated diffusion. Na+-Ca2+ exchange is an example of countertransport (antiport) and is a secondary active transport process.

Atropine causes dry mouth by inhibiting which of the following receptors? (A) á1 Receptor (B) â1 Receptor (C) â2 Receptor (D) Muscarinic receptor (E) Nicotinic receptor

D [Chapter 2, I C 2 b]. Atropine blocks cholinergic muscarinic receptors. Because saliva production is increased by stimulation of the parasympathetic nervous system, atropine treatment reduces saliva production and causes dry mouth.

After extensive testing, a 60-year-old man is found to have a pheochromocytoma that secretes mainly epinephrine. Which of the following signs would be expected in this patient? (A) Decreased heart rate (B) Decreased arterial blood pressure (C) Decreased excretion rate of 3-methoxy- 4-hydroxymandelic acid (VMA) (D) Cold, clammy skin

D [Chapter 2, I C; Table 2.2]. Increased circulating levels of epinephrine from the adrenal medullary tumor stimulate both á-adrenergic and â-adrenergic receptors. Thus, heart rate and contractility are increased and, as a result, cardiac output is increased. Total peripheral resistance (TPR) is increased because of arteriolar vasoconstriction, which leads to decreased blood flow to the cutaneous circulation and causes cold, clammy skin. Together, the increases in cardiac output and TPR increase arterial blood pressure. 3-Methoxy-4-hydroxymandelic acid (VMA) is a metabolite of both norepinephrine and epinephrine; increased VMA excretion occurs in pheochromocytomas.

After extensive testing, a 60-year-old man is found to have a pheochromocytoma that secretes mainly epinephrine. Symptomatic treatment would be best achieved in this man with (A) phentolamine (B) isoproterenol (C) a combination of phentolamine and isoproterenol (D) a combination of phentolamine and propranolol (E) a combination of isoproterenol and phenylephrine

D [Chapter 2, I; Table 2.3]. Treatment is directed at blocking both the á-stimulatory and â-stimulatory effects of catecholamines. Phentolamine is an á-blocking agent; propranolol is a â-blocking agent. Isoproterenol is a â1 and â2 agonist. Phenylephrine is an á1 agonist.

Which of the following responses occurs as a result of tapping on the patellar tendon? (A) Stimulation of Ib afferent fibers in the muscle spindle (B) Inhibition of Ia afferent fibers in the muscle spindle (C) Relaxation of the quadriceps muscle (D) Contraction of the quadriceps muscle (E) Inhibition of á-motoneurons

D [Chapter 2, III C 1; Figure 2.9]. When the patellar tendon is stretched, the quadriceps muscle also stretches. This movement activates Ia afferent fibers of the muscle spindles, which are arranged in parallel formation in the muscle. These Ia afferent fibers form synapses on á-motoneurons in the spinal cord. In turn, the pool of á-motoneurons is activated and causes reflex contraction of the quadriceps muscle to return it to its resting length.

A 17-year-old boy is brought to the emergency department after being injured in an automobile accident and sustaining significant blood loss. He is given a transfusion of 3 units of blood to stabilize his blood pressure. Before the transfusion, which of the following was true about his condition? (A) His total peripheral resistance (TPR) was decreased (B) His heart rate was decreased (C) The firing rate of his carotid sinus nerves was increased (D) Sympathetic outflow to his heart and blood vessels was increased

D [Chapter 3, IX C; Table 3.6; Figure 3.21]. The blood loss that occurred in the accident caused a decrease in arterial blood pressure. The decrease in arterial pressure was detected by the baroreceptors in the carotid sinus and caused a decrease in the firing rate of the carotid sinus nerves. As a result of the baroreceptor response, sympathetic outflow to the heart and blood vessels increased, and parasympathetic outflow to the heart decreased. Together, these changes caused an increased heart rate, increased contractility, and increased total peripheral resistance (TPR) (in an attempt to restore the arterial blood pressure).

Secretion of HCl by gastric parietal cells is needed for (A) activation of pancreatic lipases (B) activation of salivary lipases (C) activation of intrinsic factor (D) activation of pepsinogen to pepsin (E) the formation of micelles

D [Chapter 6, V B 1 c]. Pepsinogen is secreted by the gastric chief cells and is activated to pepsin by the low pH of the stomach (created by secretion of HCl by the gastric parietal cells). Lipases are inactivated by low pH.

Which step in the biosynthetic pathway for thyroid hormones produces thyroxine (T4)? (A) Iodide (I.) pump (B) I. ¨I2 (C) I2 + tyrosine (D) Diiodotyrosine (DIT) + DIT (E) DIT + monoiodotyrosine (MIT)

D [Chapter 7, IV A 4]. The coupling of two molecules of diiodotyrosine (DIT) results in the formation of thyroxine (T4). The coupling of DIT to monoiodotyrosine (MIT) produces triiodothyronine (T3).

Inhibition of which step in the steroid hormone synthetic pathway blocks the production of all androgenic compounds in the adrenal cortex, but not the production of glucocorticoids or mineralocorticoids? (A) Aldosterone synthase (B) Aromatase (C) Cholesterol desmolase (D) 17,20-Lyase (E) 5á-Reductase

D [Chapter 7, V A 1; Figure 7.11]. 17,20-Lyase catalyzes the conversion of glucocorticoids to the androgenic compounds dehydroepiandrosterone and androstenedione. These androgenic compounds are the precursors of testosterone in both the adrenal cortex and the testicular Leydig cells.

In a 35-day menstrual cycle, ovulation occurs on day (A) 12 (B) 14 (C) 17 (D) 21 (E) 28

D [Chapter 7, X E 2]. Menses occurs 14 days after ovulation, regardless of cycle length. Therefore, in a 35-day menstrual cycle, ovulation occurs on day 21. Ovulation occurs at the midpoint of the menstrual cycle only if the cycle length is 28 days.

A 17-year-old African-American girl recently had her ears pierced for the first time. She now presents to her primary care provider with the complaint of a large "tumor-like" growth in the immediate site of one of the piercings. This lesion is likely a (A) benign fibrous histiocytoma. (B) dermatofibrosarcoma protuberans. (C) fibroepithelial polyp. (D) keloid. (E) xanthoma.

D. A keloid is a result of excessive production of collagenous fibrous tissue and is characterized by a tumorlike scar consisting of dense bundles of structurally abnormal collagen. Keloids have a marked tendency to recur after resection. Propensity to keloid formation is markedly increased in persons of African lineage.

One advantage of live, attenuated vaccines is (A) They do not produce persistent low-grade infections (B) The viral strain does not revert to virulent forms (C) They have an unlimited shelf life (D) They induce a wide spectrum of antibodies

D. A live attenuated vaccine has the advantage of producing persistent infection, thus continuously stimulating the immune system with different antigens. A few viral strains in vaccines may revert to virulent form. A disadvantage is their limited shelf life.

An autopsy is performed on a 35-year- old African-American man who died after a brief illness characterized by papilledema, severe hypertension, left ventricular hypertrophy and failure, and renal dysfunction. The most likely findings in the kidney are (A) finely granular renal surface and hyaline arteriolosclerosis of afferent arterioles. (B) swollen, hypercellular, "bloodless" glomeruli. (C) nodular mesangial accumulations of basement membrane-like material and hyaline arteriolosclerosis of afferent and efferent arterioles. (D) surface covered with multiple petechial hemorrhages, hyperplastic arteriolosclerosis, and necrotizing glomerulitis. (E) swollen, pale kidneys and marked accumulation of lipid in convoluted tubules.

D. A rapidly fatal course with severe hypertension, left ventricular hypertrophy and failure, papilledema, and renal dysfunction is characteristic of malignant hypertension. This syndrome is most frequently seen in relatively young African-American men. The defining renal arteriolar lesion, malignant nephrosclerosis (hyperplastic arteriolosclerosis, fibrinoid necrosis, necrotizing arteriolitis), and the associated necrotizing glomerular lesion result in capillary rupture and the consequent "flea-bitten" appearance of the surfaces of the kidneys due to petechial hemorrhages.

A diagnosis of acute hematogenous osteomyelitis is made in a 5-year-old boy who had presented with the sudden onset of a high fever. He had been limping and had had erythema, edema, and pain around his right knee for several days. Which of the following is true of this condition? (A) It occurs with peak incidence in the elderly. (B) It most commonly affects the iliac crests. (C) Surgical incision and drainage is almost always required. (D) It is most commonly caused by Staphylococcus aureus. (E) It is more common in females.

D. Acute hematogenous osteomyelitis occurs with peak incidence in children, most commonly affects the metaphyses of long bones, and is more common in boys. In the acute stage, pyogenic osteomyelitis often resolves with antibiotic therapy. If the disorder is allowed to progress to necrosis and sequestrum formation, surgical intervention is usually required.

A 42-year-old woman presents with complaints of severe headaches, blurred vision, slurred speech, and loss of muscle coordination. Her last pregnancy 5 years ago resulted in a hydatidiform mole. Laboratory results show a high hCG level. Which of the following conditions is a probable diagnosis? (A) Vasa previa (B) Placenta previa (C) Succenturiate placenta (D) Choriocarcinoma (E) Membranous placenta

D. After a hydatidiform mole, it is very important to assure that all the invasive trophoblastic tissue is removed. High levels of hCG are a good indicator of retained trophoblastic tissue because such tissue produces this hormone. In this case, the trophoblastic tissue has developed into a malignant choriocarcinoma and metastasized to the brain, causing her symptoms of headache, blurred vision, and so on.

What is the mechanism of action of the aminoglycosides? (A) Damage to the membrane (B) Inhibit the DNA gyrase (C) Inhibit mycolic acid synthesis (D) Block initiation complex (E) Inhibit peptide chain elongation

D. Aminoglycosides bind to multiple sites on both the 30S and 50S ribosomes, thereby preventing the tRNA from forming initiation complexes.They are bactericidal for many aerobic Gram-negative bacteria.

A 35-year-old, HIV-positive man presents with productive cough, hemoptysis, fever, night sweats, weight loss, and anorexia. Chest radiograph demonstrates a cavitary lesion in the apex of the left lung. Sputum examination reveals acidfast bacilli. The patient is diagnosed with tuberculosis, in which the classic histologic feature is granulomatous inflammation. The epithelioid cells and multinucleated giant cells of this form of chronic inflammation are derived from which of the following? (A) Basophils (B) CD4ðq T lymphocytes (C) Eosinophils (D) Monocytes and macrophages (E) Plasma cells

D. Both epithelioid cells and multinucleated giant cells are modified macrophages. A macrophage is a mononuclear phagocyte that has migrated into tissue.

A 63-year-old woman develops metastatic colon cancer. The pathologist confirms that a biopsy specimen retrieved from a recent colonoscopy demonstrates that the tumor overexpresses epidermal growth factor receptor (EGFR). The oncologist decides to add a monoclonal antibody to EGFR to her treatment. Which of the following would be added? (A) Rituximab (B) Erlotinib (C) Gefitinib (D) Cetuximab (E) Traztuzamab

D. Cetuximab inhibits the EGF receptor by binding to the extracellular domain of the receptor. Other EGFR signaling inhibitors include erlotinib and gefinitib, although both of these molecules are orally active and penetrate the cell to perturb EGFR signaling from within the cell. Rituximab and Traztuzamab are both antibodies as well, but are used in the treatment of non-Hodgkin lymphoma and breast cancer, respectively.

Your patient has traveled to Haiti and took prophylactic chloroquine but stopped just as he left the region and still developed Plasmodium vivax malaria. What stage(s) was/were not eliminated because he stopped taking the drug early? (A) Erythrocytic schizonts/merozoites (B) Sporozoites (C) Bradyzoites (D) Liver schizonts/merozoites (E) Gametocytes

D. Chloroquine kills only the erythrocytic schizonts/merozoites, so you must take the chloroquine for 4 weeks after leaving the malarial area to allow all stages to continue past the liver stages into the sensitive forms.

Which of the following is useful in an acute gout attack? (A) Probenecid (B) Sulfinpyrazone (C) Allopurinol (D) Colchicine (E) Celecoxib

D. Colchicine is often used to treat an acute gouty attack. Probenecid (A) and sulfinpyrazone (B) reduce urate levels by preventing reabsorption of uric acid. These agents are used for chronic gout. Allopurinol (C) is a xanthine oxidase inhibitor; it is also used for the treatment of chronic gout. Celecoxib is a COX-2 inhibitor.

A 34-year-old man, who is positive for human immunodeficiency virus (HIV) and a resident of the Ohio-Mississippi River valleys region, developed a systemic illness characterized by hepatosplenomegaly and generalized lymphadenopathy. Which of the following microscopic descriptions of organisms from a resected lesion is most consistent with a presumptive diagnosis of disseminated histoplasmosis? (A) Budding yeast forms surrounded by empty haloes (B) Cup-shaped forms demonstrable by silver stain within a foamy amorphous intra-alveolar exudate (C) Intracavitary mycelial forms (D) Minute fungal yeast forms within phagocytes (E) Thick-walled spherules filled with endospores

D. Disseminated histoplasmosis is characterized by widespread dissemination of macrophages filled with fungal yeast forms.

A 3-year-old boy is brought to the clinic because of fever and "fussiness," and he is diagnosed as having acute otitis media. In this acute inflammatory reaction, which of the following cells would have reached the site of inflammation first? (A) Basophils (B) Lymphocytes (C) Monocytes-macrophages (D) Neutrophils (E) Plasma cells

D. During the first several hours of an inflammatory process, the predominant inflammatory cells are neutrophils. After 1 or 2 days, neutrophils are largely replaced by longer-lived monocytes-macrophages.

A 20-year-old woman presents at the emergency department with severe abdominal pain on the right side with signs of internal bleeding. She indicated that she has been sexually active without contraception and missed her last menstrual period. Based on this information, which of the following disorders must be included as an option in the diagnosis? (A) Ovarian cancer (B) Appendicitis (C) Normal pregnancy (D) Ectopic tubal pregnancy (E) Toxemia of pregnancy

D. Ectopic tubal pregnancy must always be an option in the diagnosis when a woman in her reproductive years presents with such symptoms. Ninety percent of ectopic implantations occur in the uterine tube. Ectopic tubal pregnancies result in rupture of the uterine tube and internal hemorrhage, which presents a major threat to the woman's life. The uterine tube and embryo must be surgically removed. The symptoms may sometimes be confused with appendicitis.

What is the most important characteristic of the causative agent leading to this particular transmission? (A) The virus is more common in the summer months (B) Glycosylated surface proteins are less sensitive to chlorination (C) Gram-negative bacteria are more resistant than Gram-positive bacteria (D) Naked viruses are more resistant to chlorination

D. First you had to know that adenoviruses are naked. If chlorination is not properly maintained in pools, the virus is not sufficiently damaged to inhibit binding or viability and it leads to spread in pools. Enveloped viruses are more easily damaged by chlorination,

A patient with hay fever switches from IgG formation to IgE production via (A) Interleukin (IL)-1 (B) IL-2 (C) IL-3 (D) IL-4 (E) IL-5

D. IL-4 causes the switch from IgG to IgE.

An immunodeficiency in a 5 year old boy with chronic granulomatous disease is (A) Inability of polymorphonuclear leukocytes (PMN) to ingest bacteria (B) Reduced levels of the fifth component of complement (C 5a) (C) Dysgammaglobulinemia (D) Inability of PMNs to kill already ingested bacteria

D. Patients with CGD lack the enzyme superoxide dismutase and have depressed hydrogen peroxide levels, functional entities in eliminating already ingested bacteria.

Mobile genetic elements that code for antibiotic resistance genes in bacteria but are incapable of self-replication are (A) Mesosomes (B) R factor (C) Temperate RNA phages (D) Transposons (E) Virulent DNA phages

D. Transposons are incapable of independent replication but may contain antibiotic resistance genes as well as insertion sequences that provide for transfer of genetic information to bacterial chromosomes or plasmids.

Which of the following complement components attaches to the crystallizable fragment of immunoglobulin M? (A) C2b (B) C4b2a (C) C2b (D) C5b (E) C1qrs

E

An emerging pathogen has been associated with serious lower respiratory tract disease. Both amprenavir and saquinavir have been demonstrated to be effective in treating the infection. What step in the pathogenesis of the infectious agent is affected by these drugs? (A) Gross-linking of the agents cell wall (B) Inhibition of the agents DNA replication (C) Inhibition of the agents reverse transcriptase (D) Premature release of agents mRNA from the translation complex (E) Processing of polyproteins

E. Both amprenavir and saquinavir are antivirals that inhibit the HIV protease, which is involved in the cleavage of HIV polyproteins.

A 60-year-old woman with a 10-year history of severe rheumatoid arthritis presents with splenomegaly and neutropenia. Which of the following is the most likely diagnosis? (A) Ankylosing spondylitis (B) Hypertrophic osteoarthropathy (C) Osteoarthritis (D) Rheumatoid arthritis (E) Felty syndrome

E. Felty syndrome is the combination of splenomegaly, neutropenia, and rheumatoid arthritis.

2,5A synthetase is induced by (A) Acyclovir (B) Amantadine (C) Cytarabine (D) Foscarnet (F) Ribavirin

E. Interferon, a host-encoded glycoprotein that is produced in response to virus infection, induces the synthesis of several antiviral proteins, including 2,5A synthetase.

A 2-year-old girl with a history of repeated pulmonary infections is found to have elevated chloride in a sweat test. An additional expected finding is (A) hypercalcemia. (B) hypotension. (C) increased metabolism. (D) renal failure. (E) steatorrhea.

E. Repeated pulmonary infections and a positive sweat test are characteristic of cystic fibrosis. In this condition, viscid secretions cause defective exocrine gland function. The lungs and pancreas are the most significant sites of involvement, and the disorder is marked by repeated bouts of pneumonia and by pancreatic failure with wasting and steatorrhea.

What compound is only found in Grampositive bacteria? (A) Capsule (B) Lipopolysaccharide (C) Outer membrane (D) Peptidoglycan (E) Teichoic acid

E. Teichoic and teichuronic acids, which are polymers containing ribitol or glycerol, are found in the cell walls or cell-wall membranes of Gram-positive bacteria.

The most likely diagnosis in a 24-yearold woman with the nephrotic syndrome, progressive azotemia, and thickening of glomerular capillary loops apparent on light microscopy is (A) Alport syndrome. (B) diabetic nephropathy. (C) focal segmental glomerulosclerosis. (D) lipoid nephrosis. (E) membranous glomerulonephritis.

E. The observation of thickened glomerular capillary loops apparent on light microscopy permits the diagnosis of membranous glomerulonephritis. This condition is most frequent in young women and is characterized clinically by the nephrotic syndrome and progressive azotemia.

A cofactor in Burkitt's lymphoma is (A) Cytomegalovirus infection (B) Hepatitis A virus infection (C) Hepatitis B virus infection (D) Influenza virus type A infection (E) Mycoplasma pneumoniae infection (F) Epstein-Barr virus infection

F. Antecedent Epstein-Barr virus infection in a malarial region is associated with Burkitt's lymphoma.

A male heterozygous for Rh factor mates with an Rh negative female. Genetic theory would predict that (A) No offspring would be Rh positive (B) 25% of the offspring would be Rh positive (C) 50% of the offspring would be Rh positive (D) 100% of the offspring would be Rh positive

c. Since the male is heterozygous for the Rh+ gene, the possibility exists for Rh+ offspring. The dominant male Rh+ gene can combine with each of the two female Rh- genes, resulting in the chance for 50% of the offspring being Rh+.

Your patient is compromised and has a life-threatening Histoplasmosis meningitis. Which of the following antifungal drugs is fungicidal and appropriate for initial systemic use until the infection only requires maintenance therapy? (A) Amphotericin B (B) Chloramphenicol (C) Ketoconazole (D) Griseofulvin (E) Itraconazole (F) Nystatin

iA. Both amphotericin B (the correct answer) and nystatin are fungicidal, but nystatin is not used for systemic infections. Chloramphenicol is bactericidal. Ketoconazole and itraconazole are both fungistatic, and griseofulvin is not used systemically and localizes only in keratinized tissues.

The genetic mechanism responsible for the conversion of a nontoxigenic strain of Corynebacterium diphtheriae to a toxigenic strain is (A) Lysogenic phage conversion (B) In vivo transformation (C) Reciprocal genetic recombination (D) Conjugation

A. A toxigenic strain of Corynebacterium diphtheriae is produced as a result of lysogenic phage conversion after a temperate bacteriophage infects a nontoxigenic strain of the organism.

A Peace Corps worker is sent back to the states from East Africa because he has had a very high fever. African sleeping sickness is diagnosed. How is it transmitted? (A) Tsetse fly bite (B) Invasion of skin in water (C) Respiratory droplets and direct mucosal contact (D) Mosquito bite (E) Reduviid bug bite (F) Sandfly bite

A. African trypanosomiasis (African sleeping sickness) is transmitted by tsetse flies.

A 25-year-old resident presents to the emergency room with severe eye pain. She has not been home for 2 days and has had the current pair of contacts continuously in her eyes for 6 weeks. What major protective mechanism of the eye could not keep up with such contaminated contacts, resulting in infection? (A) Lysozyme (B) B cells (C) NK cells (D) T cells (E) Teichoic acids

A. Lysozyme found in tears is one of our major protective mechanisms of the eye. It is, in general, more effective against Gram-positive organisms.

A noncompliant HIV-positive patient with a CD4+ cell count of 40/mm3 presents with a pulmonary infection caused by an organism that requires 4 weeks to grow on Lowenstein-Jensen medium. What is the best descriptor of the most likely causative agent? (A) Acid-fast organism (B) Dimorphic fungus (C) Filamentous fungus (D) Gram-positive coccus (E) Gram-negative coccus (F) Gram-negative rod

A. Mycobacterium avium-intracellulare or Mycobacterium tuberculosis, both acid-fast organisms, are the most likely causes of this pulmonary infection in a patient with acquired immunodeficiency syndrome. Both can be cultured on Lowenstein-Jensen but, in general, broth systems providing a similar high lipid content for making the mycobacterial cell wall have replaced Lowenstein-Jensen. Pneumocystis jiroveci, now considered a fungus, cannot be cultured on any medium.

What is the mechanism of action of â-blockers in heart disease? (A) Prolongation of AV conduction (B) Activation of the sympathetic system (C) Promotion of automaticity (D) Increase in heart rate (E) Arteriolar vasodilation

A. â-Blockers prolong AV conduction. They reduce sympathetic stimulation (B). These agents depress automaticity (C). â-Blockers decrease heart rate (D), and can cause arteriolar vasoconstriction (E).

A characteristic of IL-1 is (A) It inhibits T cells (B) It initiates the acute phase reactant response (C) Its synthesis is restricted to phagocytic cells (D) It suppresses tumor necrosis factor

B. Following tissue injury, IL-1 along with TNF-a initiates the acute phase reactant response.

Loose bacterial surface polysaccharide plays a critical role in (A) Gonorrhea (B) Meningitis with no underlying trauma (C) Mycoplasma pneumonia (D) Pyelonephritis (E) Urinary tract infection

B. For all major causative agents of meningitis that are extracellular (and these are the major ones), the capsule (polysaccharide) is important for successful hematogenous spread to the central nervous system; examples include Cryptococcus, meningitis-causing strains of Haemophilus influenzae (type b capsule), Neisseria meningitidis, and Streptococcus pneumoniae. All of these causative agents have polysaccharide capsules that allow survival in the bloodstream in an immunologically naive individual so they can reach the blood-brain barrier. The major virulence factors in urinary tract and ascending urinary tract infections are pili or adhesions that attach to the uroepithelium. Mycoplasma does not have a capsule.

A patient with acquired immunodeficiency syndrome has severe, nonresolving watery diarrhea. It is not gray or greasy. Acid-fast oocysts are seen in the stools. What is the most likely causative agent? (A) Enterohemorrhagic Escherichia coli (B) Cryptosporidium (C) Enterotoxic Escherichia coli (D) Giardia (E) Salmonella

B. Oocysts are only formed by protozoans eliminating A, C, and E. Acid-fast oocysts are only found in Cryptosporidium, Cyclospora, or Isospora infections.

An 18-year-old man presents with left knee pain he's had for several months. Knee radiograph demonstrates elevation of the periosteum of the bone, with areas of a "sunburst" appearance. What is the likely diagnosis? (A) Osteochondroma (B) Osteosarcoma (C) Giant cell tumor (D) Ewing sarcoma (E) Chondrosarcoma

B. The most common primary malignant neoplasm of bone is osteosarcoma. The classic radiographic findings are the Codman triangle (periosteal elevation by new bone formation) and the "sunburst" appearance (extension of tumor cells through the periosteum).

Anti-A isohemagglutinins are present in persons with which one of the following blood types? (A) Type A (B) Type B (C) Type AB

B. Type B (and type O) individuals carry anti-A antibodies.

Secretion of which of the following hormones is stimulated by extracellular fluid volume expansion? (A) Antidiuretic hormone (ADH) (B) Aldosterone (C) Atrial natriuretic peptide (ANP) (D) 1,25-Dihydroxycholecalciferol (E) Parathyroid hormone (PTH)

C [Chapter 3, VI C 4]. Atrial natriuretic peptide (ANP) is secreted by the atria in response to extracellular fluid volume expansion and subsequently acts on the kidney to cause increased excretion of Na+ and H2O.

Which of the following is a potential side effect of clozapine? (A) Cholestatic jaundice (B) QT prolongation (C) Agranulocytosis (D) Photosensitivity (E) Galactorrhea

C. Agranulocytosis occurs more frequently with clozapine than with other agents, requiring routine blood tests. It is the only agent that improves the negative symptoms of schizophrenia. Cholestatic jaundice and photosensitivity are common with chlorpromazine. Galactorrhea is a side effect of older high-potency agents that block dopamine. QT prolongation is a complication of agents such as thioridazine and ziprasidone.

Which of the following is an antineoplastic agent that has been shown to help patients with rheumatoid arthritis? (A) Valdecoxib (B) Ketorolac (C) Methotrexate (D) Entocort (E) Auranofin

C. From the presented list, only methotrexate is known to be an antineoplastic agent. This medication has been used successfully in rheumatoid arthritis and other rheumatologic conditions. Valdecoxib (A) is a COX-2 inhibitor that has been recently removed from the market. Ketorolac (B) is a powerful analgesic used for multiple autoimmune conditions. Entocort (D) is a glucocorticoid that can be used in some arthritides. Auranofin (E) is a gold compound that is rarely used anymore.

While playing in the forest, a 13-year-old boy was bitten in the lower left leg by a raccoon. What finding would suggest a serious neurologic disease could result from the bite? (A) Cowdry type A inclusion bodies in skin scrapings (B) Molluscum bodies on epithelial cells shed from raccoon's saliva (C) Negri bodies in skin scrapings (D) Presence of Downey cells in the boy's blood 2 weeks following the bite

C. Intracytoplasmic inclusion bodies called Negri bodies are found in nerve cells infected by the rabies virus, which can cause a serious neurological infection unless appropriately treated. (The patient should have been immediately treated prophylactically for rabies!)

Which of the following is the most sensitive type of serologic test? (A) Virus neutralization (B) Hemadsorption (C) Enzyme-linked immunosorbent assay (D) Nucleic acid hybridization

C. The ELISA test can measure nanogram amounts of hormones.

The T-cell antigen receptor is associated with which of the following characteristics? (A) It is a monomeric immunoglobulin M (B) It requires free antigen for triggering (C) It is associated with CD4 or CD8 (D) It is nonspecific

C. The T-cell antigen receptor is specific, responding only to antigen fragments bound to HLA. Choice A describes the B-cell receptor.

Joe has been HIV positive for the past 6 years. His disease has been slowly progressing. Mike, his only partner for the past 3 years, is free of the disease. A plausible explanation for Mike's lack of infection would be (A) His CD4/CD8 ratio is probably greater than 2 (B) He has a high concentration of NK cells that kill the virus (C) He lacks the coreceptor CXCR4 (D) He has built up a high titer of anti-GP160 antibody

C. The absence of the obligate coreceptor CXCR4 for HIV prevents viral entry into CD4+ cells. There is no evidence that NK cells, a high CD4/CD8 ratio, or anti GP-160 antibodies is protective.

Which of the following would be useful in the management of arrhythmia due to Wolf-Parkinson-White syndrome? (A) Digoxin (B) Lidocaine (C) Amiodarone (D) Adenosine (E) Atropine

D. Adenosine, a class V antiarrhythmic, is used for the treatment of paroxysmal supraventricular tachycardias, including those of Wolf-Parkinson-White syndrome. Digoxin (A) and amiodarone (C) can be used for the management of atrial fibrillation. Lidocaine is used in the treatment of many arrhythmias (B). Atropine is used for bradyarrhythmias.

A 27-year-old worker at a day care center has recently been feeling tired, has a slight fever, and has felt nauseated and vomited several times. Yesterday, she had abdominal pain and chills, and today she voided dark urine. Lab tests for serum enzymes indicated elevated AST and ALT. The genetic material of the virus most likely to cause her symptoms is (A) Double-stranded DNA (B) Double-stranded RNA (C) Single-stranded DNA (D) Single-stranded RNA

D. Hepatitis A virus (HAV) is the most likely virus to cause the reported symptoms, which indicate hepatitis rather than some other disease. HAV is a positive-sense ss RNA virus.

A new reverse transcriptase has been discovered. In addition to its effect on HIV disease, it should be checked for its potential in treating (A) Infectious hepatitis (B) Infectious mononucleosis (C) Measles (D) Serum hepatitis (E) Shingles

D. Serum hepatitis is the only disease listed in which the causative agent has a reverse transcriptase involved in replication.

What is the most likely causative agent? (A) Aspergillus (B) Fusarium (C) Acanthamoeba (D) Pseudomonas (E) Staphylococcus

D. Three of the causative agents—Aspergillus, Fusarium, and Pseudomonas—are involved in eye trauma. Pseudomonas is the most common associated with wearing extended wear contact lenses too long and not taking them out at night. Aspergillus and Fusarium are both fungi, so they can be eliminated on the basis of the description. Acanthamoeba is an amoeba and does cause eye infection but is generally associated with homemade contact lens solution. Staphylococcus is Gram positive and mainly associated with styes.

Which of the following hormones acts on the anterior lobe of the pituitary to inhibit secretion of growth hormone? (A) Dopamine (B) Gonadotropin-releasing hormone (GnRH) (C) Insulin (D) Prolactin (E) Somatostatin

E [Chapter 7, III B 3 a (1)]. Somatostatin is secreted by the hypothalamus and inhibits the secretion of growth hormone by the anterior pituitary. Notably, much of the feedback inhibition of growth hormone secretion occurs by stimulating the secretion of somatostatin (an inhibitory hormone). Both growth hormone and somatomedins stimulate the secretion of somatostatin by the hypothalamus.

An autoimmune disease characterized by absence of T cells, hypocalcemia, and tetany with lowered cell-mediated immunity is (A) Ulcerative colitis (B) Multiple sclerosis (C) Chronic granulomatous disease (D) Systemic lupus erythematosus (SLE) (E) Congenital thymic aplasia (DiGeorge syndrome)

E. Congenital thymic aplasia is caused by an unknown intrauterine injury to the third and fourth pharyngeal pouches around the 12th week of gestation. Failure of the parathyroid glands to develop results in tetany and hypocalcemia. Injury to the thymus accounts for the absence of T cells.

A 7-year-old boy is brought to the neurologist by his mother. She states that the boy's teacher says there are times in class when he stares "into space" and smacks his lips. In the office the boy has one such episode while having an electroencephalogram (EEG), which demonstrates a 3-per-second spike and wave tracing. Which drug is the best for this condition? (A) Phenytoin (B) Carbamazepine (C) Prednisone (D) Lorazepam (E) Ethosuximide

E. Ethosuximide is the drug of choice for absence seizures in children. Valproic acid has more side effects and therefore is a second-line drug. Prednisone is used in infantile seizures. Phenytoin and carbamazepine can be used in partial seizures or in tonic-clonic seizures. Lorazepam is often used in the treatment of status epilepticus.

A pathologist examining a histologic preparation from an autopsy finds a lesion with abundant granulation tissue. This finding is most likely to be indicative of which of the following? (A) Cat-scratch disease (B) Foreign body reaction (C) Histoplasmosis (D) Tuberculosis (E) Wound healing

E. Granulation tissue is formed in healing wounds and consists of young fibroblasts and newly formed capillaries. Cat-scratch disease, foreign body reaction, histoplasmosis, and tuberculosis are all well-known causes of granulomatous inflammation and have nothing to do with granulation tissue.

A 60-year-old woman with uterine bleeding is found to have endometrial hyperplasia and an ovarian tumor. Which is the most likely ovarian tumor? (A) Serous cystadenocarcinoma (B) Krukenberg tumor (C) Dysgerminoma (D) Teratoma (E) Granulosa cell tumor

E. Granulosa cell tumors are sex cord-stromal tumors that typically secrete estrogen. For this reason, endometrial hyperplasia or endometrial carcinoma may be a concomitant finding in women diagnosed with a granulosa cell tumor.

A typical IgG antibody molecule has all of the following characteristics except (A) It consists of at least two identical heavy and two identical light chains (B) It has two antigen binding sites (C) It has specificity for only one antigen (D) It is a glycosylated molecule (E) It has two constant domains on each of the heavy chains

E. The IgG molecule has three constant domains on the heavy chain.

A 25-year-old man is seen because he has a purulent urethral discharge. Microscopic examination reveals the presence of gram-negative diplococci within neutrophilic phagocytes. Which of the following structures is most resistant to infection with the likely organism responsible for this man's findings? (A) Urethra (B) Prostate (C) Seminal vesicles (D) Epididymis (E) Testes

E. The clinical description is most consistent with infection with Neisseria gonorrhoeae, which most often manifests in men as acute purulent urethritis. Without treatment, gonorrheal infection can extend to the prostate and seminal vesicles and sometimes to the epididymis. The testes are rarely involved.

A segmented, ambisense genome is found in (A) Respiratory syncytial virus (B) Parvovirus (C) Reovirus (D) Parainfluenza virus (E) Bunyavirus (F) Arenavirus

F. The short genome RNA molecule of arenavirus is ambisense-that is, the 30 half has negative sense and the 50 half has positive sense.

A hospitalized burn patient develops toxic shock syndrome. An enterotoxin secreting Staphylococcus is isolated from the burn lesion. This superantigen acts by (A) Inducing the secretion of high concentrations of cytokines (B) Blocking the epitope binding region of HLA molecules (C) Decreasing the numbers of polymorphonuclear cells (D) Binding to the Vb region on multiple B cells

A

Activation of which of the following receptors increases total peripheral resistance (TPR)? (A) á1 Receptor (B) â1 Receptor (C) â2 Receptor (D) Muscarinic receptor (E) Nicotinic receptor

A [Chapter 2, I C 1 a]. When adrenergic á1 receptors on the vascular smooth muscle are activated, they cause vasoconstriction and increased total peripheral resistance (TPR).

A 5-year-old boy has a severe sore throat, high fever, and cervical adenopathy. It is suspected that the causative agent is Streptococcus pyogenes. Which of the following is involved in producing fever in this patient? (A) Increased production of interleukin-1 (IL-1) (B) Decreased production of prostaglandins (C) Decreased set-point temperature in the hypothalamus (D) Decreased metabolic rate (E) Vasodilation of blood vessels in the skin

A [Chapter 2, VI C]. Streptococcus pyogenes causes increased production of interleukin-1 (IL-1) in macrophages. IL-1 acts on the anterior hypothalamus to increase the production of prostaglandins, which increase the hypothalamic set-point temperature. The hypothalamus then "reads" the core temperature as being lower than the new set-point temperature and activates various heat-generating mechanisms that increase body temperature (fever). These mechanisms include shivering and vasoconstriction of blood vessels in the skin.

Which of the following conditions decreases the likelihood of edema formation? (A) Arteriolar constriction (B) Venous constriction (C) Standing (D) Nephrotic syndrome (E) Inflammation

A [Chapter 3, VII C; Table 3.2]. Constriction of arterioles causes decreased capillary hydrostatic pressure and, as a result, decreased net pressure (Starling forces) across the capillary wall; filtration is reduced, as is the tendency for edema. Venous constriction and standing cause increased capillary hydrostatic pressure and tend to cause increased filtration and edema. Nephrotic syndrome results in the excretion of plasma proteins in the urine and a decrease in the oncotic pressure of capillary blood, which also leads to increased filtration and edema. Inflammation causes local edema by dilating arterioles.

Which diuretic is administered for the treatment of acute mountain sickness and causes an increase in the pH of urine? (A) Acetazolamide (B) Chlorothiazide (C) Furosemide (D) Spironolactone

A [Chapter 5, IX C 1; Tables 5.9 and 5.11]. Acetazolamide, a carbonic anhydrase inhibitor, is used to treat respiratory alkalosis caused by ascent to high altitude. It acts on the renal proximal tubule to inhibit the reabsorption of filtered HCO3 - so that the person excretes alkaline urine and develops mild metabolic acidosis.

Arterial pH of 7.29, arterial [HCO3 -] of 14 mEq/L, increased urinary excretion of NH4 +, and hyperventilation would be observed in a (A) patient with chronic diabetic ketoacidosis (B) patient with chronic renal failure (C) patient with chronic emphysema and bronchitis (D) patient who hyperventilates on a commuter flight (E) patient who is taking a carbonic anhydrase inhibitor for glaucoma (F) patient with a pyloric obstruction who vomits for 5 days (G) healthy person

A [Chapter 5, IX D 1; Table 5.9]. The blood values are consistent with metabolic acidosis, as would occur in diabetic ketoacidosis. Hyperventilation is the respiratory compensation for metabolic acidosis. Increased urinary excretion of NH4 + reflects the adaptive increase in NH3 synthesis that occurs in chronic acidosis. Patients with metabolic acidosis secondary to chronic renal failure would have reduced NH4 + excretion (because of diseased renal tissue).

A 53-year-old man with multiple myeloma is hospitalized after 2 days of polyuria, polydipsia, and increasing confusion. Laboratory tests show an elevated serum [Ca2+] of 15 mg/dL, and treatment is initiated to decrease it. The patient's serum osmolarity is 310 mOsm/L. The treatment drug is administered in error and produces a further increase in the patient's serum [Ca2+]. That drug is (A) a thiazide diuretic (B) a loop diuretic (C) calcitonin (D) mithramycin (E) etidronate disodium

A [Chapter 5, VI C]. Thiazide diuretics would be contraindicated in a patient with severe hypercalcemia because these drugs cause increased Ca2+ reabsorption in the renal distal tubule. On the other hand, loop diuretics inhibit Ca2+ and Na+ reabsorption and produce calciuresis. When given with fluid replacement, loop diuretics can effectively and rapidly lower the serum [Ca2+]. Calcitonin, mithramycin, and etidronate disodium inhibit bone resorption and, as a result, decrease serum [Ca2+].

End-organ resistance to which of the following hormones results in polyuria and elevated serum osmolarity? (A) Antidiuretic hormone (ADH) (B) Aldosterone (C) 1,25-Dihydroxycholecalciferol (D) Parathyroid hormone (PTH) (E) Somatostatin

A [Chapter 5, VII C; Table 5.6]. End-organ resistance to antidiuretic hormone (ADH) is called nephrogenic diabetes insipidus. It may be caused by lithium intoxication (which inhibits the Gs protein in collecting duct cells) or by hypercalcemia (which inhibits adenylate cyclase). The result is inability to concentrate the urine, polyuria, and increased serum osmolarity (resulting from the loss of free water in the urine).

Which of the following would be expected to increase after surgical removal of the duodenum? (A) Gastric emptying (B) Secretion of cholecystokinin (CCK) (C) Secretion of secretin (D) Contraction of the gallbladder (E) Absorption of lipids

A [Chapter 6, II A 2 a]. Removal of the duodenum would remove the source of the gastrointestinal (GI) hormones, cholecystokinin (CCK), and secretin. Because CCK stimulates contraction of the gallbladder (and, therefore, ejection of bile acids into the intestine), lipid absorption would be impaired. CCK also inhibits gastric emptying, so removing the duodenum should accelerate gastric emptying (or decrease gastric emptying time).

Which gastrointestinal secretion is hypotonic, has a high [HCO3 -], and has its production inhibited by vagotomy? (A) Saliva (B) Gastric secretion (C) Pancreatic secretion (D) Bile

A [Chapter 6, IVA 2-4 a]. Saliva has a high [HCO3 -] because the cells lining the salivary ducts secrete HCO3 -. Because the ductal cells are relatively impermeable to water and because they reabsorb more solute (Na+ and Cl-) than they secrete (K+ and HCO3 -), the saliva is rendered hypotonic. Vagal stimulation increases saliva production, so vagotomy (or atropine) inhibits it and produces dry mouth.

A 36-old-woman with galactorrhea is treated with bromocriptine. The basis for bromocriptine's action is by acting as an agonist for (A) dopamine (B) estradiol (C) follicle-stimulating hormone (FSH) (D) gonadotropin-releasing hormone (GnRH) (E) human chorionic gonadotropin (HCG) (F) oxytocin (G) prolactin

A [Chapter 7, III B 4 a, c (2)]. Prolactin secretion by the anterior pituitary is tonically inhibited by dopamine secreted by the hypothalamus. If this inhibition is disrupted (e.g., by interruption of the hypothalamic-pituitary tract), then prolactin secretion will be increased, causing galactorrhea. The dopamine agonist bromocriptine simulates the tonic inhibition by dopamine and inhibits prolactin secretion.

A 30-year-old woman has the anterior lobe of her pituitary gland surgically removed because of a tumor. Without hormone replacement therapy, which of the following would occur after the operation? (A) Absence of menses (B) Inability to concentrate the urine in response to water deprivation (C) Failure to secrete catecholamines in response to stress (D) Failure to secrete insulin in a glucose tolerance test (E) Failure to secrete parathyroid hormone (PTH) in response to hypocalcemia

A [Chapter 7, III B]. Normal menstrual cycles depend on the secretion of follicle-stimulating hormone (FSH) and luteinizing hormone (LH) from the anterior pituitary. Concentration of urine in response to water deprivation depends on the secretion of antidiuretic hormone (ADH) by the posterior pituitary. Catecholamines are secreted by the adrenal medulla in response to stress, but anterior pituitary hormones are not involved. Anterior pituitary hormones are not involved in the direct effect of glucose on the beta cells of the pancreas or in the direct effect of Ca2+ on the chief cells of the parathyroid gland.

Which of the following hormones causes constriction of vascular smooth muscle through an inositol 1,4,5-triphosphate (IP3) second messenger system? (A) Antidiuretic hormone (ADH) (B) Aldosterone (C) Dopamine (D) Oxytocin (E) Parathyroid hormone (PTH)

A [Chapter 7, III C 1 b]. Antidiuretic hormone (ADH) causes constriction of vascular smooth muscle by activating a V1 receptor that uses the inositol 1,4,5-triphosphate (IP3) and Ca2+ second messenger system. When hemorrhage or extracellular fluid (ECF) volume contraction occurs, ADH secretion by the posterior pituitary is stimulated via volume receptors. The resulting increase in ADH levels causes increased water reabsorption by the collecting ducts (V2 receptors) and vasoconstriction (V1 receptors) to help restore blood pressure.

Which of the following substances is converted to a more active form after its secretion? (A) Testosterone (B) Triiodothyronine (T3) (C) Reverse triiodothyronine (rT3) (D) Angiotensin II (E) Aldosterone

A [Chapter 7, IX A; Figure 7.16]. Testosterone is converted to a more active form (dihydrotestosterone) in some target tissues. Triiodothyronine (T3) is the active form of thyroid hormone; reverse triiodothyronine (rT3) is an inactive alternative form of T3. Angiotensin I is converted to its active form, angiotensin II, by the action of angiotensinconverting enzyme (ACE). Aldosterone is unchanged after it is secreted by the zona glomerulosa of the adrenal cortex.

Which step in the steroid hormone synthetic pathway is stimulated by angiotensin II? (A) Aldosterone synthase (B) Aromatase(C) Cholesterol desmolase (D) 17,20-Lyase (E) 5á-Reductase

A [Chapter 7, V A 2 b; Figure 7.11]. Angiotensin II increases production of aldosterone by stimulating aldosterone synthase, the enzyme that catalyzes the conversion of corticosterone to aldosterone.

Which of the following genetic mechanisms is responsible for the conversion of nontoxigenic strains of Corynebacterium diphtheriae to toxigenic strains? (A) Lysogenic phage conversion (B) In vivo transformation (C) Reciprocal genetic recombination (D) Conjugation

A. A toxigenic strain of Corynebacterium diphtheriae is produced as a result of lysogenic phage conversion after a temperate bacteriophage infects a nontoxigenic strain of the organism.

Following tooth extraction, which member of the gingival normal flora will grow contiguously into the damaged tissues, which produces tissue swelling, and into sinus tracts, with granules that may erupt to the surface? (A) Actinomyces israelii (B) Nocardia asteroides (C) Mycobacterium kansasii (D) Fusobacterium nucleatum

A. Actinomyces israelii grows contiguously in tissues, crossing anatomic barriers, causing lumpy jaw, which is characterized by swelling, and sometimes, sinus tract formation with granules. It often invades bone. Mycobacterium tuberculosis may be hematogenously spread to any tissue.

A 19-year-old boy is seen because of bilateral enlargement of the kidneys. His father, paternal uncle, and several other family members have similar abnormalities. Which of the following aneurysms is frequently associated with this disorder? (A) Berry aneurysm of the circle of Willis (B) Dissecting aneurysm (C) Fusiform aneurysm of the abdominal aorta (D) Saccular aneurysm of the thoracic aorta

A. Adult polycystic kidney is frequently associated with berry aneurysm of the circle of Willis, often in association with cysts in the liver or pancreas.

A mother brings in her 5-year-old son at the request of his summer camp counselor, who claims "the boy is hyperactive and doesn't seem to be as smart as the other kids." The mother agrees but has not done anything about until now. The mother indicates that her son was "pretty small at birth" and had a ventricular septal defect that was repaired soon after birth. The mother further tells you that "except for being small and the heart problem, everything else in the pregnancy was just fine; but you know I did have a drink every now and then." Relevant physical exam findings include hypertelorism, smooth philtrum, short palpebral fissures, flat nasal bridge, maxillary (midface) hypoplasia, and a thin upper lip. Relevant laboratory findings include magnetic imaging (MRI) showing holoprosencephaly. Which of the following is the most likely diagnosis? (A) Alcohol consumption during pregnancy (B) Thalidomide consumption duringpregnancy (C) Phenytoin consumption during pregnancy (D) Prader-Willi syndrome (E) Wolf-Hirschhorn syndrome

A. Alcohol consumption during pregnancy. Alcohol is an organic compound delivered to the fetus through recreational or addictive (i.e., alcoholism) drinking by pregnant women. This drug can cause fetal alcohol syndrome, which results in mental retardation, microcephaly, holoprosencephaly, limb deformities, craniofacial abnormalities (i.e., hypertelorism, smooth philtrum, short palpebral fissures, flat nasal bridge, maxillary (midface) hypoplasia, and a thin upper lip), and cardiovascular defects (i.e., ventricular septal defects). Fetal alcohol syndrome is the leading cause of preventable mental retardation. The threshold dose of alcohol has not been established, so "no alcohol is good alcohol" during pregnancy.

A 45-year-old man presents with breast enlargement, erectile dysfunction, and decreased libido. On physical examination, an intratesticular mass is palpated. Which of the following is the most likely diagnosis? (A) Androblastoma (Sertoli cell tumor) (B) Endodermal sinus tumor (C) Mature teratoma (D) Mixed germ cell tumor (E) Seminoma

A. Androblastoma (Sertoli cell tumor), a nongerm cell tumor derived from the sex cord, is most often benign. These tumors, along with Leydig (interstitial) cell tumors, may be hormone-producing, sometimes elaborating estrogens and presenting in males with gynecomastia and other feminizing effects.

A 43-year-old woman has a recent diagnosis of Hashimoto thyroiditis. This disease's autoimmune nature is suggested by which of the following features? (A) Association with human leukocyte antigen (HLA)-DR5 and HLA-B5 (B) Atrophy of thyroid follicles (C) Palpable thyroid (D) Presence of Hürthle cells (E) Signs and symptoms of hypothyroidism

A. Association with certain human leukocyte antigen (HLA) types, autoantibodies, and increased incidence in persons with other autoimmune disorders are frequent occurrences in autoimmune disorders. In addition, Hashimoto thyroiditis is characterized by dense lymphocytic infiltrates with germinal center formation, striking morphologic evidence of immune cell (B lymphocyte) participation.

There is an outbreak of watery diarrhea in 6 members of a party of 20 who ate at a Chinese restaurant the day before. Fried rice is implicated. What is the most likely causative agent? (A) Bacillus cereus (B) Giardia lamblia (C) Norwalk agent (D) Rotavirus (E) Salmonella enteritidis (F) Staphylococcus aureus

A. Bacillus cereus, found in rice, is not killed by steaming. The addition of eggs and other ingredients to make fried rice encourages growth if the fried rice is not held at a high temperature. Onset of watery diarrhea may occur within 2 hours or as long as 18 hours after consumption and is in response to the presence of toxin.

Which of the following would be found in the urine of a patient with multiple myeloma? (A) Bence-Jones proteins (B) Complement components (C) Heavy chains (D) Fc chains

A. Bence-Jones proteins are light chains found in the urine of myeloma patients whose plasma cells are secreting light chains in excess.

A febrile patient presents with diarrhea containing both blood and pus. The lab cultures on a special medium incubated at 42C under microaerophilic conditions grow a Gram-negative spiral-shaped organism that is isolated. What is the most likely genus? (A) Campylobacter (B) Helicobacter (C) Salmonella (D) Shigella (E) Vibrio

A. Both Campylobacter and Helicobacter are microaerophiles requiring a special medium. They are also both spiral Gram-negative bacteria but do not grow at 42C. Salmonella and Shigella are facultative anaerobes, so they would grow microaerophilically, but neither will grow at 42C. Vibrio is comma shaped and requires an alkaline medium and 35C to 37C for good growth.

What is the mechanism of action of Botulinum toxin? (A) Cleavage of terminal neuronal docking proteins preventing the release of acetylcholine (B) Blocking the transport of choline into neurons (C) Inhibition of choline acetyltransferase (D) Inhibition of acetylcholinesterase (E) Blocking the synapse of acetylcholine at the ganglia

A. Botulinum toxin (a metalloprotease) cleaves docking proteins and blocks exocytosis of acetylcholine from storage vesicles, producing a flaccid paralysis. Choline acetyltransferase, choice C, is an enzyme catalyzing synthesis of acetylcholine from an acetate and choline. Sodium-dependent transport of choline can be blocked by hemicholinium (choice B). Enzyme acetylcholinesterase is responsible for catalyzing hydrolysis of acetylcholine (choice D). Acetylcholine synapses at the ganglia of many neurons and tissues, and this step is not blocked by botulinum toxin (choice E).

CD8 is a surface membrane protein on T cells, which has the following characteristic (A) It recognizes class I human leukocyte antigens (HLA) (B) It recognizes class II HLA (C) It is a marker for T helper cells (D) It is strongly chemotactic

A. CD8 on the T-cell membrane binds to HLA class I antigens, initiating the process leading to CMI.

A female infant is born to a 16-year-old mother. The infant shows signs and symptoms of hepatosplenomegaly, jaundice, low birth weight, chorioretinitis, and microcephaly. The mother appears healthy and reports an unremarkable pregnancy except for a period during the 3rd month when she developed a fever, sore throat, and became extremely fatigued for a period of 10 days. The most likely causative pathogen for the infant's disease is (A) CMV (B) Listeria monocytogenes (C) Parvovirus B19 (D) Rubella virus (E) Treponema pallidum

A. CMV is the most likely pathogen; it was probably passed to the infant during a primary CMV infection of the mother during her first trimester of pregnancy.

A cystic ovarian mass was palpated in a 23-year-old woman. If X-ray films revealed calcifications in the mass, which of the following would be most likely? (A) Teratoma (B) Brenner tumor (C) Mucinous cystadenocarcinoma (D) Krukenberg tumor (E) Choriocarcinoma

A. Calcification within a cystic ovarian tumor in a young woman is most characteristic of mature teratoma of the ovary, a benign lesion and the most frequently occurring ovarian tumor.

A 35-year-old woman consults a gynecologist because she has postcoital vaginal bleeding. The Papanicolaou (Pap) smear is abnormal. Colposcopy and cervical biopsy lead to a diagnosis of carcinoma of the cervix. Which of the following is most characteristic of this disorder? (A) Association with human papillomavirus (HPV) infection (B) History of exogenous estrogen therapy (C) Most common gynecologic malignancy (D) Secretion of AFP (E) Spontaneous regression following menopause

A. Carcinoma of the cervix is associated with infection with certain serotypes of HPV. Other characteristics include a history of early sexual activity, squamous cell morphology, and frequent origin at the squamocolumnar junction. Leiomyomas (fibroids) may increase in size during pregnancy and decrease in size following menopause. Endometrial carcinoma is the most common gynecologic malignancy and is associated with hyperestrinism from estrogen therapy. Endodermal sinus (yolk sac) tumors produce á-fetoprotein.

A 17-year-old boy presents with right lower quadrant pain with guarding and rebound. A computed tomography (CT) scan demonstrates appendicitis, and he is taken to the operating room. What would be a good antibiotic to administer prophylactically before the surgery? (A) Cefazolin (B) Cefoxitin (C) Ceftriaxone (D) Aztreonam (E) Oxacillin

A. Cefazolin, a first-generation cephalosporin, is often used for surgical prophylaxis because it has activity against most gram-positive and some gram-negative organisms. Second-generation agents (cefoxitin) and third-generation agents (ceftriaxone) are not used because they have less gram-positive coverage. Aztreonam lacks activity against anaerobes and gram-positive organisms. Oxacillin is primarily active against staphylococci.

Which of the following is a common adverse effect of quinidine? (A) Cinchonism (B) Lupus-like syndrome (C) Seizures (D) Constipation (E) Pulmonary fibrosis

A. Cinchonism, or ringing in the ears and dizziness, is common after quinidine use. Lupus-like syndrome (B) can be observed after the use of procainamide. Seizures may occur with the use of lidocaine (C). Diarrhea can occur with the use of quinidine, not constipation (D). Pulmonary fibrosis is a long-term complication of using amiodarone (E).

A 10-day-old boy with projectile vomiting and a palpable midepigastric mass most likely has (A) congenital pyloric stenosis. (B) infantile polycystic kidney. (C) intussusception. (D) tracheoesophageal fistula. (E) Wilms tumor.

A. Congenital pyloric stenosis is an obstruction of the gastric outlet caused by hypertrophy of the pyloric muscularis. The hypertrophic muscle is often perceived as a palpable mass. The principal manifestation of this condition, more common in boys, is projectile vomiting, most often occurring in the first 3 to 6 weeks of life.

A 65-year-old man is seen for the recent onset of jaundice, weight loss, and anorexia. Abdominal examination reveals a distended, palpable gallbladder. Laboratory studies reveal conjugated hyperbilirubinemia, positive urine tests for bilirubin, and total absence of urobilinogen in the urine and stools. The probable diagnosis is (A) adenocarcinoma of the pancreas. (B) amebic abscess of the liver. (C) hepatic vein thrombosis. (D) hepatitis A infection. (E) hereditary spherocytosis.

A. Conjugated hyperbilirubinemia and positive urine tests for bilirubin are indicative of obstructive jaundice. The further finding of the complete absence of urine and stool urobilinogen indicates total common bile duct obstruction. Additionally, the palpable gallbladder (Courvoisier sign) strongly suggests that the etiology is a malignant tumor, such as adenocarcinoma of the head of the pancreas.

Dantrolene (A) Inhibits calcium release from the sarcoplasmic reticulum (B) Functions as a GABAB receptor agonist (C) Facilitates GABA activity in the central nervous system (CNS) (D) Reactivates acetylcholinesterase (E) Competitively inhibits the effects of acetylcholine

A. Dantrolene is used in the treatment of malignant hyperthermia and works by inhibiting the release of calcium from the sarcoplasmic reticulum. Baclofen, an antispasmatic used in the treatment of multiple sclerosis, inhibits synaptic transmission as a GABAB-receptor agonist. Benzodiazepines function to facilitate GABA activity in the central nervous system (CNS) and spinal cord. Pralidoxime (2-PAM) reactivates acetylcholinesterase. Nondepolarizing neuromuscular junction blockers such as atracurium competitively inhibit the effects of acetylcholine.

A father brings his 8-year-old son to the clinic and tells you that "he is bleeding a lot" and that "the kid comes in from playing with a lot of bruises." When talking to the son, he tells you that he is "one of the coolest kids in school" because "he can pull his skin out all over the place." Then, he proceeds to demonstrate this fact by pulling his ears out several inches away from his body. His father tells you that last year his son was rushed to the hospital and had emergency surgery because "he had a hole in his intestines." Relevant physical findings include highly elastic, velvety skin; fragile skin that bruises easily; and loose, unstable, hypermobile joints. Relevant laboratory findings include genetic testing revealing a mutation in the gene for peptidyl lysine hydroxylase. Which of the following is the most likely diagnosis? (A) Ehlers-Danlos syndrome (B) Marfan syndrome (C) Junctional epidermolysis bullosa (D) Osteogenesis imperfecta (E) Achondroplasia

A. Ehlers-Danlos syndrome. Ehlers-Danlos syndrome is an autosomal dominant genetic disorder involving the gene for peptidyl lysine hydroxylase, which is an enzyme necessary for the hydroxylation of lysine residues of collagen. It affects mainly type I and type III collagen. It is characterized by extremely stretchable and fragile skin, hypermobile joints, aneurysms of blood vessels, and rupture of the bowel.

A woman comes in with her 16-year-old daughter and states that her daughter "has not had a menstrual period yet." The daughter says that she is not sexually active and that she is not on any form of birth control. Relevant physical exam findings include ambiguous genitalia, amenorrhea, and early appearance of axillary and pubic hair. Relevant laboratory findings include elevated urinary 17-ketosteroids, elevated serum dehydroepiandrosterone (DHEA) sulfate, and normal or decreased 17-hydroxycorticosteroids, genetic testing reveals 46,XX genotype, and CT head scan reveals no sign of tumor. Which of the following is the most likely diagnosis? (A) Female pseudo-intersexuality (B) Turner syndrome (C) Complete androgen insensitivity (D) Pituitary tumor (E) Male pseudo-intersexuality

A. Female pseudo-intersexuality (FP) occurs when an individual has only ovarian tissue histologically and masculinization of the female external genitalia. These individuals have a 46,XX genotype. FP is most often observed clinically in association with a condition in which the fetus produces an excess of androgens (e.g., congenital adrenal hyperplasia [CAH]). CAH is caused most commonly by mutations in genes for enzymes involved in adrenocortical steroid biosynthesis (e.g., 21-hydroxylase deficiency, 11 â-hydroxylase deficiency). In 21-hydroxylase deficiency (90% of all cases), there is virtually no synthesis of cortisol or aldosterone, so intermediates are funneled into androgen biosynthesis, thereby elevating androgen levels. The elevated levels of androgens lead to masculinization of a female fetus. FP produces the following clinical findings: mild clitoral enlargement, complete labioscrotal fusion with a phalloid organ, or macrogenitosomia (in the male fetus). Because cortisol cannot be synthesized, negative feedback to the adenohypophysis does not occur, so adrenocorticotropic hormone (ACTH) continues to stimulate the adrenal cortex, resulting in adrenal hyperplasia. Because aldosterone cannot be synthesized, the patient presents with hyponatremia ("salt-wasting") with accompanying dehydration and hyperkalemia. Treatment includes immediate infusion of intravenous saline and long-term steroid hormone replacement, both cortisol and mineralocorticoids (9á-fludrocortisone). Although Turner syndrome is also a cause of primary amenorrhea, individuals with Turner syndrome have a 45,XO genotype. A pituitary tumor can be excluded due to negative CT scan findings.

A 48-year-old man with early-stage colon cancer undergoes a partial resection of the colon. Molecular analysis reveals that the tumor tissue harbors a mutation in codon 12 of the ras oncogene. The mutant gene codes for a ras protein product that has (A) decreased GTPase activity. (B) decreased reverse transcriptase activity. (C) increased protein phosphatase activity. (D) increased responsiveness to growth factors. (E) increased tyrosine kinase activity.

A. GTPase activity, which is required for inactivation, is decreased in mutant ras (p21) proteins. This change is mediated by reduced responsiveness to GTPaseactivating protein.

Which of the following would be an appropriate treatment to begin in a patient with Crohn's disease? (A) Glucocorticoids (B) Sulfasalazine (C) Bismuth subsalicylate (D) Octreotide (E) Loperamide

A. Glucocorticoids are used in the management of moderate cases of Crohn's disease. 5-Amino salicylic acid (5-ASA) compounds such as sulfasalazine are used in mild cases of ulcerative colitis. Octreotide is used for diarrhea secondary to increased release of gastrointestinal hormones. Bismuth subsalicylate and loperamide can be used in the treatment of uncomplicated diarrhea.

HLA II molecules play an important role in the immune response. One of their functions is (A) To present peptidic epitopes to CD 4+ T helper cells (B) To present peptidic epitopes to CD 8+ T helper cells (C) To interact with an epitope on the membranes of most nucleated cells (D) To interact with the gene regions: DP, DQ, DR

A. HLA-2 presents epitopes to CD4+ Th cells. HLA-1 presents epitopes to CD8+ Th cells.

A 4-year-old boy is seen after he suddenly develops a fever, abdominal pain and tenderness, hematuria, and palpable purpuric skin lesions on his buttocks and the extensor surfaces of the arms and legs. The most likely diagnosis is (A) Henoch-Schönlein purpura. (B) idiopathic (immune) thrombocytopenic purpura (ITP). (C) Kawasaki disease. (D) polyarteritis nodosa. (E) thrombotic thrombocytopenic purpura (TTP).

A. Henoch-Schönlein purpura is an IgA immune complex disease characterized by involvement of small vessels (venules, capillaries, arterioles) with multiple lesions, all about the same age, and is a form of hypersensitivity or leukocytoclastic vasculitis. The disorder may involve only the skin, presenting as palpable purpura, or it may involve a variety of other sites, including the glomeruli, gastrointestinal tract, lungs, or brain.

A 27-year-old man who recently arrived in the United States from Central America is found to have hypochromic microcytic anemia. Which of the following is the most likely cause of this finding? (A) Hookworm infestation (B) Fish tapeworm infestation (C) A strict vegetarian diet (D) Prolonged use of the antiseizure medication phenytoin (E) Anemia of chronic disease

A. Hypochromic microcytic anemia is most often associated with iron deficiency secondary to chronic blood loss. Hookworm infestation causes chronic blood loss and should not be confused with fish tapeworm infestation, which causes megaloblastic anemia. Folate deficiency with megaloblastic anemia can occur in severely malnourished persons (often alcoholics) or in association with increased demand for folate in pregnancy. Cobalamin (vitamin B12) deficiency megaloblastic anemia can occur in pernicious anemia, in strict vegetarians (vitamin B12 is only found in foods of animal origin), and in association with surgically induced intestinal blind loops overgrown with microorganisms with high avidity for cobalamin. The anemia of chronic disease is most often normochromic and normocytic, but can be hypochromic and microcytic. Usually, signs and symptoms of the underlying chronic disease are evident.

Which of the following substances is an endogenous pyrogen? (A) Interleukin 1 (IL-1) (B) IL-2 (C) IL-3 (D) IL-4 (E) IL-5

A. IL-1 is an endogenous pyrogen along with IL-6 and TNF-a.

Which of the following is the principal immunoglobulin (Ig) in exocrine secretions? (A) IgA (B) IgG (C) IgM (D) IgD (E) IgE

A. IgA is the principal Ig in exocrine secretions in the oral and gastric cavities.

A 40-year-old woman in the 30th week of gestation presents to the emergency room because she has vaginal bleeding and lower abdominal pain. The uterus is tender to palpation and there are signs of fetal distress. Because of hematuria and rectal bleeding, disseminated intravascular coagulation (DIC) is suspected. Which of the following findings would be most supportive of the diagnosis of DIC? (A) Increased fibrin degradation products (B) Decreased activated partial thromboplastin time (APTT) (C) Decreased prothrombin time (PT) (D) Normal thrombin time (E) Thrombocytosis

A. In DIC, widespread thrombosis activates the fibrinolytic system, with degradation of both fibrin and fibrinogen; therefore, fibrin and fibrinogen degradation products are markedly increased. Since platelets are consumed, thrombocytopenia, not thrombocytosis, is an expected finding. In addition, the consumption of coagulation factors results in prolongation of the PT, a measure of the extrinsic pathway of coagulation; the APTT, a measure of the intrinsic pathway of coagulation; and the thrombin time, a measure of fibrinogen concentration. The clinical history strongly suggests premature separation of the placenta (abruptio placentae), a well-known cause of DIC.

In the previous case, the substance found in the patient's CSF that gives the positive agglutination test also plays a role in the pathogenesis. What is that role? (A) It inhibits phagocytic uptake in the bloodstream of a nonimmune individual (B) It promotes invasion of the bloodstream (C) It enhances opsonization (D) It causes a strong inflammatory response loosening tight junctions, allowing invasion of the central nervous system (E) It allows binding to the vascular endothelium, triggering the uptake and passage into the CNS

A. In addition to a Gram stain of sediment of CSF, another rapid diagnostic tool for CNS infection is a panel of latex particle agglutination tests for common extracellular bacterial agents of meningitis. Because more than one strain of an agent may cause meningitis, each test utilizes known antibody or antibodies to a capsular polysaccharide(s) of a single bacterial causative agent of meningitis.

Goodpasture's syndrome is characterized by glomerulonephritis in genetically susceptible individuals. The responsible antigen is (A) A glycoprotein dispersed uniformly on the glomerular basement membrane (B) A circulating antigen-antibody complex (C) Rheumatoid factor (D) A complement (C05a) induced influx of neutrophils

A. In contrast to GLN in SLE (where circulating antigen-antibody complexes deposit randomly in the kidney in a lumpy bumpy pattern), the antigen in Goodpasture's syndrome is part of the glomerular basement membrane and its reaction with antibody produces a linear fluorescent pattern.

A 56-year-old man collapsed at work and died 20 minutes later in the emergency room while blood was being drawn. The patient's history revealed an episode of prolonged chest discomfort 3 months earlier. Which of the following is least likely? (A) Death from arrhythmia (B) Fibrotic scar in the ventricular septum (C) Loss of myocardial striations and beginning infiltration with neutrophils (D) Normal values for serum creatine kinase, troponin I, and myoglobin (E) Severe atherosclerotic narrowing of the anterior descending branch of the left coronary artery with overlying thrombus formation

A. In the first several hours after MI, the most common cause of death is arrhythmia. Although evidence of acute coronary artery obstruction may be found, morphologic myocardial changes and serum myocardial marker protein elevations are most often delayed for several hours. A myocardial fibrotic scar is evidence of an old prior MI.

Bacteria are protected from phagocytosis by (A) The capsule (B) Lipopolysaccharide (C) Lipoprotein (D) The outer membrane (E) Peptidoglycan

A. In the immunologically naive, bacterial capsules prevent the phagocytic uptake and, therefore, the killing of the bacterium.

Which of the following is true regarding infliximab? (A) It is a recombinant antibody to TNF-á (B) It is a humanized antibody to TNF-á (C) It is a fusion protein that binds to TNF-á receptor (D) It is a recombinant protein resembling IL-1 (E) It is a recombinant protein composed of a portion of LFA-3

A. Infliximab is a recombinant antibody to TNF-á; it has been successfully used in the treatment of Crohn's disease, rheumatoid arthritis, and some other autoimmune conditions. Choice B represents Humira, an agent also used for rheumatoid arthritis. Choice C is Etanercept, a subcutaneous agent approved for the treatment of rheumatoid arthritis. Choice D represents anakinra, an IL-1 blocker also used for rheumatoid arthritis. Choice E is alefacept, an agent used for psoriasis.

The antihypertensive action of which of the following drugs is due, in part, to diminished degradation of bradykinin? (A) Enalapril (B) Losartan (C) Aliskiren (D) Fludrocortisone (E) Furosemide

A. Inhibition of ACE reduces the formation of the active metabolite AgII, but also reduces the proteolytic breakdown of the potent vasodilator. This contributes significantly to the antihypertensive action of ACE inhibitors. The remaining drugs to not alter bradykinin metabolism.

Which agent is most likely to be used to treat hairy cell leukemia? (A) Interferon alfa-2b (B) Interleukin-2 (C) All-trans-retinoic acid (D) Rituximab (E) Daunorubicin

A. Interferon alfa-2b is used for the treatment of hairy cell leukemia, chronic myeloid leukemia, Kaposi sarcoma, and lymphomas. Interleukin-2 is used in the treatment of metastatic renal cell carcinoma. All-trans-retinoic acid is used to induce remission in M3 acute myelogenous leukemia (AML). Although sometimes used to treat hairy cell leukemia, rituximab is used primarily to treat CD20+ non-Hodgkin lymphoma. Daunorubicin is an antibiotic-type compound used in the treatment of some leukemias and lymphomas.

A 5-year-old child in a refugee camp in sub-Saharan Africa is seen by a volunteer doctor in the camp and is diagnosed as having severe protein-calorie malnutrition consistent with kwashiorkor. Which of the following is a characteristic of this disorder? (A) Impaired apolipoprotein synthesis (B) Increased fat storage in adipocytes (C) More severe than marasmus (D) Myocardial fatty change exceeds that in liver (E) Precursor to cirrhosis

A. Kwashiorkor is a form of protein-calorie malnutrition attributed to a relative lack of protein despite a diet relatively high in carbohydrates. Marasmus, in contrast, is more severe, and results from a major lack of calories from any source. Carbohydrate metabolites are converted to lipid, which is processed and stored by the liver; however, protein sources to serve as precursor amino acids for apolipoprotein synthesis are lacking. The result is hepatic fatty change, not cirrhosis.

The patient in the previous question returns to see his neurologist 3 years later. At this time the patient's symptoms have progressed, and he has now marked bradykinesia and a profound shuffling gait. In an attempt to prevent further deterioration, the neurologist prescribes a catechol-O-methyltransferase (COMT) inhibitor on top of the patient's levodopa and carbidopa. Which agent below is likely to have been added? (A) Entacapone (B) Selegiline (C) Ropinirole (D) Amantadine (E) Benztropine

A. Levodopa is metabolized, in part by catechol-O-methyltransferase (COMT); therefore, an inhibitor such as entacapone is an adjunct treatment for patients on levodopa. It does however increase the side effects including diarrhea, postural hypotension, nausea, and hallucinations. Selegiline is a monoamine oxidase inhibitor (MAOI) used in the treatment of Parkinson's disease. Ropinirole is a nonergot dopamine agonist used in early Parkinson's disease that may decrease the need for levodopa in later stages of the disease. Amantadine has an effect on the rigidity of the disease as well as the bradykinesia, although it has no effect on the tremor. Benztropine is muscarinic cholinoceptor antagonist used as an adjunct drug in Parkinson disease.

A 37-year-old woman recently had a large soft tissue sarcoma surgically resected from her retroperitoneum. She is to receive both radiation and chemotherapy, with cyclophosphamide as part of her chemotherapy. Which agent should be given in conjunction with this drug? (A) MESNA (B) Allopurinol (C) Leucovorin (D) Cilastatin (E) MOPP

A. MESNA is often given with cyclophosphamide and ifosfamide to help detoxify metabolic products that can cause hemorrhagic cystitis. Allopurinol is given with chemotherapy agents such as busulfan to reduce renal precipitation of urate. Leucovorin is given to rescue patients in the case of methotrexate toxicity. Cilastatin is an inhibitor of imipenem degradation. MOPP is a multidrug regimen (mechlorethamine, Oncovin (vincristine), procarbazine, and prednisone) used in the treatment of Hodgkin's disease.

A 22-year-old man comes into the office complaining of blurred vision. He states that he "has not had problems seeing before." He remarks that "his dad and sister had the same problem around his age." Relevant physical exam findings include long, spidery fingers (arachnodactyly), hypermobile joints, arm span much greater than body height, and dislocation of the lens (ectopia lentis). Relevant laboratory findings include CT scan showing a dilated aorta, and genetic testing reveals a mutation for the fibrillin-1 gene on chromosome 15q21.1. Which of the following is the most likely diagnosis? (A) Marfan syndrome (B) Klippel-Feil syndrome (C) Osteogenesis imperfecta (D) Ehlers-Danlos syndrome (E) Achondroplasia

A. Marfan syndrome. Marfan syndrome (MS) is an autosomal dominant genetic disorder caused by a mutation in the gene for the protein fibrillin-1 on chromosome 15q21.1, which is an essential component of elastic fibers. These individuals are unusually tall and have exceptionally long, thin limbs, ectopia lentis (dislocation of the lens), severe near-sightedness, and heart valve incompetence.

Which of the following is a noncompetitive NMDA receptor inhibitor that can be used to treat Alzheimer disease? (A) Memantine (B) Donepezil (C) Tacrine (D) Tolcapone (E) Pramipexole

A. Memantine is an NMDA-receptor inhibitor that is well tolerated and, although controversial, has been shown in some studies to slow the rate of cognitive decline in Alzheimer patients. Donepezil and tacrine are acetylcholinesterase inhibitors that have been shown, like memantine, to have similar controversial activities. Tolcapone is a catechol-O-methyltransferase (COMT) inhibitor rarely used in Parkinson disease because of the possibility of hepatic necrosis. Pramipexole is used as a dopamine receptor agonist in the management of Parkinson disease.

A distraught father comes in with his 10-year-old son, saying that his son began "turning blue" when he was out playing catch with him. His son remarked that he "just felt really tired" when he was running after the ball. The father is concerned that his son will not be able to play in the big game this weekend. Relevant physical exam findings include loud holosystolic ejection murmur on auscultation, cyanosis, and clubbing of fingernails. Relevant laboratory findings include echocardiogram showing right ventricular hypertrophy. Which of the following is the most likely diagnosis? (A) Membranous ventricular septal defect (B) Eisenmenger complex (C) Atrial septal defect (D) Patent ductus arteriosus (E) Coarctation of the aorta

A. Membranous ventricular septal defect (VSD). Membranous VSD is caused by faulty fusion of the right bulbar ridge, left bulbar ridge, and atrioventricular (AV) cushions. It results in a condition in which an opening between the right and left ventricles allows free flow of blood. A large VSD is initially associated with a left ¨ right shunting of blood, increased pulmonary blood flow, and pulmonary hypertension. One of the secondary effects of a large VSD and its associated pulmonary hypertension is proliferation of the tunica intima and tunica media of pulmonary muscular arteries and arterioles, resulting in a narrowing of their lumen. Ultimately, pulmonary resistance may become higher than systemic resistance and cause right ¨ left shunting of blood and cyanosis. At this stage, the characteristic of the patient has been termed the gEisenmenger complex.h This is the most common type of VSD. An atrial septal defect (ASD) would have a fixed, split S2, systolic ejection murmur. A patent ductus arteriosus (PDA), which is normally detected in infants, would have a continuous, machine-like murmur. Coarctation of the aorta would show a holosystolic murmur; however, there was no finding of a lack of a femoral pulse or rib notching.

A 23-year-old who just returned from a 2- week missionary trip to rural Haiti presents with a 2-day history of watery diarrhea. What is most likely causing his problem? (A) A bacterium producing an enterotoxin that is also an exotoxin (B) A bacterium producing an enterotoxin that is also an endotoxin (C) A bacterium that is invading tissue (D) A bacterium that is invading and producing an exotoxin

A. No mention of high fever or vomiting is included in the case, so the disease is most likely ''traveler's diarrhea'' commonly caused by enterotoxic E. coli. ETEC produces two exotoxins (LT and ST) both acting on the intestine, so they can also be called enterotoxins. It is common that when bacteria invade the gastrointestinal tract tissue, inflammation and prostaglandins are triggered, causing higher fever, abdominal pain, and diarrhea. Choice D is type I Shigella dysenteriae, which is both invasive and produces the shiga toxin.

A 29-year-old male who recently immigrated to the United States sees his physician for a burning sensation in his epigastrium. He is referred to a gastroenterologist who performs esophogastroduodenoscopy with biopsy that demonstrate ulcers with the presence of Helicobacter pylori. Use of which of the following regimens would provide the most effective and shortest treatment? (A) Pepto Bismol, clarithromycin, amoxicillin, and omeprazole (B) Pepto Bismol, metronidazole, tetracycline, and ranitidine (C) Clarithromycin, metronidazole, and omeprazole (D) Clathromycin, amoxicillin, and omeprazole (E) Pepto Bismol, metronidazole, and amoxicillin

A. Pepto Bismol, clarithromycin and amoxicillin, and omeprazole can be used for 7 days to eradicate Helicobacter pylori associated with peptic ulcer disease (metronidazole and tetracycline are additional choices for antibiotics). Choice E (Pepto Bismol, metronidazole, and amoxicillin) was the original triple therapy. The use of a proton pump inhibitor is usually preferred to an H2-blocker, as in Choice B. Regimens containing clarithromycin are used for cases of resistance to metronidazole.

A 37-year-old woman who is in her third trimester comes into your clinic complaining of bleeding that lasted for about "an hour or two." She remarks she noticed that the bleeding was "very bright red" in color but felt no noticeable pain. She said that she did nothing to cause the bleeding and "was concerned for the safety of her baby." Relevant physical exam findings include no abdominal or pelvic pain on palpation. Relevant laboratory findings include transvaginal ultrasound showing an intact, normally implanted placenta; however, the placenta was located in close proximity of the internal os. Which of the following is the most likely diagnosis? (A) Placenta previa (B) Placental abruption (C) Placenta accreta (D) Velamentous placenta (E) Membranous placenta

A. Placenta previa. Placenta previa occurs when the placenta attaches in the lower part of the uterus, covering the internal os. The placenta normally implants in the posterior superior wall of the uterus. Uterine (maternal) blood vessels rupture during the later part of pregnancy as the uterus begins to gradually dilate. The mother may bleed to death, and the fetus will also be placed in jeopardy because of the compromised blood supply. Because the placenta blocks the cervical opening, delivery is usually accomplished by cesarean section. This condition is clinically associated with repeated episodes of bright, red vaginal bleeding. Placental abruption would have shown a separation of the placenta and showed dark-red bleeding accompanied by abdominal pain. Placenta accreta would have shown the placenta implanted much deeper in the myometrium.

A frantic mother brings her newborn infant into the emergency room. You immediately notice that the infant is pale, irritable, diaphoretic, and dyspneic. A quick physical exam reveals hepatomegaly, absent femoral pulses, and pulses poor in all four extremities. The infant shows signs of heart failure and shock. An ECG shows pure right ventricular hypertrophy. A chest radiograph reveals generalized cardiomegaly with increased pulmonary vascular markings due to pulmonary venous congestion. The mother tells you that her baby was released from the hospital given a clean bill of health. Which of the following is the most likely diagnosis? (A) Postductal coarctation of the aorta (B) Membranous ventricular septal defect (C) Patent ductus arteriosus (D) Tetralogy of Fallot (E) Congenital diaphragmatic hernia

A. Postductal coarctation of the aorta. Postductal coarctation of the aorta occurs when the aorta is abnormally constricted distal to the origin of the left subclavian artery and inferior to the ductus arteriosus. Congenital coarctation reveals tunica intima hyperplasia and tunica media thickening that forms a posterolateral ridge that encircles the aortic lumen. Symptoms may occur in the neonatal period when the patent ductus arteriosus and the patent foramen ovale close, so that the entire cardiac output to the lower extremity must cross the narrowed aortic segment.

An 83-year-old male with multiple medical problems develops worsening constipation during his hospitalization for lower extremity cellulitis. The hospitalist decides to start giving a laxative. Which of the following is an appropriate choice and why? (A) Psyllium, because it is a bulkforming laxative good for chronic constipation (B) An osmotic agent, such as senna, which is administered rectally (C) A stool softener such as lactulose administered rectally (D) A stool softener such as methyl cellulose that inhibits water reabsorption (E) A salt-containing osmotic agent such as docusate, useful in preventing constipation

A. Psyllium and methyl cellulose are bulk-forming agents good for chronic constipation. The osmotic agent lactulose is given orally. Stool softeners such as docusate are useful in preventing constipation. Salt-containing osmotic agents such as magnesium sulfate are good for acute evacuation of the bowels. Senna is an irritant agent that stimulates intestinal motility.

A 65-year-old woman with a longstanding history of severe rheumatoid arthritis presents with proteinuria, hypertension, edema, and hypoalbuminemia. Which of the following is the most likely diagnosis? (A) Renal amyloidosis (B) Diabetic nephropathy (C) Membranous glomerulonephritis (D) Minimal change disease (E) Poststreptococcal glomerulonephritis

A. Renal amyloidosis can occur in both primary and secondary amyloidosis. In the latter, the most frequently occurring underlying illness is rheumatoid arthritis.

A 4 year old boy has a yearlong history of repeated staphylococcal and streptococcal infections. He was referred to the genetic counseling unit where he was found to have depressed superoxide dismutase activity. Treatment with which of the following is most likely to be successful? (A) Interferon gamma (B) CD 4+ T helper cells (C) Insertion of the gene for adenosine deaminase (D) Transforming growth factor b

A. Repeated infections with depressed superoxide activity characterize chronic granulomatous disease. Interferon-g has been successful in treatment. Loss of adenine deaminase is the lesion found in SCID patients. TGF-b inhibits T-cell and B-cell proliferation.

Following neonatal thymectomy you would expect (A) Depletion of the periarteriolar region of the spleen (B) Elimination of germinal center formation (C) An increase in ability to reject skin grafts (D) An increase in autoimmunity

A. T cells home mainly in the periarteriolar region of the spleen and lymph nodes. B cells and macrophages make up the major cell populations in the germinal centers.

A 16-month-old boy has had recurrent bouts of cyanosis since birth. His parents tell you that "he cannot keep up with the other children of his age." The parents indicate that their boy frequently turns blue, breathes heavily on exertion, and sometimes experiences these difficulties for no reason. On many occasions, they observed their son in a squatting position. Relevant physical exam findings include systolic ejection murmur, cyanosis, clubbing of the fingernails, and a parasternal heave. Relevant laboratory findings include radiographs showing an enlarged right ventricle and "boot-shaped" heart, electrocardiogram showing right ventricular hypertrophy, and echocardiogram showing pulmonary stenosis, right ventricular hypertrophy, overriding aorta, and a ventricular septal defect. Which of the following is the most likely diagnosis? (A) Tetralogy of Fallot (B) Tricuspid atresia (C) Total anomalous pulmonary venous return (D) Transposition of the great arteries (E) Persistent truncus arteriosus

A. Tetralogy of Fallot. Tetralogy of Fallot (TF) is caused by an abnormal neural crest cell migration such that there is skewed development of the aorticopulmonary (AP) septum. TF results in a condition in which the pulmonary trunk obtains a small diameter while the aorta obtains a large diameter. TF is characterized by four classic malformations: pulmonary stenosis, right ventricular hypertrophy, overriding aorta, and a ventricular septal defect (VSD). Note the mnemonic PROVE. TF is associated clinically with marked cyanosis, in which the clinical consequences depend primarily on the severity of the pulmonary stenosis.

An outbreak of respiratory disease that leads to pneumonia occurs in a nursing home. It is characterized by an interstitial x-ray pattern and both amantadine and zanamivir are effective treatments. What is the causative agent? (A) Influenza A virus (B) Influenza B virus (C) Legionella pneumonia (D) Metapneumonovirus (E) Mycoplasma pneumonia

A. The M2 protein of the influenza A virus forms a protein channel that facilitates uncoating of the virus. Amantadine inhibits channel function of influenza A virus, but not influenza B virus. Zanamivir inhibits the viral neuraminidase of both influenza A and B viruses.

All Shigellae have some invasive capability limited to mucosa of the ileum and colon but some Shigella dysenteriae strains causes more severe disease. Why? (A) They produce a toxin similar to enterohemorrhagic E. coli (B) These strains are motile (C) They have greater resistance to stomach acid (D) They produce a toxin similar to enterohemorrhagic E. coli (E) They produce a capsule (Vi antigen)

A. The Shiga toxin-producing strains nick the 60S ribosomes of the human cells, causing severe dysentery and may also cause hemolytic uremic syndrome.

A volunteer physician in a village in Rwanda sees a 45-year-old man who has an aortic diastolic murmur and a "water- hammer" pulse. Blood pressure is 200/70. Which of the following associated abnormalities is most likely? (A) Aneurysm of ascending aorta (B) Cerebral infarct (C) Gangrene of small bowel (D) Mural thrombosis (E) Myocardial infarct

A. The combination of aortic diastolic murmur, "water-hammer" pulse, and wide pulse pressure is an indicator of aortic valve insufficiency. Although now rare in the United States, tertiary syphilis remains the most common cause of this abnormality in many parts of the world. Aneurysm of the ascending aorta commonly accompanies this valvular lesion. The other listed choices are all complications of atherosclerosis, which infrequently involves the ascending aorta.

A 19-year-old pregnant student presents at her health service allergy clinic with rhinorrhea, sneezing, itching, and conjunctivitis. A RAST test finds IgE antibodies to ragweed antigens. A characteristic of this condition is (A) IL-4 mediated IgE synthesis has increased (B) Desensitization can occur with repeated injection of small doses of IgE (C) Drugs that elevate cGMP can reduce symptoms (D) Placental transfer of ragweed antibody can sensitize her unborn child

A. The cytokine IL-4 is necessary for the switch to IgE synthesis. Desensitization can occur with repeated injections of allergen, not IgE. IgE antibody does not cross the placenta. Drugs that elevate cGMP enhance symptoms.

In a high-frequency recombination (Hfr) cross with an F- bacterial cell, each having a single DNA molecule, what is the most likely outcome? (A) Bacterial genes will be transferred from the Hfr cell to the Fcell, but there will be no change in the ''sex'' of either cell (B) Some genes will be transferred and the recipient cell will become Hfr (C) Only plasmid genes will be transferred (D) Each cell may acquire genes from the other

A. The donor cell, which transfers part of one of the two strands of its DNA, will duplicate any areas of single-stranded DNA and so will not change genotype, including its ''maleness.'' Only a portion of the integrated fertility factor and some bacterial genes on that strand of DNA will be transferred, so the recipient cell may pick up some new bacterial genes. But because the last thing to be transferred would be the rest of the fertility factor, the cell almost never becomes Hfr.

A 20-year-old man is hospitalized with fever, shaking chills, and widespread cutaneous hemorrhages. He complains of severe headache, and nuchal rigidity is noted on physical examination. Examination of the peripheral blood and CSF reveals gramnegative diplococci within neutrophils. A well-known complication of this disorder is hemorrhage into the (A) adrenal cortex. (B) anterior pituitary. (C) brain stem. (D) pancreas. (E) subarachnoid space.

A. The findings are characteristic of meningococcemia with meningococcal meningitis. A well-recognized complication of meningococcemia is the Waterhouse- Friderichsen syndrome, which is catastrophic adrenal insufficiency and vascular collapse caused by hemorrhagic necrosis of the adrenal cortex, often with associated DIC.

A male infant was seen for recurrent bacterial infections beginning at age 6 months. Immunoglobulin assay reveals absent IgG. An additional expected finding is (A) absence of germinal centers in the lymph nodes. (B) autosomal recessive inheritance. (C) decreased CD4ðq to CD8ðq T lymphocyte ratio. (D) defective leukocytic bacterial killing. (E) impaired phagocytosis. In this infant, the period of well-being for the first 6 months of life is best explained by (A) antibacterial substances supplied by breast-feeding. (B) deficient opsonization due to immaturity of complement synthesis. (C) delayed responsiveness of lymphocytes to mitogenic stimuli. (D) protection by maternal antibodies. (E) need for a viral infection to trigger immune destruction of thymic tissue.

A. The findings described are characteristic of congenital agammaglobulinemia of Bruton, an X-linked disorder characterized morphologically by agammaglobulinemia, absence of plasma cells, and absent or poorly defined germinal centers in the lymph nodes. Leukocyte functions, such as phagocytosis and bacterial killing, are unimpaired. T lymphocytes are unaffected. D. Maternal antibodies provide passive immunization and protection from bacterial infection during the first months of life in children with congenital agammaglobulinemia.

Following a 12-year-old girl's birthday party, which took place in her backyard and involved summer sport games and swimming, several parents complained that their children developed red, itchy eyes, a sore throat, and a slight fever. The children's eyes were examined and no petechial hemorrhages were observed, but preauricular adenopathy was present. What is the most likely causative agent? (A) Adenovirus (B) HSV-1 (C) Influenza virus (D) Moraxella catarrhalis (E) Streptococcus pneumoniae

A. The given clinical symptoms suggest viral pharyngoconjunctival fever rather than bacterial conjunctivitis, where pus would have been present and preauricular nodes absent. Adenovirus frequently causes a pharyngoconjunctival fever that can be transmitted from person to person in contaminated swimming pools.

Repeated Neisseria meningitidis septicemias in an individual should raise physician awareness of what underlying condition? (A) C5-C8 deficiencies (B) Chronic hepatitis B infection (C) Ketosis-prone diabetes (D) Multiple myeloma (E) Severe neutropenia

A. The killing of Neisseria meningitidis organisms is primarily dependent on complement-mediated cell lysis. Patients with genetic deficiencies in C5 to C8 cannot carry out complement-mediated lysis of bacterial cells and have repeated septicemias with N. meningitidis.

A 54-year-old man presents with a painless unilateral swelling just anterior to the ear. Biopsy of the mass suggests it is a pleomorphic adenoma, and he is scheduled for surgery to remove the mass. Which of the following is true of this tumor? (A) Often recurs (B) Is less prevalent than Warthin tumors of the salivary gland (C) Totally resectable (D) Results from mumps infection (E) Often results in Horner syndrome

A. The majority of salivary gland tumors occur in the parotid, and most are pleomorphic adenomas. This tumor most often presents as a painless mass just anterior to the ear. Due to its proximity to the facial nerve, it is often not completely resectable and therefore tends to recur. Mumps infection can lead to parotid swelling that is often bilateral and painful. Horner syndrome is a consequence of some lung tumors, not of salivary gland tumors.

A rubbery, well-encapsulated, freely movable mass is found in the breast of a 20-year-old woman. Which of the following is most likely? (A) Fibroadenoma (B) Fibrocystic disease (C) Intraductal papilloma (D) Paget disease (E) Phyllodes tumor

A. The most common cause of a breast mass in women younger than age 25 is fibroadenoma. Characteristically, this benign tumor presents as a firm, rubbery, painless, well-circumscribed lesion.

A 55-year-old alcoholic man presents with jaundice and recent weight loss. Imaging reveals a large mass in the head of his pancreas. Biopsy reveals an infiltrate of small malignant glands set in a background of prominent desmoplasia. Which of the following is true of this cancer? (A) PanIN represents a precursor lesion. (B) It most commonly arises in the tail of the pancreas. (C) It is not associated with smoking. (D) It is invariably associated with heavy drinking. (E) Ovarian-type stroma is common.

A. The patient has pancreatic adenocarcinoma, a common malignancy of the pancreas. It is more common in smokers. Although chronic pancreatitis is a risk factor, there is not a straightforward link with alcohol use and indeed, the chronic pancreatitis most often associated with pancreatic cancer is not the conventional type observed in alcoholics. Pancreatic adenocarcinomas are most common in the head of the pancreas and are thought to arise in pancreatic dysplasia known as PanIN. PanIN usually shows mutations in KRAS while subsequent mutations, often involving p53, p16, and DPC4/SMAD4 are required for full-blown invasive cancer. Mucinous cystic neoplasms of the pancreas are relatively indolent tumors which are most common in the pancreatic tail and which usually show ovarian-type stroma, which is not a feature of pancreatic adenocarcinoma.

An 11 day old neonate has been brought to his pediatrician because infant has developed a sharp cough. Physical examination and a chest film confirm pneumonia, yet no organisms are seen on Gram stain of the pus. What is the most likely causative agent? (A) An obligate intracellular organism unable to make its own ATP (B) A diplococcus that has a high potential to cause blindness as well as the pneumonia (C) An organism that has no peptidoglycan or other cell-wall polymer (D) An obligate intracellular organism infective vascular endothelium (E) An organism that may be demonstrated by acid-fast stain and is largely intracellular

A. The presentation is most likely Chlamydia trachomatis conjunctivitis and pneumonia. This obligate intracellular organism is unable to make its own ATP. Choice B (gonococcus) should have been eliminated on Gram stain, and most would have been intracellular but not in phagosomes. Choice C is mycoplasma, which is not intracellular and rarely would cause pneumonias in neonates this young. Choice D would be a rickettsia, which is both unlikely and not a good fit for the disease. Choice E (mycobacterium) also is unlikely to cause pneumonia in someone this young and is not known to cause eye infections producing pus.

A 17-year-old girl develops an acute syndrome characterized by low-grade fever, lassitude, pharyngitis, generalized lymphadenopathy, and a palpable liver and spleen. A peripheral blood smear reveals the presence of "atypical lymphocytes." A heterophil test for antibodies reacting with sheep erythrocytes is negative. Infection with which one of the following viruses is most likely? (A) Cytomegalovirus (CMV) (B) Epstein-Barr virus (EBV) (C) Measles virus (D) Papillomavirus (E) Parvovirus

A. The scenario is strongly suggestive of infectious mononucleosis—a benign, self-limited lymphoproliferative disorder that is most often caused by EBV infection; however, the heterophil test for sheep red cell agglutinins is negative, and so-called heterophil-negative infectious mononucleosis is most often caused by CMV.

An 18-year-old male college student came to the Student Health Services complaining of a sore throat, fatigue, and difficulty in swallowing. A physical examination showed an enlarged spleen, a palpable, tender liver, and swollen cervical lymph nodes. His blood is positive for heterophile antibody. What would you expect to see in a blood smear? (A) Atypical T cells called Downey cells (B) Gram-positive rods (C) Gram-negative rods (D) Mononuclear cells with Cowdry type A inclusions (E) Multinucleated giant cells

A. The young man has infectious mononucleosis caused by EBV, which produces a heterophile antibody that is the basis of the Monospot test, and the Downey cells, which are reactive T cells.

Tiagabine works by (A) Inhibiting GABA uptake by inhibiting the GABA transporter (B) Increasing GABA by stimulating its release from neurons (C) Increasing GABA-stimulated chloride channel opening (D) Prolonging GABA-induced channel opening (E) Blocking T-type calcium channels

A. Tiagabine is an anticonvulsant used in conjunction with drugs such as phenytoin. Its mechanism is related to its ability to inhibit GABA transport into the cell, thereby decreasing GABA uptake. Gabapentin works by stimulating the release of GABA from neurons. Benzodiazepines function to increase GABA-stimulated chloride channel opening, whereas barbiturates prolong GABA-induced chloride channel opening. Ethosuximide works by blocking T-type calcium channels.

A 60-year-old man presents with painless hematuria. Transurethral biopsy of the bladder is performed, and histologic evaluation demonstrates the presence of carcinoma of the urinary bladder. Which of the following is the most common type of carcinoma of the urinary bladder? (A) Transitional cell carcinoma (B) Squamous cell carcinoma (C) Adenocarcinoma (D) Clear cell carcinoma (E) Small cell carcinoma

A. Transitional cell carcinoma is the most common tumor of the urinary collecting system and can occur in renal calyces, pelvis, ureter, or bladder.

A 20-year-old woman with episodic hypertension is found to have a marked increase in urinary VMA. Several close relatives have had vascular tumors of the eye and cysts of the liver, kidney, and pancreas. Which of the following familial syndromes is most likely? (A) von Hippel-Lindau disease (B) von Recklinghausen disease (C) Marfan syndrome (D) Familial hypercholesterolemia (E) Tuberous sclerosis

A. von Hippel-Lindau disease is an autosomal dominant disorder characterized by multiple vascular tumors and multiple cysts of the liver, kidney, and pancreas. The renal cysts have a high potential for malignant transformation. Retinal and CNS hemangioblastomas are characteristic, as are pheochromocytomas.

Which of the following substances would be expected to cause an increase in arterial blood pressure? (A) Saralasin (B) V1 agonist (C) Acetylcholine (ACh) (D) Spironolactone (E) Phenoxybenzamine

B [Chapter 3, III E; VI B]. V1 agonists simulate the vasoconstrictor effects of antidiuretic hormone (ADH). Because saralasin is an angiotensin-converting enzyme (ACE) inhibitor, it blocks the production of the vasoconstrictor substance angiotensin II. Spironolactone, an aldosterone antagonist, blocks the effects of aldosterone to increase distal tubule Na+ reabsorption and consequently reduces extracellular fluid (ECF) volume and blood pressure. Phenoxybenzamine, an á-blocking agent, inhibits the vasoconstrictor effect of á-adrenergic stimulation. Acetylcholine (ACh), via production of endotheliumderived relaxing factor (EDRF), causes vasodilation of vascular smooth muscle and reduces blood pressure.

Which of the following changes occurs during moderate exercise? (A) Increased total peripheral resistance (TPR) (B) Increased stroke volume (C) Decreased pulse pressure (D) Decreased venous return (E) Decreased arterial PO2

B [Chapter 3, IX B; Table 3.5]. During moderate exercise, sympathetic outflow to the heart and blood vessels is increased. The sympathetic effects on the heart cause increased heart rate and contractility, and the increased contractility results in increased stroke volume. Pulse pressure increases as a result of the increased stroke volume. Venous return also increases because of muscular activity; this increased venous return further contributes to increased stroke volume by the Frank-Starling mechanism. Total peripheral resistance (TPR) might be expected to increase because of sympathetic stimulation of the blood vessels. However, the buildup of local metabolites in the exercising muscle causes local vasodilation, which overrides the sympathetic vasoconstrictor effect, thus decreasing TPR. Arterial Po2 does not decrease during moderate exercise, although O2 consumption increases.

A 17-year-old boy is brought to the emergency department after being injured in an automobile accident and sustaining significant blood loss. He is given a transfusion of 3 units of blood to stabilize his blood pressure. Which of the following is a consequence of the decrease in blood volume in this patient? (A) Increased renal perfusion pressure (B) Increased circulating levels of angiotensin II (C) Decreased renal Na+ reabsorption (D) Decreased renal K+ secretion

B [Chapter 3, IX C; Table 3.6; Figure 3.21; Chapter 5 IV C 3 b (1)]. The decreased blood volume causes decreased renal perfusion pressure, which initiates a cascade of events, including increased renin secretion, increased circulating angiotensin II, increased aldosterone secretion, increased Na+ reabsorption, and increased K+ secretion by the renal tubule.

During which phase of the cardiac cycle does ventricular pressure rise but ventricular volume remain constant? (A) Atrial systole (B) Isovolumetric ventricular contraction (C) Rapid ventricular ejection (D) Reduced ventricular ejection (E) Isovolumetric ventricular relaxation (F) Rapid ventricular filling (G) Reduced ventricular filling

B [Chapter 3, V; Figure 3.15]. Because the ventricles are contracting during isovolumetric contraction, ventricular pressure increases. Because all of the valves are closed, the contraction is isovolumetric. No blood is ejected into the aorta until ventricular pressure increases enough to open the aortic valve.

Plasma renin activity is lower than normal in patients with (A) hemorrhagic shock (B) essential hypertension (C) congestive heart failure (D) hypertension caused by aortic constriction above the renal arteries

B [Chapter 3, VI B]. Patients with essential hypertension have decreased renin secretion as a result of increased renal perfusion pressure. Patients with congestive heart failure and hemorrhagic shock have increased renin secretion because of reduced intravascular volume, which results in decreased renal perfusion pressure. Patients with aortic constriction above the renal arteries are hypertensive because decreased renal perfusion pressure causes increased renin secretion, followed by increased secretion of angiotensin II and aldosterone.

Which of the following would cause an increase in glomerular filtration rate (GFR)? (A) Constriction of the afferent arteriole (B) Constriction of the efferent arteriole (C) Constriction of the ureter (D) Increased plasma protein concentration (E) Infusion of inulin

B [Chapter 5, II C 6; Table 5.3]. Glomerular filtration rate (GFR) is determined by the balance of Starling forces across the glomerular capillary wall. Constriction of the efferent arteriole increases the glomerular capillary hydrostatic pressure (because blood is restricted in leaving the glomerular capillary), thus favoring filtration. Constriction of the afferent arteriole would have the opposite effect and would reduce the glomerular capillary hydrostatic pressure. Constriction of the ureter would increase the hydrostatic pressure in the tubule and, therefore, oppose filtration. Increased plasma protein concentration would increase the glomerular capillary oncotic pressure and oppose filtration. Infusion of inulin is used to measure the GFR and does not alter the Starling forces.

Which diuretic causes increased urinary excretion of Na+ and K+ and decreased urinary excretion of Ca2+? (A) Acetazolamide (B) Chlorothiazide (C) Furosemide (D) Spironolactone

B [Chapter 5, IV C 3 a; VI C 2; Table 5.11]. Thiazide diuretics act on the early distal tubule (cortical diluting segment) to inhibit Na+ reabsorption. At the same site, they enhance Ca2+ reabsorption so that urinary excretion of Na+ is increased while urinary excretion of Ca2+ is decreased. K+ excretion is increased because the flow rate is increased at the site of distal tubular K+ secretion.

. Arterial pH of 7.25, arterial Pco2 of 30 mm Hg, and decreased urinary excretion of NH4 + would be observed in a (A) patient with chronic diabetic ketoacidosis (B) patient with chronic renal failure (C) patient with chronic emphysema and bronchitis (D) patient who hyperventilates on a commuter flight (E) patient who is taking a carbonic anhydrase inhibitor for glaucoma (F) patient with a pyloric obstruction who vomits for 5 days (G) healthy person

B [Chapter 5, IX D 1; Table 5.9]. The blood values are consistent with metabolic acidosis with respiratory compensation. Because the urinary excretion of NH4 + is decreased, chronic renal failure is a likely cause.

A 40-year-old woman has an arterial pH of 7.25, an arterial Pco2 of 30 mm Hg, and serum [K+] of 2.8 mEq/L. Her blood pressure is 100/80 mm Hg when supine and 80/50 mm Hg when standing. What is the cause of her abnormal blood values? (A) Vomiting (B) Diarrhea (C) Treatment with a loop diuretic (D) Treatment with a thiazide diuretic

B [Chapter 5, IX D; Table 5.9]. The arterial blood values and physical findings are consistent with metabolic acidosis, hypokalemia, and orthostatic hypotension. Diarrhea is associated with the loss of HCO3 - and K+ from the gastrointestinal (GI) tract, consistent with the laboratory values. Hypotension is consistent with extracellular fluid (ECF) volume contraction. Vomiting would cause metabolic alkalosis and hypokalemia. Treatment with loop or thiazide diuretics could cause volume contraction and hypokalemia, but would cause metabolic alkalosis rather than metabolic acidosis.

A 37-year-old woman suffers a severe head injury in a skiing accident. Shortly thereafter, she becomes polydipsic and polyuric. Her urine osmolarity is 75 mOsm/L, and her serum osmolarity is 305 mOsm/L. Treatment with 1-deamino-8-d-arginine vasopressin (dDAVP) causes an increase in her urine osmolarity to 450 mOsm/L. Which diagnosis is correct? (A) Primary polydipsia (B) Central diabetes insipidus (C) Nephrogenic diabetes insipidus (D) Water deprivation (E) Syndrome of inappropriate antidiuretic hormone (SIADH)

B [Chapter 5, VII C; Table 5.6]. A history of head injury with production of dilute urine accompanied by elevated serum osmolarity suggests central diabetes insipidus. The response of the kidney to exogenous antidiuretic hormone (ADH) (1-deamino-8-d-arginine vasopressin [dDAVP]) eliminates nephrogenic diabetes insipidus as the cause of the concentrating defect.

Which gastrointestinal secretion is inhibited when the pH of the stomach contents is 1.0? (A) Saliva (B) Gastric secretion (C) Pancreatic secretion (D) Bile

B [Chapter 6, II A 1 c; IV B 4 a]. When the pH of the stomach contents is very low, secretion of gastrin by the G cells of the gastric antrum is inhibited. When gastrin secretion is inhibited, further gastric HCl secretion by the parietal cells is also inhibited. Pancreatic secretion is stimulated by low pH of the duodenal contents.

Somatostatin inhibits the secretion of which of the following hormones? (A) Antidiuretic hormone (ADH) (B) Insulin (C) Oxytocin (D) Prolactin (E) Thyroid hormone

B [Chapter 6, II B 1; Chapter 7 III B 3 a (1), VI D]. The actions of somatostatin are diverse. It is secreted by the hypothalamus to inhibit the secretion of growth hormone by the anterior lobe of the pituitary. It is secreted by cells of the gastrointestinal (GI) tract to inhibit the secretion of the GI hormones. It is also secreted by the delta cells of the endocrine pancreas and, via paracrine mechanisms, inhibits the secretion of insulin and glucagon by the beta cells and alpha cells, respectively. Prolactin secretion is inhibited by a different hypothalamic hormone, dopamine.

Which of the following is absorbed by facilitated diffusion? (A) Glucose in duodenal cells (B) Fructose in duodenal cells (C) Dipeptides in duodenal cells (D) Vitamin B1 in duodenal cells (E) Cholesterol in duodenal cells (F) Bile acids in ileal cells

B [Chapter 6, V A 2 b]. Monosaccharides (glucose, galactose, and fructose) are the absorbable forms of carbohydrates. Glucose and galactose are absorbed by Na+-dependent cotransport; fructose is absorbed by facilitated diffusion. Dipeptides and water-soluble vitamins are absorbed by cotransport in the duodenum, and bile acids are absorbed by Na+-dependent cotransport in the ileum (which recycles them to the liver). Cholesterol is absorbed from micelles by simple diffusion across the intestinal cell membrane.

Which of the following substances acts on its target cells via an inositol 1,4,5-triphosphate (IP3)-Ca2+ mechanism? (A) Somatomedins acting on chondrocytes (B) Oxytocin acting on myoepithelial cells of the breast (C) Antidiuretic hormone (ADH) acting on the renal collecting duct (D) Adrenocorticotropic hormone (ACTH) acting on the adrenal cortex (E) Thyroid hormone acting on skeletal muscle

B [Chapter 7; Table 7.2]. Oxytocin causes contraction of the myoepithelial cells of the breast by an inositol 1,4,5-triphosphate (IP3)-Ca2+ mechanism. Somatomedins (insulin-like growth factor [IGF]), like insulin, act on target cells by activating tyrosine kinase. Antidiuretic hormone (ADH) acts on the V2 receptors of the renal collecting duct by a cyclic adenosine monophosphate (cAMP) mechanism (although in vascular smooth muscle it acts on V1 receptors by an IP3 mechanism). Adrenocorticotropic hormone (ACTH) also acts via a cAMP mechanism. Thyroid hormone induces the synthesis of new protein (e.g., Na+,K+-adenosine triphosphatase [ATPase]) by a steroid hormone mechanism.

A 20-year-old female has signs and symptoms consistent with genital warts. What would have been the best approach to prevent this disease? (A) Immunization with a live reassortment vaccine (B) Immunization with recombinant-produced viruslike particles (C) Passive immunization with antineuraminidase antibodies (D) Prophylactic treatment with acyclovir

B. A quadrivalent recombinant vaccine (Gardasil) containing viruslike particles of types 6, 11, 16, and 18 (these types cause 90% of genital warts) is available for prophylactic immunization to prevent genital warts and cervical cancer by types 16 and 18.

A 65-year-old man has anemia, splenomegaly, and extramedullary hematopoiesis. He has experienced easy fatigability, weight loss, and weakness. Bone marrow biopsy reveals marked proliferation of fibrous tissue (myelofibrosis) consistent with agnogenic myeloid metaplasia. Which of the following is a characteristic finding in this disorder? (A) Depletion of bone marrow megakaryocytes (B) Teardrop-shaped erythrocytes (C) Autosplenectomy (D) Neoplastic plasma cells in the bone marrow (E) Blast crisis in the peripheral blood

B. Agnogenic (idiopathic) myeloid metaplasia is characterized by extensive non-neoplastic myelofibrosis and extramedullary hematopoiesis resulting in hepatosplenomegaly. Teardrop-shaped erythrocytes, as well as scattered nucleated red cells and granulocytic precursor cells, can be found in the peripheral blood smear. Although marrow myeloid (granulocytic) and erythroid precursor cells are depleted, megakaryocytes tend to be spared and even increased in number.

A 45-year-old woman dies several days after a partial small bowel resection for repair of a volvulus. The surgery had apparently gone well, but shortly afterward she developed intractable fever, hypotension, multiorgan failure, and marked respiratory distress. Just prior to death, chest radiographs showed complete "whiteout" of both lungs. At autopsy, both lungs were found to have collapsed or distended alveoli, many of which were lined with fibrin-rich hyaline membranes. The cause of these pulmonary findings is best characterized as (A) aspiration. (B) diffuse alveolar damage. (C) generalized atelectasis. (D) lobar consolidation. (E) pneumothorax.

B. Alveolar hyaline membrane formation is a characteristic finding in ARDS. The common factor in ARDS is diffuse alveolar damage induced by a number of agents or conditions, one of which is septic shock. Other prominent causes include trauma, uremia, gastric aspiration, inhalation of chemical irritants, oxygen toxicity, Mycoplasma infection, and the severe acute respiratory syndrome (SARS).

A pathologist examines an excisional biopsy specimen and confirms the working diagnostic impression of adenocarcinoma. Because of the nature of the tumor, he requests molecular diagnostic evaluation by the molecular pathology laboratory. Amplification of the HER-2/neu oncogene is demonstrated. This finding is a negative prognostic indicator in carcinoma of the (A) adrenal. (B) breast. (C) kidney. (D) stomach. (E) thyroid.

B. Amplification of the HER-2/neu oncogene is frequently observed in breast cancer, and an increased degree of amplification is thought to be a negative prognostic indicator.

A 65-year-old man presents with urinary hesitancy, frequency, urgency, sensation of incomplete bladder emptying, and straining to start the urinary stream. Digital rectal examination is performed. Further workup reveals the diagnosis of nodular prostatic hyperplasia. Which of the following is an association of this disorder? (A) Tends to arise in the peripheral zone of the prostatic glands (B) An increase in serum prostate-specific antigen (PSA) may occur (C) Low incidence in older men (D) May frequently progress to bony osteoblastic metastases (E) Often improves over time without intervention

B. An increase in PSA may occur in both nodular prostatic hyperplasia and in carcinoma of the prostate. Both of these disorders have a peak incidence in elderly men, and nodular prostatic hyperplasia tends to arise in the central zone of prostatic glands.

A 23-year-old woman presents with hypertension, anxiety, and palpitations. Her thyroid-stimulating hormone levels are normal, but she has increased levels of urinary catecholamines. She is referred to an endocrine surgeon after a computed tomography (CT) scan shows a unilateral pheochromocytoma. The surgeon should start therapy with which of the following agents prior to removing the lesion? (A) Dopamine (B) Phenoxybenzamine (C) Pancuronium (D) Pseudoephedrine (E) Isoproterenol

B. An á-adrenoreceptor antagonist such as phenoxybenzamine is indicated for the treatment of pheochromocytomas in the preoperative state as well as if the tumor is inoperable. â-Blockers, such as isoproterenol, are then used systemically, following effective á-blockade, to prevent the cardiac effects of excessive catecholamines. Pseudoephedrine is an á-adrenoreceptor antagonist available over the counter to relieve nasal discharge. There is no role for adrenergic receptor agonists such as dopamine or for that matter nondepolarizing muscle relaxants such as pancuronium.

A 23-year-old woman, who is 23 weeks pregnant, develops a bladder infection due to Pseudomonas spp. She has a documented allergy to penicillin. What is the best choice of treatment given the patient's history and condition? (A) Cefoxitin (B) Aztreonam (C) Imipenem (D) Piperacillin (E) Ciprofloxacin

B. Aztreonam is active against Pseudomonas spp., appears to be safe during pregnancy, and does not show cross-hypersensitivity with penicillins. Piperacillin, cefoxitin, and imipenem all have some overlap in penicillin-allergic patients. Although ciprofloxacin is good in nonpregnant patients, it is absolutely contraindicated in pregnancy.

Which of the following can occur in an adult patient treated with chloramphenicol? (A) Gray baby syndrome (B) Bone marrow suppression (C) Disulfiram-like reaction (D) Nephrotoxicity (E) Ototoxicity

B. Bone marrow suppression results in pancytopenia in treated patients, which in rare cases can lead to aplastic anemia. Gray baby syndrome is associated with chloramphenicol use in infants. Disulfiram-like reactions can occur with some cephalosporins. Aminoglycosides and vancomycin can result in nephrotoxicity and ototoxicity.

Infection with which of the following organisms is more often noted for the production of a lymphocytosis rather than a mononucleosis? (A) Epstein-Barr virus (B) Bordetella pertussis (C) Human immunodeficiency virus (D) Listeria monocytogenes

B. Bordetella pertussis is unusual among bacterial infections in that it causes a lymphocytosis. A mononucleosis-like presentation may occur during the first year of human immunodeficiency virus infection.

A 17-year-old male patient was placed on carbamazepine therapy by his neurologist to control newly developed seizures of unknown etiology. The patient was also recently given a macrolide antibiotic by his family physician for a presumed "walking pneumonia." Halfway through his antibiotic course, the patient again developed seizures. What could account for this new seizure activity? (A) Inhibition of the cytochrome P-450 monooxygenase system (B) Induction of the cytochrome P-450 monooxygenase system (C) Impairment of renal excretion of the antiseizure medication (D) Induction of glucuronyl transferase activity in the liver (E) Reduction in the amount of nicotinamide adenine dinucleotide phosphate (NADPH)

B. Both carbamazepine and macrolide antibiotics are known inducers of the cytochrome P-450 system. Thus, it is likely that the original therapeutic levels of antiseizure medicine were decreased to nontherapeutic levels when the metabolism of the drug was increased with the addition of the antibiotic. Some common drugs that inhibit P-450 include cimetidine, chloramphenicol, and disulfiram. Impaired renal excretion results in increased, not decreased, levels of drugs. The induction of glucuronyl transferase is a possible drug interaction, although less likely in this case. The P-450 system requires nicotinamide adenine dinucleotide phosphate (NADPH); therefore, a deficiency would result in decreased, not increased, activity by the system.

A 45-year-old man presents with a form of bacterial infection in which the invading microorganisms are opsonized prior to their engulfment by phagocytic cells. Of the following complement components or groups of components, which is most likely involved? (A) C3a and C5a (B) C3b (C) C4a (D) C5a alone (E) C5b-9

B. C3b is an important opsonin. C3a and C5a (anaphylatoxins) mediate degranulation of mast cells and basophils. In addition, C5a is a potent chemotactic agent for neutrophils. C5b-9 is the membrane attack complex that mediates complementinduced cell lysis.

A 56-year-old man is seen in the emergency room because of the acute onset of severe crushing precordial chest pain that began on the golf course 1 hour earlier and has persisted until the time of admission. The ECG reveals precordial QS waves and elevated ST segments. Although normal at admission, both the serum CK-MB and cTn-I are significantly elevated 12 hours later. These changes are related to which type of necrosis? (A) Caseous (B) Coagulative (C) Fat (D) Gangrenous (E) Liquefactive

B. Chest pain, ECG findings of QS waves with elevated ST segments, and elevated serum CK-MB and cTn-I are diagnostic of acute MI. MI is the prototype of coagulative necrosis.

A senior at Rich Suburban High School is involved in orchestra and has been on the honor role most years. She does not do drugs. She has had an equally ''squeaky clean'' type of boyfriend since last fall, and they had unprotected sex on three occasions. Six months later she now has recurring pelvic pain with a probable diagnosis of pelvic inflammatory disease. What is the most likely causative agent? (A) Candida albicans (B) Chlamydia trachomatis (C) Neisseria gonorrhoeae (D) Haemophilus ducreyi (E) Treponema pallidum

B. Chl. trachomatis and N. gonorrhoeae are the only two choices listed commonly causing PID. However, Chl. trachomatis is about three times more common and the most common bacterial STD in the general population at large. N. gonorrhoeae is more likely to be seen in economically disadvantaged urban teens.

A 45-year-old male with a 60-pack/year history of smoking presents to his primary care provider with loss of appetite, nausea, vomiting, and muscle weakness. His chest CT reveals enlarged hilar lymph nodes and a suspicious mass in the left hilar region. A presumptive diagnosis of lung cancer is made. Laboratory results reveal low levels of sodium, which in this setting has likely contributed to the syndrome of inappropriate ADH secretion. Which medication might be helpful for this patient's symptoms? (A) Clofibrate (B) Conivaptan (C) Allopurinol (D) Acetazolamide (E) Furosemide

B. Conivaptan is a non-peptide ADH antagonist and as such is useful in the treatment of SIADH, which is commonly seen in patients with lung cancer. Clofibrate (choice A) increases the release of ADH centrally. Allopurinol, acetazolamide, and furosemide do not affect actions of ADH to an appreciable degree (choices C, D, and E).

What is the function of the cyclic AMP binding proteins in facultative bacteria? (A) To decrease levels of elongation factors (B) To enhance RNA polymerase activity (C) To facilitate binding of repressor proteins (D) To increase transcription initiation (E) To suppress the Pasteur effect

B. Cyclic AMP binding proteins bind to specific DNA sequences near promoters, which facilitate RNA polymerase binding to the promoters, thus enhancing transcription of genes associated with those promoters.

Which of the following is an antidote for iron overdose? (A) Protamine (B) Deferoxamine (C) Vitamin K (D) Fresh frozen plasma (E) Charcoal

B. Deferoxamine is an iron-chelating agent and as such can be given in cases of iron supplement overdose. Protamine (A) is an antidote for heparin. Vitamin K (C) and fresh frozen plasma (D) are given for coumarin reversal. Charcoal is an agent sometimes used for gastric lavage.

A viral disease in which antiviral antibodies are thought to have an ''enhancing'' effect so that reinfection with the virus causes a more serious disease is (A) Coxsackie A virus (B) Dengue virus (C) Hanta virus (D) Rubella virus (E) West Nile virus

B. Dengue virus is an arbovirus that causes a disease sometimes known as ''break bone fever'' because of the muscle and joint pain associated with it. Infection by a different virus serotype following a primary infection causes a more serious disease that is thought to be due to the enhancing effect that antibodies to the initial virus have on the second virus's infection properties.

While delivering a newborn baby girl, you notice that she has abnormal facies, but otherwise the delivery is uncomplicated. About 48 hours after birth, the baby girl develops seizures and muscle spasms. She is lethargic, mildly tachypneic, and jittery. Relevant physical findings include peculiar facies, low-set ears, widely spaced eyes, small mandible, no detectable thymus on palpation, muscle rigidity, harsh holosystolic murmur along the lower left sternal border, and a slight cyanotic tinge to the skin. Relevant laboratory findings include hypocalcemia, a low T lymphocyte count, radiograph showing absent thymic shadow, and cardiac ultrasound showing a congenital heart defect in the conotruncal region, and genetic testing reveals a deletion on chromosome 22q. Which of the following is the most likely diagnosis? (A) Patau syndrome (B) DiGeorge syndrome (C) Miller-Decker syndrome (D) Prader-Willi syndrome (E) Treacher Collins syndrome

B. DiGeorge syndrome (DS) is caused by a microdeletion in the long arm of chromosome 22 (22q11), which is also called the DiGeorge chromosomal region (DGCR). DS occurs when pharyngeal pouches 3 and 4 fail to differentiate into the thymus and parathyroid glands. DS is usually accompanied by facial anomalies resembling first arch syndrome (micrognathia, low-set ears) due to abnormal neural crest cell migration, cardiovascular anomalies due to abnormal neural crest cell migration during formation of the aorticopulmonary septum, immunodeficiency due to absence of thymus gland, and hypocalcemia due to absence of parathyroid glands. DS has a phenotypic and genotypic similarity to velocardiofacial syndrome (VCFS), that is, both DS and VCFS are manifestations of a microdeletion at 22q11. The genes for the following have been mapped to 22q11 or the DGCR (although their role is far from clear): catechol-O-methyltransferase (COMT; an enzyme used in catecholamine metabolism), GpIbb (receptor for von Willebrand factor), DGCR3 (a leucine zipper transcription factor), and citrate transport protein (CTP).

A 60-year-old man presents with proteinuria, hypertension, edema, and hypoalbuminemia. Histologic findings in the glomeruli of his kidneys include mesangial accumulation of basement membrane-like material. Which of the following is the most likely diagnosis? (A) Renal amyloidosis (B) Diabetic nephropathy (C) Membranous glomerulonephritis (D) Minimal change disease (E) Poststreptococcal glomerulonephritis

B. Diabetic nephropathy is marked by diffuse or nodular mesangial accumulations of glycosylated basement membrane-like material.

A patient with severe burns that have become infected with Pseudomonas aeruginosa has a circulating exotoxin whose activity is similar to what other organism? (A) Botulinum toxin (B) Diphtheria toxin (C) Pertussis toxin (D) Shiga toxin (E) Tetanus toxin

B. Diphtheria toxin and Pseudomonas exotoxin A are both adenosine diphosphate- ribosylating toxins that irreversibly inactivate elongation factor 2 and inhibit protein synthesis. Although they have similar modes of action, they differ in their cellular targets and antigenicity.

A 63-year-old man with a history of multiple myocardial infarctions is admitted for shortness of breath. A diagnosis of congestive heart failure is made on clinical grounds, and a cardiologist orders a positive inotropic agent for his heart failure. He is also concerned about maintaining perfusion to the kidneys, so an agent that increases renal blood flow is also desirable. Which of the following agents produces both of these effects? (A) Epinephrine (B) Dopamine (C) Isoproterenol (D) Terbutaline

B. Dopamine is useful in the management of congestive heart failure, as it has both positive inotropic effects on the heart and preserves blood flow to the kidneys. Epinephrine and isoproterenol increase cardiac contractility while decreasing peripheral resistance. Albuterol is a â1 agonist used in the management of asthma, and terbutaline is another â-2 agonist used to suppress labor, in the event of threatened labor of a premature fetus.

An 81-year-old man with a history of coronary artery disease and a recent diagnosis of hypothyroidism presents to the emergency department with an acute myocardial infarction (MI). What is the most likely causative agent? (A) Medroxyprogesterone (B) Levothyroxine (C) Thiocyanate (D) Flutamide (E) Diethylstilbestrol (DES)

B. Elderly patients with subclinical hypothyroidism are at risk for arrhythmias, angina, or myocardial infarction (MI) if they have underlying cardiovascular disease when they begin treatment with thyroid hormones such as levothyroxine. These potential adverse effects occur because of increased cardiovascular work load as well as the direct effect of thyroid hormone on the heart.

A 54-year-old woman is undergoing an experimental high-dose regimen with adriamycin and cyclophosphamide for breast cancer. As this treatment is particularly myelosuppressive, the oncologist is worried that her white blood cell count will drop dangerously low, making her susceptible to opportunistic infections. In addition to the chemotherapy, what is the oncologist likely to administer to prevent neutropenia? (A) Epoetin alfa (B) Filgrastim (C) Interferon alfa-2b (D) Oprelvekin (E) Amifostine

B. Filgrastim is a recombinant form of granulocyte colony-stimulating factor (G-CSF) given to prevent chemotherapy-induced neutropenia. Epoetin alfa is commonly used to prevent anemia while on chemotherapy. Oprelvekin is an agent used to help treat chemotherapy-induced thrombocytopenia. Interferon alfa-2b is used in the management of specific leukemias and lymphomas. Amifostine is given to patients receiving radiation to the head and neck to preserve salivary function.

Compared to valproic acid, carbamazepine, and ethosuximide, which of the following is a unique complication of phenytoin use? (A) Hepatotoxicity (B) Gingival hyperplasia (C) Thrombocytopenia (D) Aplastic anemia (E) Stevens-Johnson syndrome

B. Gingival hyperplasia is a unique side effect of phenytoin, which can be partially avoided by meticulous oral hygiene. Several anticonvulsants can cause hepatotoxicity, including valproic acid. Aplastic anemia is a rare, but a potential complication of carbamazepine, ethosuximide. Valproic acid is also associated with thrombocytopenia. Ethosuximide has been associated with a severe form of erythema multiforme, the Steven- Johnson syndrome.

Which of the following can be used to treat a 22-year-old with a recent diagnosis of schizophrenia? (A) Baclofen (B) Haloperidol (C) Chloral hydrate (D) Phenobarbital (E) Imipramine

B. Haloperidol is an antipsychotic agent used in acute psychotic attacks and for the treatment of schizophrenia. It is a dopamine-receptor antagonist that acts predominately at the dopamine D2 receptor. Baclofen is a GABAB receptor antagonist that is used in the treatment of spinal cord injuries. Choral hydrate is a hypnotic agent that works similarly to ethanol. Phenobarbital is a barbiturate used in the treatment of seizures and as an anesthetic. Imipramine is a tricyclic antidepressant and is not used in schizophrenia.

Latent infection of neurons occurs with (A) Cytomegalovirus (B) Herpes simplex virus (C) Measles virus (D) Poliomyelitis virus (E) Rabies virus

B. Herpes simplex viruses have the ability to become latent in neurons and reactivate (replicate) under certain conditions that are not well understood.

A 33-year-old man comes in complaining of "fever and chills" and that he "has to constantly go to the bathroom." He also indicates that he has pain just below the abdominal area on the right side. He states he has not had sexual intercourse in more than 6 months. He suspects that it may be urinary tract infection because he "has had a lot of them over the years." Relevant physical exam findings include flank pain and costovertebral angle tenderness. Relevant laboratory findings include normal calcium levels and computed tomography (CT) scan showing an unusual kidney appearance. Which of the following is the most likely diagnosis? (A) Urachal fistula (B) Horseshoe kidney (C) Pyelonephritis (D) Kidney stones (E) Polycystic kidney disease

B. Horseshoe kidney. The symptoms that the man had (fevers, chills, flank pain, and costovertebral angle tenderness) are classic signs of pyelonephritis as a result of a urinary tract infection (UTI). In this case, the UTI is a result of a urinary tract obstruction caused by a horseshoe kidney. The most common type of renal fusion is the horseshoe kidney. A horseshoe kidney occurs when the inferior poles of the kidneys fuse across the midline. Normal ascent of the kidneys is arrested because the fused portion gets trapped behind the inferior mesenteric artery. Kidney rotation is also arrested so that the hilum faces ventrally.

A 45-year-old man with a history of chronic obstructive pulmonary disease (COPD) manifests marked clubbing and joint pain of the fingers. Radiographs demonstrate soft tissue swelling at the ends of his fingers and osteolysis of the terminal phalanges. Which of the following is the most likely diagnosis? (A) Ankylosing spondylitis (B) Hypertrophic osteoarthropathy (C) Osteoarthritis (D) Rheumatoid arthritis (E) Felty syndrome

B. Hypertrophic osteoarthropathy, manifesting as clubbing of the fingers and associated periostitis of the distal radius and ulna, is associated with chronic lung disease, cyanotic heart disease, and other systemic disorders.

A mother brings her 1-month-old son into the clinic, complaining of her son "vomiting all over the place when he tries to eat something." She said her son's vomiting looks like it was "shot out of a cannon." Relevant physical findings include a small, nontender, palpable mass on the right costal margin. Relevant laboratory findings include barium swallow radiograph showing a narrow pyloric channel and abdominal ultrasound showing a hypertrophic pylorus. Which of the following is the most likely diagnosis? (A) Esophageal hiatal hernia (B) Hypertrophic pyloric stenosis (C) Malrotation of the midgut with volvulus (D) Esophageal stenosis (E) Biliary atresia

B. Hypertrophic pyloric stenosis. Hypertrophic pyloric stenosis occurs when the muscularis externa in the pyloric region hypertrophies, causing a narrow pyloric lumen that obstructs food passage. It is associated clinically with projectile, nonbilious vomiting after feeding and a small, palpable mass at the right costal margin. An increased incidence of hypertrophic pyloric stenosis has been found in infants treated with the antibiotic erythromycin.

A 56-year-old man has severe chest pain, and angiography demonstrates acute occlusion of the circumflex branch of the left coronary artery. Blood flow is successfully restored by percutaneous transluminal coronary angioplasty (PTCA) with stenting. During the period of cellular hypoxic injury, which of the following intracellular changes is likely to have occurred? (A) Decreased calcium (B) Decreased pH (C) Decreased sodium (D) Increased activity of NaðqKðq pump (E) Increased adenosine triphosphate (ATP)

B. Intracellular pH is decreased in severe hypoxic cell injury. This change is caused by mitochondrial damage, which in turn results in decreased oxidative phosphorylation and diminished ATP synthesis. Decreased ATP stimulates glycolysis with lactate formation, thus resulting in decreased intracellular pH. Decreased ATP also diminishes the activity of the membrane-associated Na1K1 pump, allowing an influx of sodium and water. The final steps leading to cell death in severe hypoxic injury are associated with massive influx of extracellular calcium.

A recent Asian immigrant was found to be positive for the tuberculin skin test. The positive result most likely can be attributed to (A) Antibody against mycobacteria plus complement (B) Previous BCG vaccination (C) Activation of previously sensitized NK cells (D) Niacin production by PPD

B. Many Asian immigrants have been immunized against TB with BCG vaccines. Neither sensitized NK cells, niacin, nor antibody are participants in the tuberculin test.

An 8-year-old boy presents with irritability, vomiting, and gait ataxia. CT scan demonstrates a midline tumor of the cerebellum. Which of the following is the most likely diagnosis? (A) Glioblastoma multiforme (B) Medulloblastoma (C) Meningioma (D) Schwannoma (Neurilemmoma) (E) Oligodendroglioma

B. Medulloblastoma, a highly malignant tumor, is one of the most frequently occurring malignancies of early childhood. The posterior cranial fossa is the most common site of this tumor. Other high-incidence tumors of early childhood include acute leukemia, Wilms tumor, and adrenal neuroblastoma.

A 54-year-old diabetic woman was seen in the emergency room 3 weeks ago with complaints of swelling, warmth, and pain in her foot. She was diagnosed with cellulitis and sent home on a 10-day course of an oral first-generation cephalosporin. She returns with severe diarrhea, and Clostridium difficile is suspected. What is the initial treatment for this condition? (A) Clindamycin (B) Metronidazole (C) Ciprofloxacin (D) Neomycin (E) Silver sulfadiazine

B. Metronidazole is the preferred treatment for Clostridium difficile colitis, which probably resulted from the patient's use of a broad-spectrum antibiotic for her initial infection. Vancomycin is considered in the treatment of Clostridium difficile colitis in refractory cases. The use of Clindamycin is often associated with Clostridium difficile colitis. Ciprofloxacin can be used for the treatment of diverticulitis, but not colitis. Neomycin is used to sterilize the bowel, which is not the goal in this case. Silver sulfadiazine is used to treat skin infections in burn patients.

In June, an 18-year-old man develops a sore throat with a fever and a nonproductive cough that develops into pneumonia with a severe, prolonged hacking cough but little sputum production. Cryoagglutinins are present. He is treated appropriately and successfully with azithromycin. What is the nature of the most likely causative agent? (A) Acid-fast organism (B) Chlamydia (C) DNA virus (D) Gram-negative rod (E) Gram-positive coccus (F) Mycoplasma

B. Mycoplasma is a common cause of pneumonia in teenagers and young adults. During the course of the infection, some autoagglutinating antibodies (cold agglutinins) may be formed against red blood cells. The antibodies are inactive at normal body temperature, but agglutinate red blood cells at 4C.

A 25-year-old woman has recently been diagnosed with a suspicious pigmented skin lesion and is waiting for the results of a biopsy. She is concerned because she had heard that such lesions vary, with some being more serious than others. Which of the following pigmented lesions is most likely to metastasize early? (A) Dysplastic nevus (B) Nodular melanoma (C) Pigmented nevus (D) Juvenile melanoma (E) Superficial spreading melanoma

B. Nodular melanoma tends to expand vertically rather than horizontally, a phenomenon associated with a more aggressive course and a greater likelihood of metastasis. Among the malignant melanomas, nodular melanoma has the poorest prognosis.

A 60-year-old male patient has had pain from chronic gout for more than 20 years. Until recently, he has been managed successfully with allopurinol but he has recently suffered a series of debilitating attacks. Which of the following would be a reasonable next approach in treating his gout? (A) Febuxostat (B) Pegloticase (C) Indomethacin (D) Furosemide (E) Celocoxib

B. Pegloticase is a recombinant uricase, an enzyme mutated and non-functional in humans. Uricase metabolizes uric acid to water-soluble allantoin. Pegloticase is approved for cases of refractory gout. It is highly effective but must be administered by infusion and is expensive.

A 56-year-old man with a history of stable angina was seen in the emergency room 1 hour following the onset of unrelenting substernal pain not relieved by nitroglycerin. An electrocardiogram (ECG) revealed deep Q waves across the precordium, ST segment elevations, and inverted T waves. Serum levels of creatinine kinase MB (CK-MB) and cardiac troponin I (cTn-I) were within the normal range. What is the best explanation for these findings? (A) Lactate dehydrogenase (LDH) should have been ordered rather than CK-MB and cTn-I. (B) The diagnosis is acute myocardial infarction (MI), and CK-MB and cTn-I were determined too early in the course of the disease. (C) The diagnosis is unstable angina rather than acute MI. (D) The findings are indicative of a dissecting aneurysm (dissecting hematoma) of the aorta. (E) CK-MB alone can be misleading, and more definitive information would have been expected from total CK determination.

B. Persistent chest pain unrelieved by nitroglycerin and the abnormal ECG findings are diagnostic of MI. It would be unusual to observe significant elevations of CK-MB and cTn-I as early as 1 hour following an MI. These markers rise in parallel and are weakly positive in about 6 hours and reach peak levels in about 10 to 15 hours following an MI. CK-MB returns to normal levels in 3 to 7 days, while cTn-I can remain elevated a week or longer.

A 23-year-old woman with systemic lupus erythematosus (SLE) is placed on high doses of the steroid prednisone. Prednisone is a potent inhibitor of the enzyme phospholipase A2, which normally functions in which of the following events? (A) Cyclooxygenase pathway (B) Liberation of arachidonic acid from membrane phospholipids (C) Lipoxygenase pathway (D) Synthesis of prostacyclin (PGI2) (E) Synthesis of thromboxane A2 (TxA2)

B. Phospholipase A2 catalyzes the release of arachidonic acid from membrane phospholipids. Arachidonic acid metabolism then proceeds through two major pathways, the lipoxygenase and cyclooxygenase pathways. The lipoxygenase pathway yields HETE and leukotrienes. The cyclooxygenase pathway yields thromboxanes and prostaglandins. Prostacyclin (PGI2) is synthesized in endothelial cells, and thromboxane A2 (TxA2) is synthesized in platelets. It should be noted that prednisone inhibits both the cyclooxygenase and lipoxygenase pathways by inhibiting the formation of precursors to each pathway.

A 67-year-old man, a heavy smoker, is seen because of dyspnea and cough. A chest X-ray reveals abnormal densities, and a computed tomography (CT) scan is suggestive of a neoplasm involving the pleura. A biopsy confirms the diagnosis of mesothelioma. Other than cigarette smoke, this finding suggests exposure to which toxin? (A) Aflatoxin B1 (B) Asbestos (C) Diethylstilbestrol (DES) (D) Ionizing radiation (E) â-Naphthylamine

B. Pleural and peritoneal mesotheliomas are associated with exposure to asbestos, and the apparent tumorigenic effect of asbestosis is markedly enhanced by cigarette smoking. Aflatoxin B1 is associated with hepatocellular carcinoma. Clear cell adenocarcinoma of the vagina has been a hazard to daughters exposed during intrauterine life to DES administered to their mothers to prevent spontaneous abortion. Ionizing radiation is associated with many cancers, including leukemias, breast cancer, and thyroid malignancies. â-naphthylamine and other aniline dyes are associated with transitional cell carcinoma of the bladder.

The individual most likely to progress to chronic liver disease following an acute infection is (A) A 50-year-old who becomes HBsAg positive (B) A baby born to chronically active infected HBV mother (C) A liver transplant patient infected with HCV (D) A recent immigrant who is HEV positive (E) A young adult co infected with HBV and HDV

B. Progression to chronic disease is inversely related to the age of infection with HBV and 90% of neonates can become chronically ill. HCV infection progression is also high, but is 70% rather than 90%.

A neutropenic patient with Pseudomonas aeruginosa septicemia develops shock, which is triggered by (A) Catalase (B) Lipid A (C) Flagella from Gram-negative bacteria (D) O-specific polysaccharide side chain of endotoxin (E) Teichoic acid-peptidoglycan fragments

B. Pseudomonas is a Gram-negative organism; therefore, the patient has a Gramnegative septicemia. Endotoxic activity is associated with lipid A. No toxicity is associated with the O polysaccharides, the flagella from Gram-negative bacteria, or catalase. Teichoic acid-peptidoglycan fragments trigger Gram-positive shock.

A 25-year-old woman is seen because of numbness and pain in her fingers and hands on exposure to cold. When either her hands or feet are very cold, they turn white and then blue. These changes are characteristic of which of the following? (A) Buerger disease (thromboangiitis obliterans) (B) Raynaud disease (C) Wegener granulomatosis (D) Kawasaki disease (E) Takayasu arteritis

B. Raynaud disease is cold-induced vasospasm of arterioles and small arteries, most often involving the fingers and sometimes the hands and feet. Young, otherwise healthy women are most often affected.

Which of the following inhibit HMG-CoA reductase? (A) Nicotinic acid (B) Rosuvastatin (C) Ezetimibe (D) Cholestyramine (E) Gemfibrozil

B. Rosuvastatin is an HMG-CoA reductase inhibitor. Nicotinic acid (A) inhibits the process of esterification of fatty acids, thereby reducing plasma triglyceride levels. Ezetimibe (C) reduces cholesterol absorption. Cholestyramine can bind bile acids and prevents their enterohepatic circulation (D). Gemfibrozil (E) reduces hepatic synthesis of cholesterol.

A distraught mother brings her 2-month-old daughter into your office saying that she noticed a "lump growing from her child's bottom." She states that she "noticed it about 2 weeks ago while changing her daughter's diaper." The lump was small and she so didn't think much of it, but over time it has "grown to the size of a baseball." Relevant physical exam findings include a large spheroid mass that appeared to be very firm on palpation. Relevant laboratory findings include biopsy of the mass showing tissue containing hair, teeth, muscle fibers, and thyroid follicular cells. Which of the following is the most likely diagnosis? (A) Spina bifida with meningocele (B) Sacrococcygeal teratoma (C) Spina bifida with meningomyelocele (D) Chordoma (E) Caudal dysplasia (sirenomelia)

B. Sacrococcygeal teratoma. Sacrococcygeal teratoma (ST) is a tumor that arises from remnants of the primitive streak, which normally degenerates and disappears. ST is derived from pluripotent cells of the primitive streak and often contains various types of tissue (e.g., bone, nerve, hair). ST occurs more commonly in female infants and usually becomes malignant during infancy (must be removed by age 6 months). Caudal dysplasia (sirenomelia) refers to a constellation of syndromes ranging from minor lesions of lower vertebrae to complete fusion of the lower limbs. Caudal dysplasia is caused by abnormal gastrulation, whereby the migration of mesoderm is disturbed. Spina bifida occurs when the bony vertebral arches fail to form properly, thereby creating a vertebral defect, usually in the lumbosacral region.

A 5-year-old boy is brought to the emergency room by his parents after they found him with an empty bottle of aspirin. They are not sure how many tablets the boy has consumed. On examination, the child is hyperpneic and lethargic. While an emergency treatment is started, a sample is drawn for an arterial blood determination. What pattern is most likely to be indicated by the arterial blood gas values? (A) Mixed metabolic alkalosis and respiratory alkalosis (B) Mixed respiratory alkalosis and metabolic acidosis (C) Mixed respiratory acidosis and metabolic acidosis (D) Mixed respiratory acidosis and metabolic alkalosis (E) Mixed metabolic acidosis and metabolic alkalosis

B. Salicylate toxicity initially increases the medullary response to carbon dioxide, with resulting hyperventilation and respiratory alkalosis. Increase in lactic acid and ketone body formation results in a metabolic acidosis. All other choices are incorrect in this particular setting. The treatment includes correction of acid-base disturbances, replacement of electrolytes and fluids, cooling, alkalinization of urine, and forced diuresis.

A 34-year-old woman who is in her third trimester complains of her hands and face "swelling up a few days ago." She remarks that she has also felt like "her heart was racing a mile a minute." Relevant physical exam findings include hypertension (.160/110 mm Hg) and edema of the hands and face. Relevant laboratory findings include proteinuria (.5 g/24 hours), and ultrasound was unremarkable. Which of the following is the most likely diagnosis? (A) Molar pregnancy (B) Severe preeclampsia (C) Choriocarcinoma (D) Ectopic tubal pregnancy (E) Placental abruption

B. Severe preeclampsia. Preeclampsia is a complication of pregnancy characterized by hypertension, edema, and/or proteinuria. Severe preeclampsia refers to the sudden development of maternal hypertension (blood pressure .160/110 mm Hg), edema (hands and/or face), and proteinuria (.5 g/24 hours), usually after week 32 of gestation (third trimester). Eclampsia includes the additional symptom of convulsions. The pathophysiology of preeclampsia involves a generalized arteriolar constriction that affects the brain (seizures and stroke), kidneys (oliguria and renal failure), liver (edema), and small blood vessels (thrombocytopenia and disseminated intravascular coagulation). Treatment of severe preeclampsia involves magnesium sulfate (for seizure prophylaxis) and hydralazine (blood pressure control); once the patient is stabilized, delivery of the fetus should ensue immediately. Risk factors include nulliparity, diabetes, hypertension, renal disease, twin gestation, or hydatidiform mole (produces first trimester preeclampsia). Her symptoms of hypertension, proteinuria, and edema are all telltale signs of preeclampsia. In addition, her advancing age has left to susceptible to this condition. A molar pregnancy is normally seen in the first trimester. Renal disease is unlikely because there were no findings other than proteinuria.

A burn patient has an infected area with odiferous, blue-green pus. What is the most likely causative agent? (A) Aspergillus fumigatus (B) Pseudomonas aeruginosa (C) Staphylococcus aureus (D) Streptococcus pyogenes (E) Vibrio vulnificus

B. Some strains of Pseudomonas aeruginosa produce a blue-green pigment, which is clinically notable in burn wounds infected with Pseudomonas.

A male patient with HTN is being treated with a thiazide and a potassium sparing diuretic. His blood pressure is well controlled with this combination but he complains of tenderness fatty deposits in his pectoral area. Which of the following is the most likely to be causing this adverse effect? (A) Indapamide (B) Spironolactone (C) Amiloride (D) Chlorthalidone (E) Hydrocholorothiazide

B. Spironolactone blocks androgen and glucocorticoid receptors as well as mineralocorticoid receptors. This can cause proliferation and tenderness of mammary tissue.

A 45-year-old African-American man has marked splenomegaly. This finding is unusual and unexpected in which of the following disorders? (A) Infectious mononucleosis (B) Sickle cell anemia (C) Chronic myelogenous leukemia (D) Hereditary spherocytosis (E) Agnogenic myeloid metaplasia (idiopathic myelofibrosis)

B. Splenomegaly occurs in children with sickle cell anemia, but repeated bouts of splenic infarction and fibrosis reduce the spleen to a fibrous remnant in adults (autosplenectomy). The other listed conditions are all well-known causes of splenomegaly.

A Mexican fruit picker has developed subcutaneous nodular lesions along the lymphatics from the initial site of trauma caused by a plum thorn puncture. What is the nature of the most likely causative agent? (A) Acid-fast organism (B) Dimorphic fungus (C) Filamentous fungus (D) Gram-positive coccus (E) Gram-negative rod (F) Helminth

B. Sporothrix schenckii, a dimorphic fungus, is found in the environment on various plant materials. Subcutaneous infections begin with traumatic implantation of contaminated plant material such as slivers from mine timbers, thorns, or the combination of wires and sphagnum moss (used by floral designers). The resulting sporotrichosis is characterized by a fixed nodular subcutaneous lesion or lesions along the lymphatics from the initial trauma site. When found in tissues, the fungus grows as an oval to cigar-shaped yeast. It grows as sporulating filaments in the environment and has a worldwide distribution. Cases are most common in tropical regions because people are less likely to wear protective clothing (e.g., long pants, longsleeved shirts).

An autopsy is performed on a 60-year- old man with a history of sustained ethanol abuse. There had been a history of progressive dementia with marked memory loss manifested by a tendency to fabricate false accounts of recent events. Additionally, confusion, ataxic gait, and paralysis of eye movements had been noted. The most likely findings in the brain are (A) amyloid-containing neuritic plaques within cerebral cortex, amygdala, and hippocampus. (B) degeneration of mamillary bodies and paramedian masses of gray matter. (C) depigmentation of substantia nigra and locus ceruleus. (D) diffuse cortical atrophy with hydrocephalus ex vacuo. (E) multiple lacunar infarcts and progressive subcortical demyelination.

B. Sustained ethanol abuse and progressive dementia are strongly suggestive of the Wernicke-Korsakoff syndrome, which is due to thiamine deficiency, most often in association with chronic alcoholism. Clinical characteristics include the Wernicke triad (confusion, ataxia, and ophthalmoplegia) and often Korsakoff psychosis, characterized by memory loss and confabulation (making up stories in an attempt to hide the inability to remember). The morphologic counterparts of these changes include degeneration of the mamillary bodies and of paramedian masses of gray matter.

A 17-year-old male visiting his grandparents in Louisiana in August develops severe unrelenting headache that is not relieved by analgesics. He is very sleepy and complains about an odd odor that he thinks is in the whole house. Primary amebic meningoencephalitis is diagnosed. How was it most likely acquired? (A) Intravenous drug abuse (B) Diving or swimming in contaminated water (C) His grandfathers use of horse dung as vegetable fertilizer (D) Eating undercooked crayfish (a shell fish) (E) Handling cat litter

B. Swimming/diving/jumping in warm contaminated waters may cause infection with Naegleria, which rapidly develops into primary amebic meningoencephalitis, which is generally fatal. It is thought that the organism enters through the cribriform plate.

Beta-lactam drugs bind to penicillin, binding proteins to halt synthesis of peptidoglycan. Where are these proteins located? (A) In the cytoplasm (B) In the cytoplasmic membrane (C) In the periplasmic space (D) In the peptidoglycan itself (E) In the capsule

B. The PBPs are actually in the cytoplasmic membrane, even though the substrate is the cell peptidoglycan. Thus b-lactams must penetrate the Gram-negative outer membrane through the porins and cross the peptidoglycan to bind. In the Gram-positive bacteria, they just have to cross the peptidoglycan to bind in the membrane.

A 38-year-old woman is found to have episodic headache, palpitation, and diaphoresis, along with severe hypertension. She is also found to have hyperglycemia, but diabetes mellitus has been ruled out. These findings suggest an endocrine tumor secreting which of the following hormones? (A) Antidiuretic hormone (ADH) (B) Catecholamines (C) Insulin (D) PTH (E) Prolactin

B. The association of episodic headache, palpitation, and diaphoresis, along with severe hypertension and hyperglycemia, is most suggestive of a catecholamine- secreting pheochromocytoma. Other nondiabetic endocrine disorders associated with hyperglycemia include Cushing syndrome, either pituitary or adrenal, with hypersecretion of corticotropin or cortisol; acromegaly, with hypersecretion of growth hormone; and hyperthyroidism, with hypersecretion of thyroxine.

In the lab, a dense mixture of the above patient's isolate with plasma on a slide shows clumping. This test correlates with which early process in the pathogenesis of acute endocarditis in a healthy heart? (A) Binding to fibronectin (B) Binding to fibrinogen (C) Triggering fibrin production (D) Binding of protein A (E) Superantigen activity

B. The binding of surface components to the plasma fibrinogen cross-links the staphylococci. The fibrinogen binding is probably the major reason Staphylococcus aureus can damage the normal heart. It can also bind to fibronectin, so it is also a major causative agent of bacterial endocarditis in people with congenital defects and previously damaged hearts.

A 30-year-old woman has sudden blurring of vision in her right eye, paresthesias, and spasticity. CSF findings include oligoclonal bands on electrophoresis. Magnetic resonance imaging of the brain reveals T2 hyperintensities characteristic of demyelination in the paraventricular regions. Which of the following is a prominent characteristic of this patient's disorder? (A) Axonal degeneration (B) Optic nerve, brain stem, and paraventricular areas are favored sites (C) Confined to the peripheral nervous system (D) Associated with leukemia or lymphoma (E) Caused by prions

B. The clinical history is characteristic of multiple sclerosis, the most frequently occurring of the demyelinating diseases. Multiple sclerosis is characterized by destruction of myelin, with preservation of axons. The optic nerve, brain stem, and paraventricular areas are favored sites of demyelination. Other characteristics of note include multiple oligoclonal immunoglobulin bands on CSF electrophoresis, association with certain HLA haplotypes, and geographic distribution, with incidence increasing with distance away from the equator.

A 68-year-old woman presents with fever, chills, and cough productive of blood- tinged sputum. Fluid aspirated from the right pleural space would most likely (A) be clear and straw-colored in appearance. (B) contain large numbers of neutrophils. (C) have a glucose content somewhat higher than the serum glucose. (D) have a protein content of less than 1 g/dL. (E) have a specific gravity of 1.012.

B. The clinical history is strongly suggestive of bacterial pneumonia. Pleural fluid from this patient would typically be an exudate rather than a transudate, and would be expected to be cloudy and contain many neutrophils. The fluid would also demonstrate reduced glucose, increased protein, and increased specific gravity.

A 35-year-old man with a known history of severe chronic alcohol abuse presents with low-grade fever, jaundice, hepatomegaly, leukocytosis, and markedly abnormal liver function tests. Aspartate aminotransferase (AST) and alanine aminotransferase (ALT) are both elevated. An expected histologic finding in this condition is (A) effacement of the normal liver architecture by diffuse fibrosis and abnormal regenerating nodules. (B) Mallory hyaline inclusions, macrovesicular steatosis, and neutrophilic infiltration. (C) multiple giant cells. (D) nests or cords of well-differentiated cells separated by dense collagen lamellae. (E) parenchymal deposition of hemosiderin.

B. The clinical picture is that of alcoholic hepatitis, which is characterized by fatty change, focal liver cell necrosis, infiltrates of neutrophils, and intracytoplasmic hyaline inclusions referred to as Mallory bodies.

A 40-year-old woman presents with grayish pigmentation of the skin in a large region covering her entire posterior neck and axillae. The hyperpigmented areas started out as smaller macules but have now progressed to form palpable plaques. At times the hyperpigmented areas are pruritic. Which of the following is an important association of this skin lesion? (A) Hypercholesterolemia (B) Marker of visceral malignancy (C) Tends to recur after resection (D) Convulsions, mental retardation, and retinal detachment (E) Viral infection

B. The clinical presentation is illustrative of acanthosis nigricans, a cutaneous lesion that may be associated not only with diabetes and other endocrinopathies, but also with visceral malignancy, such as carcinoma of the stomach, lung, breast, or uterus. Other associations of malignancy include migratory venous thrombosis, which is also associated with visceral malignancies; clubbing of the fingers, which may be associated with a number of disorders, including carcinoma of the lung; and marantic endocarditis, which is associated with wasting diseases, such as widespread cancer.

A 54-year-old carpenter was brought to the hospital by ambulance after he was involved in a high-speed automobile chase that terminated in a collision into the trunk of a tree. On admission to the hospital, his skin was cold and clammy, his pulse was rapid and thready, and his blood pressure was 60 systolic and 40 diastolic. His blood alcohol was 0.29 g/dL (presumptive level for drunkenness is 0.08). In spite of blood transfusions, he died during an emergency laparotomy that revealed a ruptured spleen and a slightly enlarged liver. Microscopic examination of the liver at autopsy revealed intracytoplasmic clear vacuoles displacing the intact nuclei of the hepatocytes to the periphery of the cells. Special stains will most likely demonstrate that the vacuolar material is (A) bilirubin. (B) fat. (C) glycogen. (D) hemosiderin. (E) water.

B. The description of clear vacuoles displacing intact nuclei to the periphery is characteristic of fatty change (steatosis) of the liver; however, clear intracytoplasmic vacuolization of hepatocytes may be due to accumulations of water or glycogen, and sometimes special stains are required for confirmation of the nature of the vacuoles. In industrialized countries, such as the United States, the most common cause of fatty change of the liver is alcoholism.

Five children in Mrs. Thompson's third grade class develop a disease which begins as a bright red rash on the face and turns violet after a few days then disappears; then a maculopapular rash appears on the trunk, buttocks and extremities. It soon fades from the trunk but persists on the thighs and forearms. Two children have also had a slight fever and a sore throat; but all were not terribly sick. The genetic material of the most likely causative infectious agent is (A) Double-stranded DNA (B) Single-stranded DNA (C) Double-stranded RNA (D) Single-stranded RNA (E) Segmented single-stranded RNA

B. The disease is most probably erythema infectiosum (slapped cheek syndrome) caused by parvovirus B19, a single-stranded DNA virus.

If one were to create a single antigen vaccine to have prevented the outbreak and for use as a vaccine in women prior to pregnancy, what viral molecule would be most likely to lead to the production of successful neutralizing antibodies? (A) Matrix protein (B) Capsid protein (C) Surface glycopeptide (D) Polymerase

B. The disease is most probably erythema infectiosum (slapped cheek syndrome) caused by parvovirus B19, a single-stranded DNA virus. Parvovirus B19 is naked, so protective neutralizing antibodies would be to the viral capsid. It does not carry a polymerase but uses cellular polymerases, which is why it requires replicating cells. It is not enveloped, so it has no glycoproteins or any matrix proteins.

A 20-year-old man is found to have hemolytic anemia with jaundice and splenomegaly. A younger brother is found to be similarly affected, and his mother had had a history of splenectomy. Which of the following abnormalities is an expected finding in this patient? (A) Increased haptoglobin (B) Unconjugated hyperbilirubinemia (C) Increased urine bilirubin (D) Decreased reticulocytes (E) Marrow erythroid hypoplasia

B. The jaundice of hemolytic anemia is due to unconjugated hyperbilirubinemia. Because unconjugated bilirubin is not excreted into the urine, the type of jaundice is acholuric, jaundice without bilirubin pigment in the urine. In hemolytic anemia, haptoglobin is markedly decreased. Peripheral red cell destruction is mirrored by marrow erythroid hyperplasia with release of newly formed red cells into the peripheral blood, manifest as reticulocytosis.

In a pre-employment examination, a 23-year-old woman is found to have laboratory values consistent with mild anemia, and a blood smear is reported as demonstrating hypochromia and microcytosis. Which of the following determinations would be most useful in demonstrating that these findings were manifestations of â-thalassemia minor, as contrasted to other causes of hypochromia and microcytosis? (A) Hemoglobin A1c (B) Hemoglobin A2 (C) Hemoglobin value on routine complete blood count (CBC) (D) Histochemical demonstration of á-chain aggregates (E) Microscopic examination of peripheral blood smear

B. The most frequent causes of mild anemia with hypochromia and microcytosis include iron deficiency anemia, the anemia of chronic disease, and b- thalassemia minor. In the latter, the diagnosis is confirmed by demonstrating increased concentration of hemoglobin A2 and the characteristic CBC findings of marked microcytosis, as evidenced by a very low MCV with only moderate reduction of the hemoglobin and hematocrit. In iron deficiency anemia, serum iron is decreased, TIBC is increased, and storage iron is depleted, as indicated by decreased serum ferritin and absent bone marrow hemosiderin on Prussian blue stain. Additionally, a source of blood loss is often apparent. The anemia of chronic disease is most often normochromic and normocytic, but can be hypochromic and microcytic. In such cases, a decrease in TIBC and the presence of an obvious chronic disease are indicative of the cause of the anemia.

A 35-year-old male AIDS patient is hospitalized with a serious pneumonia. His x-ray shows an interstitial pattern of infection, and lab analysis of his sputum indicates the presence of giant cells. Which drug is the best one to be utilized to treat this infection? (A) Foscarnet (B) Ganciclovir (C) Ribavirin (D) Trifluridine (E) Zidovudine

B. The pneumonia is caused by CMV and is best treated with the CMV antiviral ganciclovir.

A 6 month old infant is brought to the emergency department having woken up from a nap with a weak cry, ptosis, unreactive pupils, and unable to lift her arms. She has had routine well-baby care. There is no sign of physical abuse or rash, and she is afebrile. The physician puts the child on a respiration monitor and calls the infectious disease consultant. What is most likely causing the problem? (A) Viral meningitis (B) Infant botulism (C) Botulism from canned goods improperly heated (D) Bacterial meningitis-Listeria monocytogenes (E) Bacterial meningitis-Streptococcus pneumoniae

B. The symptoms are classical for botulism. Common sources of botulinum toxin include canned home goods, and, in cases of infant botulism, household dust or honey. It is most likely that this is from the ingestion of environmental spores (dust or honey) rather than from canned goods, although that is a possibility. Listeria meningitis would be very uncommon this long after birth, and the child would probably be febrile and not have the descending paralysis. She should not have Streptococcal meningitis as she is vaccinated, and it would present as a febrile disease with a stiff neck. It is also not typical for viral meningitis.

Risperidone works primarily through inhibition of receptors for (A) Dopamine (B) Serotonin (C) Histamine (D) Acetylcholine (E) Norepinephrine

B. The unique affinities of various antipsychotics result in their unique activities and their unique side effects. Risperidone is an atypical antipsychotic that works by blocking the 5-HT2A serotonin receptor. The older high-potency antipsychotics inhibit dopamine receptors. Agents such as clozapine inhibit histamine receptors. Atropine is an antagonist at cholinergic receptors.

A 25-year-old woman is struck in the left breast in a "steering wheel" injury in what appeared to be a minor auto accident. After several days of pain and tenderness, she noted the persistence of a "lump" at the site of the injury. After excision biopsy, amorphous basophilic material was noted within the mass. The amorphous material is an example of (A) apocrine metaplasia. (B) dystrophic calcification. (C) enzymatic fat necrosis. (D) granulomatous inflammation. (E) mammary dysplasia.

B. The vignette describes an instance of traumatic fat necrosis, which must be distinguished from enzymatic fat necrosis. The description of amorphous basophilic material is indicative of calcification, and calcification of previous damaged tissue is termed dystrophic calcification. Dystrophic calcification must be distinguished from metastatic calcification, which occurs in the presence of hypercalcemia and affects nondamaged tissues.

A 4-year-old girl with a known chromosomal defect is seen because of fever and a skin rash. The rash consists of numerous small petechial hemorrhages with predominant involvement of the lower extremities. Her platelet count is markedly reduced, and the total white count is markedly increased. A blood smear and bone marrow aspirate reveal large numbers of undifferentiated blast cells, which by flow cytometry are found to be positive for the CD10 antigen. This complication occurs most frequently in association with which one of the following chromosomal disorders? (A) Cri du chat syndrome (B) Down syndrome (C) Fragile X syndrome (D) Klinefelter syndrome (E) Turner syndrome

B. The vignette is consistent with a hematologic diagnosis of acute lymphoblastic leukemia, a condition that occurs with markedly increased incidence in association with Down syndrome.

Demyelinating lesions, increased IgG in spinal fluid, and chronic relapsing occurrences are characteristic of which autoimmune disease? (A) Ulcerative colitis (B) Multiple sclerosis (C) Systemic lupus erythematosus (SLE) (D) Congenital thymic aplasia (E) Myasthenia gravis

B. These symptoms are characteristics of multiple sclerosis. Myasthenia gravis is associated with an antiacetylcholine receptor antibody, resulting in muscle weakness. DiGeorge syndrome, SLE, and ulcerative colitis do not have a brain component.

A Peace Corps worker is sent back to the states from East Africa because he had had a very high fever and African sleeping sickness is suspected. What characteristic of the organism makes it a fatal infection unless treated properly? (A) Highly sialylated surface proteins making it ''invisible'' to the immune system (B) Surface antigenic variation (C) Ability to turn on a T2 response (D) Damage to neutrophils (E) Polysaccharide capsule

B. Trypanosoma brucei rhodesiense and T. brucei rhodesiense both have the ability to continue to change their antigenic coats so much so that if hypergammaglobulinemia is not found in the CSF of a suspected case, it is not likely to be African sleeping sickness.

In January, a 74-year-old woman is brought to the hospital emergency room by her husband who states that she had complained of a fever and headache during the past week and during the last 2 days is confused and cannot perform her daily chores. Her physical examination indicates some weaknesses in her left side and her head MRI shows necrosis in the right temporal lobe. What is the causative agent? (A) Coxsackie A16 virus (B) HSV-1 (C) Rabies virus (D) West Nile virus (E) Western equine encephalitis virus

B. Viral encephalitis in January, which localizes to the temporal lobe, is most likely HSV-1

Which of the following would be a good option to help a patient fall asleep with minimal "hangover"? (A) Secobarbital (B) Zolpidem (C) Chlordiazepoxide (D) Flumazenil (E) Buspirone

B. Zolpidem has actions similar to those of benzodiazepines, although it is structurally unrelated. It is used as a hypnotic and anxiolytic with minimal abuse potential. Barbiturates such as secobarbital are rarely used because of their lethality on overdose. Chlordiazepoxide is a long-acting benzodiazepine, whereas most hypnotics are short-acting benzodiazepines. Flumazenil is a benzodiazepine receptor antagonist that will not reverse the effects of zolpidem. Buspirone is not used as a hypnotic and has little sedative effect.

Which of the following actions occurs when light strikes a photoreceptor cell of the retina? (A) Transducin is inhibited (B) The photoreceptor depolarizes (C) Cyclic guanosine monophosphate (cGMP) levels in the cell decrease (D) All-trans retinal is converted to 11-cis retinal (E) Increased release of an excitatory neurotransmitter

C [Chapter 2, II C 4; Figure 2.5]. Light striking a photoreceptor cell causes the conversion of 11-cis retinal to all-trans retinal; activation of a G protein called transducin; activation of phosphodiesterase, which catalyzes the conversion of cyclic guanosine monophosphate (cGMP) to 5Œ-GMP so that cGMP levels decrease; closure of Na+ channels by the decreased cGMP levels; hyperpolarization of the photoreceptor; and decreased release of glutamate, an excitatory neurotransmitter.

CO2 generated in the tissues is carried in venous blood primarily as (A) CO2 in the plasma (B) H2CO3 in the plasma (C) HCO3 - in the plasma (D) CO2 in the red blood cells (RBCs) (E) carboxyhemoglobin in the RBCs

C [Chapter 4, V B; Figure 4.9]. CO2 generated in the tissues enters venous blood and, in the red blood cells (RBCs), combines with H2O in the presence of carbonic anhydrase to form H2CO3. H2CO3 dissociates into H+ and HCO3 -. The H+ remains in the RBCs to be buffered by deoxyhemoglobin, and the HCO3 - moves into plasma in exchange for Cl-. Thus, CO2 is carried in venous blood to the lungs as HCO3 -. In the lungs, the reactions occur in reverse: CO2 is regenerated and expired.

A 28-year-old man who is receiving lithium treatment for bipolar disorder becomes polyuric. His urine osmolarity is 90 mOsm/L; it remains at that level when he is given a nasal spray of dDAVP. Which diagnosis is correct? (A) Primary polydipsia (B) Central diabetes insipidus (C) Nephrogenic diabetes insipidus (D) Water deprivation (E) Syndrome of inappropriate antidiuretic hormone (SIADH)

C [Chapter 5, VII C]. Lithium inhibits the G protein that couples the antidiuretic hormone (ADH) receptor to adenylate cyclase. The result is inability to concentrate the urine. Because the defect is in the target tissue for ADH (nephrogenic diabetes insipidus), exogenous ADH administered by nasal spray will not correct it.

A 53-year-old man with multiple myeloma is hospitalized after 2 days of polyuria, polydipsia, and increasing confusion. Laboratory tests show an elevated serum [Ca2+] of 15 mg/dL, and treatment is initiated to decrease it. The patient's serum osmolarity is 310 mOsm/L. The most likely reason for polyuria in this man is (A) increased circulating levels of antidiuretic hormone (ADH) (B) increased circulating levels of aldosterone (C) inhibition of the action of ADH on the renal tubule (D) stimulation of the action of ADH on the renal tubule (E) psychogenic water drinking

C [Chapter 5, VII D 3; Table 5.6]. The most likely explanation for this patient's polyuria is hypercalcemia. With severe hypercalcemia, Ca2+ accumulates in the inner medulla and papilla of the kidney and inhibits adenylate cyclase, blocking the effect of ADH on water permeability. Because ADH is ineffective, the urine cannot be concentrated and the patient excretes large volumes of dilute urine. His polydipsia is secondary to his polyuria and is caused by the increased serum osmolarity. Psychogenic water drinking would also cause polyuria, but the serum osmolarity would be lower than normal, not higher than normal.

The principle of positive feedback is illustrated by the effect of (A) PO2 on breathing rate (B) glucose on insulin secretion (C) estrogen on follicle-stimulating hormone (FSH) secretion at midcycle (D) blood [Ca2+] on parathyroid hormone (PTH) secretion (E) decreased blood pressure on sympathetic outflow to the heart and blood vessels

C [Chapter 7, I D; X E 2]. The effect of estrogen on the secretion of folliclestimulating hormone (FSH) and luteinizing hormone (LH) by the anterior lobe of the pituitary gland at midcycle is one of the few examples of positive feedback in physiologic systems—increasing estrogen levels at midcycle cause increased secretion of FSH and LH. The other options illustrate negative feedback. Decreased arterial PO2 causes an increase in breathing rate (via peripheral chemoreceptors). Increased blood glucose stimulates insulin secretion. Decreased blood [Ca2+] causes an increase in parathyroid hormone (PTH) secretion. Decreased blood pressure decreases the firing rate of carotid sinus nerves (via the baroreceptors) and ultimately increases sympathetic outflow to the heart and blood vessels to return blood pressure to normal.

Which of the following hormones stimulates the conversion of testosterone to 17â-estradiol in ovarian granulosa cells? (A) Adrenocorticotropic hormone (ACTH) (B) Estradiol (C) Follicle-stimulating hormone (FSH) (D) Gonadotropin-releasing hormone (GnRH) (E) Human chorionic gonadotropin (HCG) (F) Prolactin (G) Testosterone

C [Chapter 7, X A]. Testosterone is synthesized from cholesterol in ovarian theca cells and diffuses to ovarian granulosa cells, where it is converted to estradiol by the action of aromatase. Follicle-stimulating hormone (FSH) stimulates the aromatase enzyme and increases the production of estradiol.

What is transferred when an F+ cell is crossed with an F- cell? (A) Only some bacterial chromosomal genes (B) Generally the whole bacterial chromosome (C) Only the fertility factor DNA (D) Both the plasmid and chromosomal genes (E) No genes

C. An F+ cell contains the fertility factor in the plasmid state. In the cross between an F+ cell and an F- cell, chromosomal genes are not transferred because they are not covalently linked to the plasmid. Only the plasmid genes are transferred.

A 10 month old child presents with a temperature of 39.8C (103.6F) and lethargy. Brudzinski's and Kernig's signs are both present. Gram stain of sediment from CSF showed Gram-negative rods. What part of the routine health care could have prevented this disease? (A) A trivalent killed viral vaccine (B) A polysaccharide vaccine with 23 different antigens (C) A covalently linked protein-polyribitol vaccine (D) A heptavalent polysaccharide-protein conjugate vaccine (E) A quadrivalent capsular vaccine

C. Based on the description of the disease, the child has bacterial meningitis. The Gram stain suggests Haemophilus influenzae is the causative agent, which is confirmed by latex particle agglutination. The vaccine is a conjugate polysaccharide-protein vaccine as described in choice C. The other vaccines described belong to poliovirus (A), the adult Strep. pneumoniae vaccine (B), the infant Strep. pneumoniae vaccine (D), and the Neisseria meningitis vaccine.

Pepsin digestion of the IgG antibody against tetanus toxoid will (A) Result in loss of the ability to form a lattice with the toxoid (B) Produce two Fab molecules and one Fc fragment (C) Result in a (Fab) 2 molecule and destruction of the Fc fragment (D) Result in the loss of the Ch 1 heavy chain constant domain

C. Because an F(ab)2 results, pepsin digested antibody will still be able to form a lattice.

A mother brings her 5-year-old son into your office for a follow-up visit. The child previously had a bout with pneumonia, and the mother remarked that the child has been coughing up "yellow and green stuff." The mother mentioned that he has had a number of coughs and colds that were just like this in the past. Relevant physical exam findings include foul-smelling, greenish sputum with speckles of blood, orthopnea, and fever, and his chart is remarkable for cystic fibrosis. Relevant laboratory findings include spirometry showing a reduced forced expiratory volume in 1 second/forced vital capacity (FEV1/FVC) ratio, radiograph showing multiple cysts that have a "honeycomb" appearance, and CT scanning shows a dilation of bronchi. Which of the following is the most likely diagnosis? (A) Asthma (B) Bronchitis (C) Bronchiectasis (D) Pneumonia (E) Influenza

C. Bronchiectasis. Bronchiectasis is the abnormal, permanent dilation of bronchi due to chronic necrotizing infection (e.g., Staphylococcus, Streptococcus, Haemophilus influenzae), bronchial obstruction (e.g., foreign body, mucous plugs, or tumors), or congenital conditions (e.g., Kartagener syndrome, cystic fibrosis, immunodeficiency disorders). The lower lobes of the lung are predominately affected, and the affected bronchi have a saccular appearance. Clinical signs include cough, fever, and expectoration of large amounts of foul-smelling purulent sputum. Bronchiectasis may also be classified to a group of disorders known as chronic obstructive pulmonary disease (COPD), which are characterized by increased resistance to airflow during both inspiration and expiration due to airway obstruction. Other members of COPD include emphysema, chronic bronchitis, and asthma.

When complement is fixed by antigen-antibody complexes, what chemotactic factor for neutrophils is released? (A) C1 (B) C2 (C) C3a (D) C4b (E) C789 complex

C. C3a and C3b are the prime components released following the action of C3 convertase. C3a along with C5a are strong chemotactic agents.

Which of the following binds to class II histocompatibility antigens (A) CD2 (B) CD3 (C) CD4 (D) CD8 (E) CD25

C. CD4 binds to class II histocompatibility antigens, resulting in humoral immunity; CD8 binds to class I histocompatibility antigens, resulting in cell-mediated immunity.

What additional tissues are frequently involved with diphtheria besides the ''bull neck'' and the pseudomembranous pharyngitis? (A) Skin (cutaneous diphtheria) (B) Kidneys (C) Heart and nerves (D) Liver and kidneys (E) Ears and sinuses

C. Corynebacterium diphtheriae does not invade tissues; rather, the exotoxin enters the bloodstream and affects tissues, primarily the heart and nerves, causing myocarditis and recurrent laryngeal neuropathy.

Which of the following medications would provide the best relief from episodic attacks of Ménière's disease? (A) Furosemide (B) Ondansetron (C) Diazepam (D) Emetrol (E) Phentermine

C. Diazepam and lorazepam are very effective in treating the vertigo associated with Ménière's disease. Loop diuretics, such as furosemide, can precipitate vertigo secondary to volume depletion and resultant orthostatic hypotension. Ondansetron is a powerful antiemetic. Emetrol is an over-the-counter (OTC) antiemetic for infants. Phentermine is an amphetamine derivative that has been used for weight loss.

A father brings his 1-month-old daughter into the clinic, complaining that his daughter frequently "throws up after she eats" and "it just shoots across the room." Relevant physical exam findings include projectile vomiting when the infant is laid on its back after a feeding. Relevant laboratory findings include radiograph showing a portion of the stomach located in the pleural cavity. Which of the following is the most likely diagnosis? (A) Hypertrophic pyloric stenosis (B) Gastroesophageal reflux disease (C) Esophageal hiatal hernia (D) Congenital diaphragmatic hernia (E) Tracheoesophageal fistula

C. Esophageal hiatal hernia. Esophageal hiatal hernia is a herniation of the stomach through the esophageal hiatus into the pleural cavity caused by an abnormally large esophageal hiatus. An esophageal hiatal hernia renders the esophagogastric sphincter incompetent so that stomach contents reflux into the esophagus. Clinical signs in the newborn include vomiting ( frequently projectile) when the infant is laid on its back after feeding.

A 60-year-old man presents with the new onset of dyspnea, chest pain, cough, and weight loss. In the past, he had worked in construction, installing insulation in buildings. Chest radiograph shows a rightsided pleural effusion with marked pleural thickening and mass formation. A malignant neoplasm is demonstrated by biopsy. What is the most likely diagnosis? (A) Small cell carcinoma (B) Squamous cell carcinoma (C) Mesothelioma (D) Adenocarcinoma (E) Carcinoid tumor

C. Exposure to asbestos (common in construction workers, shipyard workers, or people who have worked with insulation or fire safety materials) markedly predisposes one to mesothelioma of the pleura or peritoneum and is also closely linked to primary lung carcinoma (especially in smokers).

What bacteria can use fermentation pathways but also contain superoxide dismutase? (A) Obligate aerobes (B) Obligate anaerobes (C) Facultative anaerobes (D) Aerobic heterotrophs

C. Facultative anaerobes grow in the presence or absence of oxygen; a respiratory mode is used when oxygen is present, and fermentation occurs when it is not. Facultative anaerobes contain the enzyme superoxide dismutase, which aids aerobic growth by preventing the accumulation of the superoxide ion. Obligate aerobes do not have fermentative pathways and require oxygen for growth; obligate anaerobes lack superoxide dismutase. The heterotrophs require preformed organic compounds for growth.

A patient presents with an inflamed itchy groin area. What antifungal would work as long as it is a dermatophyte but might make it worse if it is a yeast infection? (A) Miconazole (B) Nystatin (C) Griseofulvin (D) Trimethoprim-sulfamethoxazole (E) Amphotericin B

C. Griseofulvin, which is given orally and is effective on dermatophytes but may exacerbate yeast infections. Incorrect are choice A: miconazole works on both dermatophytes and yeasts. Choice B, nystatin works on yeasts. Of the fungi, choice D is used orally only for pneumocystis. And choice E (AMB) is incorrect as it is not used topically.

An autopsy is performed on a 75-yearold man, who for the past several years had had a pill-rolling tremor in his hand, slowing of his movements, and muscle rigidity. Autopsy findings reveal depigmentation of the substantia nigra and locus ceruleus. Which of the following is the most likely diagnosis? (A) Alzheimer disease (B) Huntington disease (C) Idiopathic Parkinson disease (D) Myasthenia gravis (E) Wernicke-Korsakoff syndrome

C. Idiopathic Parkinson disease is manifested morphologically by depigmentation of cells of the substantia nigra and locus ceruleus.

Chromosomal studies in a 56-year-old man indicate a (9:22) translocation, the Philadelphia chromosome, confirming the diagnosis of chronic myelocytic leukemia (CML). Which of the following might be used in his treatment? (A) Anastrozole (B) Rituximab (C) Imatinib (D) Gefitinib (E) Amifostine

C. Imatinib is an orally active small molecule inhibitor of the oncogenic bcr-abl kinase produced as a result of the Philadelphia chromosome, used to treat chronic myelogenous leukemia. It also inhibits the c-Kit receptor and can be used in treating gastrointestinal stromal tumors (GISTS). Anastrozole is used in the management of breast cancer. Rituximab is an antibody used in the treatment of non-Hodgkin lymphoma. Gefitinib is an orally active small molecule inhibitor of the EGF receptor, used in the treatment of some lung cancer. Amifostine is used as a radio-protectant, with or without cisplatin.

A biopsy of a "cold" nodule from the left lobe of the thyroid from an otherwise asymptomatic 30-year-old woman is found to demonstrate a malignant proliferation of C cells and an amyloid stroma that stains positively with a Congo red stain. This tumor is referred to as which of the following types of carcinoma? (A) Epidermoid (B) Follicular (C) Medullary (D) Papillary (E) Undifferentiated

C. Medullary carcinoma is characterized histologically by sheets of tumor cells in an amyloid-containing stroma. This neoplasm is a calcitonin-producing tumor derived from "C" cells of the thyroid. Medullary carcinoma can occur singly or as a component of multiple endocrine neoplasia (MEN) syndromes types IIa and IIb.

A 30-year-old woman presents with weakness and hyperreflexia of the left lower extremity. These symptoms had begun as mild weakness, but had slowly and progressively become more severe. A CT scan of the head reveals a well-defined right-sided parasagittal mass compressing (but not invading) the brain parenchyma. Which of the following is the most likely diagnosis? (A) Glioblastoma multiforme (B) Medulloblastoma (C) Meningioma (D) Neurilemmoma (schwannoma) (E) Oligodendroglioma

C. Meningioma is a benign tumor of the meninges that is external to the brain and is therefore usually surgically resectable. Meningioma is the second most common primary intracranial neoplasm. This tumor causes a mass effect (a space-occupying defect) that physically compresses, but does not invade, brain parenchyma. Symptoms correlate with the location of the meningioma. For example, this patient's right-sided parasagittal meningioma caused slow and progressive left-sided lower extremity weakness.

A 56-year-old woman presents with bone pain, diffuse demineralization of bone, hypercalcemia, anemia, hypergammaglobulinemia, proteinuria, and normal serum alkaline phosphatase. This set of findings is most suggestive of (A) Ewing sarcoma. (B) hyperparathyroidism. (C) multiple myeloma. (D) osteomalacia. (E) Paget disease of bone.

C. Multiple myeloma often presents with diffuse demineralization of bone, even though punched-out lesions are more characteristic. Findings in this scenario that distinguish multiple myeloma from other conditions also characterized by bony demineralization include anemia, hypergammaglobulinemia, proteinuria, and normal (rather than increased) serum alkaline phosphatase.

Which of the following should be considered to treat an acetaminophen overdose in a 17-year-old girl? (A) Trientine (B) Sorbitol (C) N-acetylcysteine (D) Ipecac (E) Diazepam

C. N-acetylcysteine is used in the case of acetaminophen toxicity. It provides sulfhydryl groups for the regeneration of glutathione stores in the body. Trientine is a copper- chelating agent sometimes used in Wilson's disease. Sorbitol is used as a cathartic to help remove toxins from the gastrointestinal tract. Ipecac has been used to induce emesis in cases of toxic ingestions. Diazepam can be used to prevent seizures when strychnine is ingested.

A frantic father rushes his 1-year-old daughter to your clinic, saying that he "thinks his daughter's leg is broken." He says that this is the third time that his daughter has broken a bone in the last 2 months, and he thinks his wife may be abusing the child while he is at work. Relevant physical exam findings include short, deformed limbs, blue sclera of the eye, and kyphoscoliosis, and medical history indicates that there may have been bone fractures at birth. Relevant laboratory findings include radiographs showing multiple, healed fractures of the limbs, and genetic testing reveals a mutation in the gene for type 1 collagen on chromosome 7q22. Which of the following is the most likely diagnosis? (A) Marfan syndrome (B) Child abuse (C) Osteogenesis imperfecta (D) Ehlers-Danlos syndrome (E) Achondroplasia

C. Osteogenesis imperfecta. Osteogenesis Imperfecta (OI) is an autosomal dominant (types I and IV) or recessive (types II and III) genetic disorder caused by a mutation in the gene for type I collagen subunits on chromosome 7q22 or 17q22. OI is characterized by extreme bone fragility, with spontaneous fractures occurring when the fetus is still in the womb and blue sclera of the eye. Severe forms of OI are fatal in utero or during the early neonatal period. Milder forms of OI may be confused with child abuse.

Cellulitis develops in a 26-year-old man after he is bitten by his girlfriend's cat. What is themost likely dominant organism involved in the infection? (A) Bartonella (Rochalimaea) henselae (B) Calymmatobacterium granulomatis (C) Pasteurella multocida (D) Toxoplasma gondii (E) Clostridium tetani

C. Pasteurella multocida, a dominant organism in the cat's mouth. Choice A, Bartonella, is the causative agent of cat scratch fever and is less common in bites. Choice B, Calymmatobacterium, causes a sexually transmitted infection, and Toxoplasma (choice D) is associated with cats but transmitted by their feces not bites. Choice E, tetanus, is not likely unless the man has not been vaccinated.

A mother brings in her 6-week-old infant son because "I just want him to get checked out." She further tells you that "you know he was born prematurely, and thank God he didn't have any serious breathing problems; but I'm still worried." Relevant physical exam findings include the finding that the infant is small but active and appears to be mildly short of breath, and a harsh, machine-like, continuous murmur in the upper left parasternal area. Which of the following is the most likely diagnosis? (A) Coarctation of the aorta (B) Membranous ventricular septal defect (C) Patent ductus arteriosus (D) Double aortic arch (E) Tetralogy of Fallot

C. Patent ductus arteriosus. Patent ductus arteriosus occurs when the ductus arteriosus.a connection between the left pulmonary artery and aorta.fails to close. Normally, the ductus arteriosus functionally closes within a few hours after birth via smooth muscle contraction to ultimately form the ligamentum arteriosum. A patent ductus arteriosus causes a left ¨ right shunting of oxygen-rich blood from the aorta back into the pulmonary circulation. This can be treated with prostaglandin synthesis inhibitors (such as indomethacin), which promote closure. It is very common in premature infants and maternal rubella infection. Clinical signs include a harsh, machine-like, continuous murmur in the upper left parasternal area.

A patient presents with abdominal pain made better for a while by ingestion of bland food. A diagnosis of Helicobacter pylori is made. If the DNA had been analyzed, a region of unique DNA coding for the Cag A protein and its type VI injection like secretion system. What is this segment of DNA called? (A) A replicon (B) An integron (C) A pathogenicity island (D) A transposon

C. Pathogenicity islands carry virulence genes and sometimes secretion systems and are often of different GC:AT ratios than most of the organism's DNA, suggesting it was acquired from another organism.

Upper and lower gastrointestinal endoscopic examinations are performed on a 45-year-old man. A lesion is found and the patient is told that the lesion has NO malignant potential. Of the following choices, which is the lesion that was most likely found? (A) Colorectal villous adenoma (B) Crohn disease (C) Duodenal peptic ulcer (D) Familial multiple polyposis (E) Ulcerative colitis

C. Peptic ulcer of the duodenum is not a precursor lesion to carcinoma. The risk of malignant transformation in familial multiple polyposis approaches 100%. Colorectal villous adenomas undergo malignant change in about 30% of cases. There is a markedly increased incidence of colon cancer in long-standing cases of ulcerative colitis. The incidence of colon cancer is also increased in Crohn disease, but to a lesser degree than in ulcerative colitis.

A 33-year-old woman presents with episodic palpitations, sweating, tremor, and a sense of apprehension. During these episodes, her blood pressure is markedly elevated. Which of the following laboratory tests is most likely to be of diagnostic significance? (A) Serum serotonin (B) Serum gastrin (C) Urine VMA (D) Urine cortisol (E) Serum PTH

C. Pheochromocytoma of the adrenal medulla (and its extra-adrenal counterpart paraganglioma) secretes the catecholamines epinephrine and norepinephrine. Increased urinary excretion of catecholamines or their metabolites metanephrine and VMA is a clinical indicator of this tumor.

A 25-year-old woman who is CEO of a new biotech company has been under considerable stress this last year trying to negotiate a contract with a major drug company. She has also been under a very rigorous exercise program because "she just can't stand any fat on her body" and ran in the Boston marathon 4 months ago. Due to her busy schedule, her eating habits have radically changed, and sometimes "the sight of food just disgusts me." She is not on any drug or medication. She tells you that recently she met "the guy" and has been sexually active with him for "about 2 months now." She comes to you because her menstrual cycle is 2 weeks late and sometimes she feels nauseated, especially in the morning. Relevant physical exam findings were unremarkable. Relevant laboratory findings include a positive â-human chorionic gonadotropin (hCG) test. Which of the following is the most likely diagnosis? (A) Secondary amenorrhea due to stress (B) Secondary amenorrhea due to anorexia nervosa (C) Pregnancy (D) Turner syndrome (E) Secondary amenorrhea due to antipsychotic drug therapy

C. Pregnancy. Amenorrhea can be primary or secondary. Primary amenorrhea is the complete absence of menstruation in a woman from puberty. The most common cause of primary amenorrhea is Turner syndrome. Secondary amenorrhea is the absence of menstruation for at least 3 months in a woman who previously had normal menstruation. Many factors can cause secondary amenorrhea, including stress, anorexia nervosa, elevated prolactin levels (e.g., prolactinoma or antipsychotic drug therapy), and pregnancy. Of these factors, only pregnancy is associated with a positive hCG test.

A 56-year-old man who had been receiving intravenous antibiotics for severe cellulitis develops fever, toxicity, and severe diarrhea. This scenario suggests which of the following disorders? (A) Celiac sprue (B) CMV infection (C) Pseudomembranous colitis (D) Ulcerative colitis (E) Whipple disease

C. Severe diarrhea, fever, and toxicity following broad-spectrum antibiotic therapy is likely due to pseudomembranous colitis. This disorder is caused by overgrowth of Clostridium difficile, a commensal microorganism indigenous to the bowel, and is marked morphologically by superficial mucosal erosions with overlying necrotic, loosely adherent mucosal debris. The clostridia remain intraluminal, but secrete an enterotoxin that is responsible for the clinical and pathologic manifestations of the disorder.

After the delivery of a healthy baby girl, a physician notices a tuft of hair on the lower back of the child. The physician asked the mother about her prenatal health care, and she said she didn't take folic acid until the second month of pregnancy because she didn't know she was pregnant until then. Relevant physical exam findings include a tuft of hair on the lower back with no noticeable sac formation. Relevant laboratory findings include radiograph showing a defect in the vertebral arches but no sac filled with fluid or spinal cord. Which of the following is the most likely diagnosis? (A) Spina bifida with meningocele (B) Spina bifida with meningomyelocele (C) Spina bifida occulta (D) Spina bifida with rachischisis (E) Caudal dysplasia (sirenomelia)

C. Spina bifida occulta. Spina bifida occurs when the bony vertebral arches fail to form properly, thereby creating a vertebral defect, usually in the lumbosacral region. Spina bifida occulta is evidenced by a tuft of hair in the lumbosacral region. Spina bifida occulta is the least severe variation and occurs in 10% of the population. Spina bifida with meningocele occurs when the meninges protrude through a vertebral defect and form a sac filled with cerebrospinal fluid (CSF). The spinal cord remains in its normal position. Spina bifida with meningomyelocele occurs when the meninges and spinal cord protrude through a vertebral defect and form a sac filled with CSF. Spina bifida with rachischisis occurs when the posterior neuropore of the neural tube fails to close during week 4 of development. This condition is the most severe type of spina bifida, causing paralysis from the level of the defect caudally.

A 55-year-old woman presents with "the worst headache of my life" and severe nausea. She states that the headache and nausea started suddenly several hours ago. A CSF tap reveals frank blood in the CSF. A CT scan of the head reveals diffuse hemorrhage over the surface of the brain. Which of the following is the most likely mechanism causing these findings? (A) Laceration of the middle meningeal artery (B) Laceration of the bridging veins (C) Rupture of a berry aneurysm in the circle of Willis (D) Rupture of a Charcot-Bouchard aneurysm (E) Hypertensive intraparenchymal hemorrhage

C. Subarachnoid hemorrhage is most often caused by rupture of a berry aneurysm of the circle of Willis. Frank blood may be found in the CSF, and diffuse hemorrhage over the surface of the brain is due to subarachnoid bleeding.

What is the mechanism of action of tacrolimus? (A) It inhibits transport to the nucleus of the transcription factor NF-AT (B) It stimulates apoptosis of some lymphoid lineages (C) It decreases the activity of calcineurin (D) It inhibits mTOR, which in turn delays the G1-S transition (E) It inhibits proliferation of promyelocytes

C. Tacrolimus decreases the activity of calcineurin, which leads to a decrease in nuclear NF-AT and the transcription of T-cell-specific lymphokines and early T-cell activation. Choice A refers to mechanism of action of cyclosporine. Choice B refers to glucocorticoids. Choice D represents the mechanism of action of sirolimus. Choice E refers to azathioprine.

An elderly woman presents with chronic fatigue, difficulty sleeping, a rapid heartbeat, and bulging eyes. Lab tests revealed autoantibodies to thyroid-stimulating hormone. The most likely diagnosis is (A) Hashimoto's disease (B) Cancer of the thyroid (C) Graves' disease (D) Pernicious anemia

C. The autoantibodies to TSH in Graves' disease compete with TSH for its receptor site and mimic TSH activity. Pernicious anemia is characterized by antibodies to the gastric parietal cell and intrinsic factor resulting in inability to absorb vitamin B12.

An 18-year-old Iowan dirt bike racer, who recently raced for the first time in the desert southwest, presents in September with cough, malaise, low-grade fever, myalgias, and chest pain. Rales are heard and respiratory infiltrates are noted on radiograph. Sputum stained with calcofluor white and viewed on an ultraviolet microscope shows large blue-white fluorescing spherical structures with numerous small round cells inside. What is the most likely causative agent? (A) Candida albicans (B) Coccidioides immitis (C) Histoplasma capsulatum (D) Influenza virus type A (E) Mycoplasma pneumoniae (F) Streptococcus pneumoniae

C. The causative agent can only be Coccidioides immitis from the description of the spherules and endospores in the sputum. Note that no definitive geographic clue is given in this question. Without the results of the microscopic examination it possibly could have been Histoplasma capsulatum or Blastomyces pneumonia. The patient is also the right age group for Mycoplasma pneumonia, but again the microscopic data point instead to Coccidioides.

Membranous glomerulonephritis is found at autopsy in a 25-year-old woman who died in renal failure. Other autopsy findings include pleuritis, diffuse interstitial fibrosis of the lungs, concentric rings of collagen surrounding splenic arterioles, and warty vegetations of the mitral and tricuspid valves affecting the surfaces behind the cusps, as well as the surfaces exposed to the forward flow of blood. Which of the following is an expected laboratory finding? (A) Increased titer of antistreptolysin O (ASO) (B) Lymphocytosis (C) Peripheral rim pattern of antinuclear antibody fluorescence (D) Positive blood cultures for Streptococcus viridans (E) Serum antibodies reactive with glomerular and pulmonary alveolar basement membranes

C. The combination of membranous glomerulonephritis, pleuritis, and Libman-Sacks endocarditis (vegetations on both surfaces of the mitral or tricuspid valves) as well as proliferative splenic arteriolitis is characteristic of SLE. Diffuse interstitial pulmonary fibrosis also occurs in SLE. A variety of antinuclear antibodies (ANAs) are found; the most specific are antibodies to the Sm antigen, antibodies to double-stranded DNA, and antibodies that result in a peripheral rim pattern of nuclear immunofluorescence.

A 50-year-old, HIV-positive man has Hodgkin disease. Lymph nodes on both sides of the diaphragm are involved, as are the liver and bone marrow. Histologic examination of an involved node reveals a diffuse infiltrate with large numbers of Reed- Sternberg cells with many bizarre sarcomatous variants. Tests for Epstein-Barr proteins are positive. From this description, which Hodgkin lymphoma variant is most likely? (A) Nodular lymphocyte predominant (B) Mixed cellularity (C) Lymphocyte depletion (D) Nodular sclerosis

C. The description is that of lymphocyte depletion Hodgkin lymphoma, the least frequently occurring form of Hodgkin lymphoma. It is marked by few lymphocytes, numerous Reed-Sternberg cells, and extensive necrosis and fibrosis. It often presents in an advanced stage and has the poorest prognosis of the Hodgkin lymphoma variants. This variant is associated with EBV infection in the great majority of cases and also is more common in persons infected with HIV.

A 65-year-old woman fell and sustained a pelvic fracture. After a short period of rapidly progressive mental confusion and respiratory insufficiency, the woman died. Numerous conjunctival petechiae were noted. These abnormalities were most likely due to which one of the following conditions? (A) Epidural hematoma (B) Aspiration pneumonia (C) Fat embolization (D) Acute tubular necrosis (E) Saddle embolus occluding bifurcation of pulmonary arteries

C. The fat embolism syndrome occurs 2 to 3 days after severe fracture injury and includes progressive CNS dysfunction and severe respiratory insufficiency. Thrombocytopenia with petechial bleeding is common, and petechial hemorrhages can result from obstruction of the microvasculature by embolic fat droplets. Respiratory insufficiency may be due to injury to pulmonary microvessels with leakage of fluid into the alveoli, resulting in ARDS.

A bone marrow aspiration from a 65-year-old man with long-standing profound anemia shows megaloblastic erythroid hyperplasia. Which of the following is the most likely diagnosis? (A) Anemia of chronic disease (B) Pelger-Huet anomaly (C) Pernicious anemia (D) Homozygous hemoglobin E (E) Thalassemia

C. The hallmark of the megaloblastic anemias is the finding of megaloblastic erythroid hyperplasia in the bone marrow; pernicious anemia is a megaloblastic anemia.

Three days after being admitted to the hospital for treatment of a gunshot wound, a 29-year-old man suffered the onset of acute respiratory distress, and diffuse bilateral infiltrates were seen in both lung fields on chest X-ray. A lung biopsy revealed the presence of intra-alveolar edema, along with hyaline membrane formation. These findings are indicative of (A) bacterial pneumonia. (B) viral pneumonia. (C) diffuse alveolar damage. (D) pulmonary hypertension. (E) left-sided heart failure.

C. The history is consistent with ARDS. ARDS is a cause of severe lifethreatening respiratory insufficiency and may be caused by a variety of etiologic agents, among them severe trauma, such as a gunshot wound. The common feature, regardless of etiology, is diffuse alveolar damage.

During a pre-employment evaluation for an executive position, a 35-year-old man is found to have a serum calcium of 12.4 mg/100 mL (normal 9 to 11), a serum phosphorus (phosphate) of 2.0 mg/100 mL (normal 3.0 to 4.5), a serum alkaline phosphatase of 150 U/L (normal 20 to 70), and a serum parathyroid hormone (PTH) of 800 pg/mL (normal 225 to 650). If untreated, the likely lesion responsible for these findings will result in consequences mediated by which of the following mechanisms? (A) Liquefactive necrosis (B) Apoptosis (C) Metastatic calcification (D) Oxygen toxicity (E) Amyloid deposition

C. The laboratory abnormalities (increased serum calcium, parathyroid hormone, and alkaline phosphatase and decreased serum phosphorus) are diagnostic of primary hyperparathyroidism, a well-known cause of hypercalcemia. One of the consequences of hypercalcemia (regardless of cause) is deposition of calcium salts in previously undamaged organs or tissues, a phenomenon known as metastatic calcification.

A 55-year-old woman has cirrhosis. Twenty years ago she received a blood transfusion for profuse bleeding associated with a complication of childbirth. Shortly thereafter, she had an acute disease diagnosed as non-A, non-B hepatitis. Throughout her lifetime, her alcohol consumption has been minimal. Which of the following viruses is most likely responsible for her past and current liver disease? (A) Hepatitis A (B) Hepatitis B (C) Hepatitis C (D) Hepatitis D (E) Hepatitis E

C. The most frequent cause of transfusion-related hepatitis is hepatitis C virus infection. Hepatitis C virus is the most frequent cause of what was formerly termed non-A, non-B hepatitis.

A 15 month old child living in a religious community that does not vaccinate their children develops meningitis. A Gram-negative rod is seen in the cerebrospinal fluid. What is the most likely causative agent? (A) An enterovirus (B) Escherichia coli (C) Haemophilus influenzae (D) Neisseria meningitidis (E) Streptococcus agalactiae

C. The only Gram-negative rods in the choices are E. coli and H. influenzae. The causative agent is more likely to be Haemophilus influenzae in a child this age.

Your patient presents with recurring pain suggestive of a duodenal ulcer and is positive for antibodies against Helicobacter pylori. What allowed H. pylori to survive the transit through the gastric lumen to start the infection? (A) Buffering capacity generated by the Cag A protein (B) Use of antacids by the patient (C) Production of an urease (D) The presence of a polysaccharide capsule that protects the cells (E) The flagellum and chemotaxis that moves the organism quickly to the mucin layer

C. The production of the urease (C) produces ammonia, which neutralizes the stomach acid in the immediate environment of each H. pylori as it migrates to the mucosa. Choice E is also part of the pathogenesis, but the question asks specifically about the survival during the transit.

A 7 year old boy develops vesicular skin lesions on his trunk, but they quickly spread to his extremities and appear on the scalp. Scrapings from the vesicles show multinucleated cells and Cowdry type A inclusion bodies. This disease could have been prevented with a vaccine composed of (A) Capsid protein (B) Inactivated virions (C) Live attenuated virus strain (D) Live reassortment virus (E) Vaccinia virus carrying capsid protein gene

C. The young boy has chickenpox caused by VZV; it can be prevented by immunization with the live attenuated Oka strain of the virus.

A mother brings in her newborn baby girl and says, "My baby coughs and gags every time I try to feed her; one time she even turned blue and it scared me." The mother also indicates that her baby always has a "mouthful of saliva." Relevant physical exam findings include a distended stomach, excessive saliva accumulation, a hint of pneumonitis, and inability to pass a catheter into the infant's stomach. Relevant laboratory findings include radiograph showing a large amount of air in the stomach. Which of the following is the most likely diagnosis? (A) Esophageal hiatal hernia (B) Hypertrophic pyloric stenosis (C) Tracheoesophageal fistula (D) Respiratory distress syndrome (E) Congenital diaphragmatic hernia

C. Tracheoesophageal fistula. Tracheoesophageal fistula is an abnormal communication between the trachea and esophagus that results from improper division of foregut by the tracheoesophageal septum. It is generally associated with esophageal atresia and polyhydramnios. Clinical features include excessive accumulation of saliva or mucus in the nose and mouth; episodes of gagging and cyanosis after swallowing milk; abdominal distention after crying; and reflux of gastric contents into lungs, causing pneumonitis. Diagnostic features include inability to pass a catheter into the stomach and radiographs demonstrating air in the infant's stomach.

Trastuzumab works by (A) Inhibiting the oncoprotein bcr-abl (B) Blocking estrogen-mediated gene transcription (C) Preventing phosphorylation of a receptor tyrosine kinase (D) Targeting cells for destruction by antibody-mediated cellular cytotoxicity (ADCC) (E) Reducing circulating levels of tumor necrosis factor (TNF)

C. Trastuzumab is an antibody to the extracellular domain of the receptor tyrosine kinase HER2/neu. In some breast cancers, HER2/neu is expressed in high levels leading to auto-phosphorylation in the absence of ligand binding. Trastuzumab blocks such signaling. Imatinib is used in treating chronic myelogenous leukemia and inhibits bcr-abl. Tamoxifen functions by inhibiting estrogen-mediated gene transcription. Rituximab targets CD20+ cells in B-cell lymphomas for ADCC. Thalidomide works in part by inhibiting TNF production.

Which of the following might be considered for the treatment of acute myelocytic anemia (M3 variant)? (A) Cisplatin (B) Lomustine (C) Tretinoin (D) Fluorouracil (E) Streptozocin

C. Tretinoin is all-trans-retinoic acid and produces remission by inducing differentiation in the M3 variant of acute myelogenous leukemia (AML), characterized by aberrant expression of a retinoic receptor-á gene. Cisplatin is often used in the treatment of cancers of the lung, head, and neck. Lomustine has good central nervous system (CNS) penetration and is used in treating brain tumors. Fluorouracil is also used in treating multiple tumors including those of the breast and colon. Lastly, streptozocin is used in the treatment of insulinomas.

A characteristic of patients with systemic lupus erythematosus (SLE) is (A) A linear deposition of immunoglobulin on the glomerular basement membrane (B) Antibody against the thyroid receptor (C) Vasculitis (D) Absence of antibodies to double-stranded DNA (E) Each of the above characteristics are present

C. Vasculitis is a prominent feature of SLE.

Vasopressin . . . (A) Reduces ADH levels (B) Increases Na+ permeability of the collecting duct (C) Inserts aquaporins into the plasma membrane of collecting duct cells (D) Increases diffusion of sodium (E) Reduces production of prostaglandins

C. Vasopressin causes specific water channels termed aquaporins II to be inserted into the plasma membrane of the luminal surface of the medullary collecting ducts. This directly affects permeability of the collecting duct (choice B). Under the conditions of dehydration, as is the case with this patient, the ADH levels increase (choice A). Choice D represents the mechanism of action of osmotic diuretics. Production of prostaglandins is reduced with the use of agents such as indomethacin (choice E).

After extensive oral surgery, a 68-year-old patient who had rheumatic fever as a child and did not take the prescribed perioperative prophylactic antibiotics subsequently developed subacute infective endocarditis. Which of the following is the most likely causative agent? (A) Enterococcus faecalis (B) Staphylococcus aureus (C) Viridans streptococci (D) Streptococcus agalactiae (E) Streptococcus pneumoniae (F) Streptococcus pyogenes

C. Viridans streptococci, which are part of the normal oral flora in humans, are noted for their ability to attach to damaged heart valves when they enter the circulation after oral surgery. If you answered Strep. pyogenes, the original trigger for rheumatic fever was an untreated Strep. pyogenes pharyngitis. Each additional exposure creates risk of additional damage. So if his school age grandkids were coming for a week during Kwanza, then he would probably need to have prophylactic antibiotics. But the question only asks about major dental work.

An 80-year-old woman on a "tea and toast" diet presents with bleeding gums, petechiae and easy bruising, and pain in her arms and legs. She states that she almost never eats fruits and vegetables. She is diagnosed with vitamin C deficiency. Which of the following is the basis of the clinical abnormalities that occur as a result of lack of vitamin C? (A) Defective calcification of osteoid matrix (B) Increased intestinal absorption of iron (C) Defective hydroxylation of proline and lysine (D) Increased proliferation of collagen and fibrous tissue (E) Destruction of endothelial cells

C. Vitamin C is required for hydroxylation of proline and lysine residues, which are required steps in collagen and osteoid matrix synthesis. Poor collagen formation contributes to impaired wound healing and fragility of capillary walls, which in turn leads to abnormal bleeding. Vitamin C also maintains the reduced state of metabolically active agents, such as iron and tetrahydrofolate. The maintenance of iron in its divalent ferrous form is required for intestinal iron absorption. Thus iron absorption is decreased, rather than increased, in vitamin C deficiency. Defective osteoid matrix formation occurs in vitamin C deficiency.

A 4 year old who has received no vaccines because of parental fear of autism develops Streptococcal pneumoniae meningitis. Which vaccine might have prevented this? (A) A 7-valent capsular vaccine (B) A 23-valent polysaccharide (C) A conjugate (protein-polysaccharide) vaccine (D) A toxoid vaccine

C. Yes, the pediatric vaccine is heptavalent but it is a heptavalent conjugate vaccine so the best answer is the protein polysaccharide conjugate vaccine. The 23-valent polysaccharide (choice B) is the vaccine used for the elderly and high-risk people.

Arterial Pco2 of 72 mm Hg, arterial [HCO3 -] of 38 mEq/L, and increased H+ excretion would be observed in a (A) patient with chronic diabetic ketoacidosis (B) patient with chronic renal failure (C) patient with chronic emphysema and bronchitis (D) patient who hyperventilates on a commuter flight (E) patient who is taking a carbonic anhydrase inhibitor for glaucoma (F) patient with a pyloric obstruction who vomits for 5 days (G) healthy person

C. [Chapter 5, IX D 3; Table 5.9]. The blood values are consistent with respiratory acidosis with renal compensation. The renal compensation involves increased reabsorption of HCO3 - (associated with increased H+ secretion), which raises the serum [HCO3 -].

A decrease in which of the following parameters in an artery will produce an increase in pulse pressure? (A) Blood flow (B) Resistance (C) Pressure gradient (D) Capacitance

D [Chapter 3, II E]. A decrease in the capacitance of the artery means that for a given volume of blood in the artery, the pressure will be increased. Thus, for a given stroke volume ejected into the artery, both the systolic pressure and pulse pressure will be greater.

In which of the following situations will arterial PO2 be closest to 100 mm Hg? (A) A person who is having a severe asthmatic attack (B) A person who lives at high altitude (C) A person who has a right-to-left cardiac shunt (D) A person who has a left-to-right cardiac shunt (E) A person who has pulmonary fibrosis

D [Chapter 3, VI D]. In a person with a left-to-right cardiac shunt, arterial blood from the left ventricle is mixed with venous blood in the right ventricle. Therefore, Po2 in pulmonary arterial blood is higher than normal, but systemic arterial blood would be expected to have a normal Po2 value or 100 mm Hg. During an asthmatic attack, Po2 is reduced because of increased resistance to airflow. At high altitude, arterial Po2 is reduced because the inspired air has reduced Po2. Persons with a right-to-left cardiac shunt have decreased arterial Po2 because blood is shunted from the right ventricle to the left ventricle without being oxygenated or "arterialized." In pulmonary fibrosis, the diffusion of O2 across the alveolar membrane is decreased.

Hypoxia causes vasoconstriction in which of the following vascular beds? (A) Cerebral (B) Coronary (C) Muscle (D) Pulmonary (E) Skin

D [Chapter 3, VIII C-F; Table 3.3]. Both the pulmonary and coronary circulations are regulated by Po2. However, the critical difference is that hypoxia causes vasodilation in the coronary circulation and vasoconstriction in the pulmonary circulation. The cerebral and muscle circulations are regulated primarily by local metabolites, and the skin circulation is regulated primarily by sympathetic innervation (for temperature regulation).

Which diuretic inhibits Na+ reabsorption and K+ secretion in the distal tubule by acting as an aldosterone antagonist? (A) Acetazolamide (B) Chlorothiazide (C) Furosemide (D) Spironolactone

D [Chapter 5, IV C 3 b (1); Table 5.11]. Spironolactone inhibits distal tubule Na+ reabsorption and K+ secretion by acting as an aldosterone antagonist.

Arterial pH of 7.52, arterial Pco2 of 26 mm Hg, and tingling and numbness in the feet and hands would be observed in a (A) patient with chronic diabetic ketoacidosis (B) patient with chronic renal failure (C) patient with chronic emphysema and bronchitis (D) patient who hyperventilates on a commuter flight (E) patient who is taking a carbonic anhydrase inhibitor for glaucoma (F) patient with a pyloric obstruction who vomits for 5 days (G) healthy person

D [Chapter 5, IX D 4; Table 5.9]. The blood values are consistent with acute respiratory alkalosis from hysterical hyperventilation. The tingling and numbness are symptoms of a reduction in serum ionized [Ca2+] that occurs secondary to alkalosis. Because of the reduction in [H+], fewer H+ ions will bind to negatively charged sites on plasma proteins, and more Ca2+ binds (decreasing the free ionized [Ca2+]).

A 32-year-old woman who is thirsty has a urine osmolarity of 950 mOsm/L and a serum osmolarity of 297 mOsm/L. Which diagnosis is correct? (A) Primary polydipsia (B) Central diabetes insipidus (C) Nephrogenic diabetes insipidus (D) Water deprivation (E) Syndrome of inappropriate antidiuretic hormone (SIADH)

D [Chapter 5, VII A 1; Table 5.6; Figure 5.14]. The description is of a normal person who is deprived of water. Serum osmolarity is slightly higher than normal because insensible water loss is not being replaced by drinking water. The increase in serum osmolarity stimulates (via osmoreceptors in the anterior hypothalamus) the release of antidiuretic hormone (ADH) from the posterior pituitary. ADH then circulates to the kidney and stimulates water reabsorption from the collecting ducts to concentrate the urine.

The receptor for this hormone has tyrosine kinase activity. (A) Adrenocorticotropic hormone (ACTH) (B) Antidiuretic hormone (ADH) (C) Aldosterone (D) Insulin (E) Parathyroid hormone (PTH) (F) Somatostatin

D [Chapter 7; Table 7.2]. Hormone receptors with tyrosine kinase activity include those for insulin and for insulin-like growth factors (IGF). The â subunits of the insulin receptor have tyrosine kinase activity and, when activated by insulin, the receptors autophosphorylate. The phosphorylated receptors then phosphorylase intracellular proteins; this process ultimately results in the physiologic actions of insulin.

A 10-year-old girl presents with an orange-red, dome-shaped papule on her right leg. Her parents state that the papule has grown from a tiny dot to over a centimeter in length in just a few months. Following biopsy, the dermatologist reassures the parents, stating that although the papule superficially resembles malignant melanoma, it is actually a benign lesion without malignant potential. Which of the following is the likely diagnosis? (A) Acanthosis nigricans (B) Actinic keratosis (C) Dysplastic nevus (D) Spitz nevus (E) Xeroderma pigmentosum

D. A Spitz nevus is a pediatric skin neoplasm that can be easily confused with malignant melanoma based on gross and microscopic morphology. It was formerly called "juvenile melanoma" however this name has fallen out of favor because it evokes unnecessary alarm, given the typically benign course of these tumors. Acanthosis nigricans is sometimes an indicator of visceral malignancy. Actinic keratosis is a premalignant epidermal lesion. Dysplastic nevus may transform into malignant melanoma. Xeroderma pigmentosum is associated with a markedly increased incidence of skin cancer caused by failure of DNA repair.

A 40-year-old man comes to your office complaining that he has a gradual swelling in the front of his neck that has been growing over the last 6 months. He says, "Doc, at first I did not pay any attention to it, but now it is so big that other people are starting to notice it." He does not complain of any pain, difficulty in swallowing, or problems with breathing. There is no history of trauma, fever, or change in his voice. A physical exam reveals a nontender, nonerythematous fluctuant mass in the midline lower neck with a slight extension to the right side of the neck. The mass moves up and down when the patient swallows and displaces anteriorly with protrusion of the tongue. No cervical lymphadenopathy is noticed. The lung fields are clear, and the heart rate and rhythm are normal. You order a CT scan, which shows a cystic mass extending to the thyroid gland and under the strap muscles. Routine lab blood tests and thyroid function tests are all normal. Which of the following is the most likely diagnosis? (A) Esophageal hiatal hernia (B) Hypertrophic pyloric stenosis (C) Tracheoesophageal fistula (D) Thyroglossal duct cyst (E) Congenital diaphragmatic hernia

D. A thyroglossal duct cyst occurs when parts of the thyroglossal duct persist and thereby form a cyst. These cysts can be present anywhere along the line of descent in fetal development of the thyroid gland, from the foramen cecum at the base of the tongue to the level of the thyroid gland. There are four types of thyroglossal duct cysts: thyrohyoid cysts (61% of cases), suprahyoid cysts (24%), suprasternal cysts (13%), and intralingual cysts (2%).

A 50-year-old woman had a partial colectomy performed for adenocarcinoma of the sigmoid colon, with apparent complete and uneventful recovery. At follow-up visits, her physician is particularly interested in changes that may occur in which of the following laboratory measures? (A) Human chorionic gonadotropin (hCG) (B) á-fetoprotein (AFP) (C) Vanillylmandelic acid (VMA) (D) Carcinoembryonic antigen (CEA) (E) Estriol

D. Although too nonspecific for initial diagnosis or screening, CEA is useful for follow-up of cancer of the colon. Increased serum hCG may be observed in normal pregnancy, hydatidiform mole, choriocarcinoma, and many testicular mixed germ cell tumors. Fetal neural tube defects are associated with increased levels of AFP in the mother. In addition, elevated levels of AFP (unrelated to pregnancy) may be associated with hepatocellular carcinoma, yolk sac (endodermal sinus) tumors, and some nonseminomatous germ cell tumors of the testes. VMA is a marker for neuroblastoma and pheochromocytoma. Granulosa cell tumors and thecomas of the ovary characteristically produce estrogen.

A nonstimulant agent that can be used to treat attention-deficit/hyperactivity disorder (ADHD) is (A) Methylphenidate (B) Caffeine (C) Dextroamphetamine (D) Atomoxetine (E) Modafinil

D. Atomoxetine is a nonstimulant drug used in the management of attention- deficit/hyperactivity disorder (ADHD) that works by inhibiting norepinephrine reuptake. The stimulate agents used for the treatment of ADHD include methylphenidate and dextroamphetamine and work by inhibiting dopamine reuptake. Caffeine is a stimulant in many beverages, which may have some role in the management of some headaches. Modafinil is a newer agent used in the treatment of narcolepsy.

What microbe has both a live-attenuated vaccine and an inactivated vaccine currently in routine use? The inactivated one is used for anyone under 2 years or over 60, or for anyone who is immunocompromised in any way. (A) Streptococcus pneumoniae (B) Diphtheria, tetanus, pertussis (C) Poliovirus (D) Influenza virus (E) Hepatitis B

D. Besides the clues of both a live and killed vaccines in routine use, you also had the fact that the vaccine was given across the spectrum of ages. Of the vaccines listed, only pneumococcal, DTP, and influenza fit that criterium. Two influenza vaccines are in current use. The live (attenuated) one is given intranasally and the killed (inactivated) one is injected. Streptococcus pneumoniae does have two vaccines, but neither is live attenuated; the pediatric pneumococcal vaccine is 7-capsular polysaccharides chemically complexed to protein. The 23-valent polysaccharide pneumococcal vaccine is used for anyone 65 years and older. Choice C is poliovirus; in the United States the attenuated strains were previously routinely used to vaccinate (Sabin vaccine given orally), but the incidence of vaccine-associated polio became too high and the United States switched to an injectable, inactivated tetravalent polio vaccine. Hepatitis B vaccine is a recombinant, single component vaccine with only the HBsAg present.

A 32-year-old HIV-positive man follows up with an infectious disease (ID) specialist in the clinic. The results of his recent blood work suggest that the virus has become resistant to multiple nucleoside reverse transcriptase inhibitors. The ID specialist decides to include in the treatment a nonnucleoside inhibitor (nevirapine), which works by binding to a site near the active site on the reverse transcriptase. Nevirapine is an example of what? (A) Full agonist (B) Reversible competitive antagonist (C) Partial agonist (D) Noncompetitive antagonist (E) Irreversible competitive antagonist

D. By definition, drugs that do not bind to the active site, such as nonnucleotide reverse transcriptase inhibitors, are noncompetitive antagonists. They function by causing changes in the active site so that it cannot bind its native substrate. Agonists are drugs that elicit the same activity as the endogenous substrate, whereas partial agonists only induce some of the activities of the endogenous substrate. Competitive inhibitors, like nucleoside reverse transcriptase inhibitors, can be either reversible or irreversible.

Which complement component is most closely related to anaphylatoxin? (A) C1qrs (B) C2b (C) C4b2a (D) C5a (E) C5b

D. C5a binds to mast cells and basophils to induce histamine, smooth muscle contraction, and other mediators, which increase vascular permeability.

The hospital blood bank desires to isolate stem cells from a patient with multiple myeloma for transplantation following his radiation treatment. The cell marker important for isolating and identifying human stem cells is (A) CD3 (B) CD4 (C) CD28 (D) CD34

D. CD34 is selective for human stem cells. CD3 is found on all T cells. CD28 is an accessory T-cell adhesion molecule, while CD4 is a marker for T helper cells.

In an experimental model, an autopsy study of a rat exposed to toxic doses of carbon tetrachloride (CCl4) revealed fatty change and necrosis of hepatocytes. The mechanism of cell injury exemplified here is (A) activation of apoptosis. (B) acute phase reaction. (C) arrest of cell cycle. (D) free radical injury. (E) thrombosis and ischemia.

D. Carbon tetrachloride (CCl4) injury is the classic model of membrane injury incurred by free radical formation. In this instance, CCl4 is converted to the active free radical CCl3 in the smooth endoplasmic reticulum by the P450 system of mixed function oxidases.

68-year-old man presented with painless hematuria. Cystoscopic examination revealed a papillary tumor. Excisional biopsy was performed, and the diagnosis of transitional cell carcinoma was made. Which of the following is a well- known association of this type of cancer? (A) Early hematogenous spread (B) Exposure to aflatoxin (C) Long-term use of methyldopa, an antihypertensive agent (D) Industrial exposure to aniline dyes, such as â-naphthylamine (E) Schistosoma haematobium infection

D. Carcinoma of the urinary bladder, almost always transitional cell carcinoma, is associated with industrial exposure to aniline dyes, such as â-naphthylamine, usually many years in the past. This type of cancer most often spreads by local extension to surrounding tissues. Hematogenous dissemination is a late finding. Exposure to aflatoxin is associated with hepatocellular carcinoma, and Schistosoma haematobium infection is associated with squamous cell bladder carcinoma, not transitional cell. Methyldopa is an association of bile duct cancer.

A 67-year-old man, a two-pack-a-day smoker since age 18, has had a productive cough over the past 20 years. Although continuous through the years, there have been episodic exacerbations of these symptoms, which have worsened during the past 4 or 5 years, lasting 3 or 4 months at a time. Arterial pO2 is decreased and pCO2 is increased. Total lung capacity measurements are normal. These findings are most suggestive of which of the following pulmonary disorders? (A) Adult respiratory distress syndrome (ARDS) (B) Bronchial asthma (C) Lung carcinoma (D) Chronic bronchitis (E) Panacinar emphysema

D. Chronic bronchitis, which is clearly linked to cigarette smoking, is defined as productive cough occurring during at least 3 consecutive months over at least 2 consecutive years.

Which of the following is a predisposing factor in the pathogenesis of clear cell adenocarcinoma of the vagina? (A) Excess estrogen stimulation (B) Herpes simplex virus infection (C) HPV infection (D) In utero exposure to DES (E) Oral contraceptive therapy

D. Clear cell adenocarcinoma of the vagina is a rare malignant tumor that is markedly increased in incidence in daughters of women who received DES therapy during pregnancy.

Clonidine works by (A) Activating â1-adrenergic receptors (B) Activating á1-adrenergic receptors (C) Activating â2-adrenergic receptors (D) Activating á2-adrenergic receptors (E) Blocking â-adrenergic receptors

D. Clonidine activates prejunctional á2-adrenergic receptors in the central nervous system (CNS) to reduce sympathetic tone, thereby decreasing blood pressure. Activation of á1-adrenergic receptors increases blood pressure, which is useful for the treatment of hypotension. â1-Adrenoreceptor agonists are used primarily for increasing heart rate and contractility. â2-Adrenergic agonists are used to dilate airways in the management of asthma. â-Adrenoreceptor antagonists are used in the treatment of angina and hypertension.

A patient on antibiotics to clear up a community acquired Pantone-Valentine leukocidinproducing Staphylococcus aureus has clindamycin added to his treatment regimen to slow the production of the various toxins including PVL. He recovers but develops Clostridiumdifficile diarrhea. Why does C. difficile cause diarrhea? (A) C. difficile multiplies and causes disease in a mechanism similar to that of Listeria (B) C. difficile multiplies, secreting two surface toxins during cell lysis (C) C. difficile stacks itself on the colonic surface to cause malabsorption and the appearance of a pseudomembrane (D) C. difficile exotoxins damage the cells, causing a disruption in transport and attracting polymorphonuclear cells to cause the appearance of a pseudomembrane

D. Clostridium difficile causes the production of exotoxins, which cause diarrhea and production of the pseudomembrane.

In this same case what virulence factor does the above agent capable of causing an ascending urinary tract and pyelonephritis have that is missing in its normal flora ''fraternal twin'' with the same genus and species? (A) Flagellum (B) Capsule (C) Lipoteichoic acid (D) P-pili (E) LT (heat labile toxin)

D. Common pili of Escherichia coli bind to the colonic mucosa but not to the ureoepithelium, so, in order to cause ascending urinary tract infections and not just be washed out by periodic urine flow, the E. coli has to have either x-adhesins (not a choice) or pyelonephritis associated pili (P-pili).

It is likely that the acquisition of antibiotic resistance in gram-negative bacilli such as vancomycin is a result of (A) Spontaneous mutation (B) Transformation (C) Transduction (D) Conjugation (E) Transposition

D. Conjugation is the principal mechanism for the acquisition of antibiotic resistance among enterobacteria and involves the transfer of resistance transfer factors on plasmids through sex pili. The other mechanisms for gene transfer, including random mutation, transformation, transduction, and transposition are not as common among these organisms.

Which of the following is an alkylating agent that may cause hemorrhagic ceptitis and cardiomyopathy? (A) Azathioprine (B) Cyclosporine (C) Tacrolimus (D) Cyclophosphamide (E) Basiliximab

D. Cyclophosphamide has been successfully used for the treatment of lupus nephritis; however, it does carry significant morbidity associated with its use. Azathioprine works by suppressing T-cell activity (A). Cyclosporine inhibits T-helper cell activation (B). Tacrolimus inhibits transcription of T-cell-specific lymphokines (C). Baciliximab is a monoclonal antibody against CD-25 used to reduce the incidence and severity of renal transplant rejection.

Which of the following is a side effect of clindamycin? (A) Dizziness (B) Bruising (C) Difficulty in hearing (D) Diarrhea (E) Tendon pain

D. Diarrhea due to pseudomembranous colitis with Clostridium difficile overgrowth is common with many broad-spectrum antibiotics, especially clindamycin. Bruising can occur with some cephalosporins. Dizziness is common with tetracyclines, such as minocycline. Ototoxicity can result in hearing loss with vancomycin and aminoglycosides. Tendon pain is possible due to the cartilage toxicity associated with fluoroquinolones.

A 43-year-old high-profile attorney sees a psychiatrist with expertise in addiction medicine. He explains that he has recently received his third drunk driving citation and fears losing his license to practice unless he stops drinking altogether. He says he "just can't stop" once he starts. He tells the physician that he doesn't have time to attend Alcoholics Anonymous and "wants a pill." The physician explains that there is something that might work if the patient is truly serious. What agent is the physician considering? (A) Lorazepam (B) Flumazenil (C) Naloxone (D) Disulfiram (E) Carbamazepine

D. Disulfiram is an inhibitor of aldehyde dehydrogenase, which blocks the breakdown of acetaldehyde to acetate during the metabolism of alcohol. The buildup of acetaldehyde results in flushing, tachycardia, hypertension, and nausea to invoke a conditioned response to avoid alcohol ingestion, including that found in over-the-counter medications and in some foods. Lorazepam is useful in the prevention of seizures as a result of alcohol withdrawal, whereas carbamazepine is used should they develop. Flumazenil is used for benzodiazepine overdose and naloxone for opioid overdose.

A 40-year-old mother brings in her 4-weekold baby boy and tells you that "my baby's face looks funny and he keeps sticking his tongue out." The mother recalls that during the pregnancy she had low á-fetoprotein (AFP) levels. Relevant physical exam findings include a flat occiput; white spots in the iris (Brushfield spots); a large, protruding tongue; small, lowset ears; short feet and hands; a flexion crease across the palms (simian crease); curvature of the fifth digit; systolic ejection murmur; and hypotonia. Relevant laboratory findings include echocardiogram showing an endocardial cushion defect (atrioventricular septal defect) and karyotype analysis showing an extra chromosome 21. Which of the following is the most likely diagnosis? (A) Cri-du-chat syndrome (B) Edwards syndrome (C) Fragile X syndrome (D) Down syndrome (E) Patau syndrome

D. Down syndrome (Trisomy 21) is characterized by moderate mental retardation (leading cause of inherited mental retardation), microcephaly, microphthalmia, colobomata, cataracts and glaucoma, flat nasal bridge, epicanthic folds, protruding tongue, simian crease in hand, increased nuchal skin folds, appearance of an "X" across the face when the baby cries as the upward-slanted palpebral fissures run in a line with the nasolabial folds, and congenital heart defects. Alzheimer neurofibrillary tangles and plaques are found in Down syndrome patients after 30 years of age. Acute megakaryocytic leukemia (AMKL) is frequently present. Trisomy 21 is the most common type of trisomy, and its frequency increases with advanced maternal age. Trisomy 21 is associated with low á-fetoprotein levels in amniotic fluid or maternal serum. A specific region on chromosome 21 seems to be markedly associated with numerous features of Trisomy 21. This region is called DSCR (Down syndrome critical region). The genes for the following have been mapped to the DSCR (although their role is far from clear): carbonyl reductase, SIM2 (a transcription factor), p60 subunit of chromatin assembly factor, holocarboxylase synthetase, ERG (a protooncogene), GIRK2 (a K1 ion channel), and PEP19 (a Ca21-dependent signal transducer). Trisomy 21 may also be caused by a chromosomal translocation between chromosomes 14 and 21

A 25-year-old woman comes into your office complaining of "spotting" and having "stomach pains" as she points to her lower abdominal area. She noted that she and her husband were trying to have a baby and that she had her last period about 5 weeks ago. She said that after talking with her girlfriends about her symptoms, she was a little afraid of what it could be, and so she decided to see a physician. Her chart shows that she has had a history of pelvic inflammatory disease. Relevant physical exam findings include a tender pelvic mass that was palpable, amenorrhea, light vaginal bleeding, and lower abdominal pain. Relevant laboratory findings include elevated â-human chorionic gonadotropin (hCG) levels but lower than expected for pregnancy, lower-than-normal progesterone levels, and ultrasound that showed a mass in the ampulla of the left uterine tube. Which of the following is the most likely diagnosis? (A) Choriocarcinoma (B) A bleeding corpus luteum (C) A spontaneous abortion (D) Ectopic tubal pregnancy (E) Appendicitis

D. Ectopic tubal pregnancy (ETP) occurs when the blastocyst implants within the uterine tube due to delayed transport. The ampulla of the uterine tube is the most common site of an ectopic pregnancy. The rectouterine pouch (pouch of Douglas) is a common site for an ectopic abdominal pregnancy. ETP is most commonly seen in women with endometriosis or pelvic inflammatory disease. ETP leads to uterine tube rupture and hemorrhage if surgical intervention (i.e., salpingectomy) is not performed. ETP presents with abnormal uterine bleeding and unilateral pelvic pain, which must be differentially diagnosed from appendicitis, an aborting intrauterine pregnancy, or a bleeding corpus luteum of a normal intrauterine pregnancy. See Chapter 2, IV.A.

A 25-year-old woman who is 32 weeks pregnant comes into the emergency room while in labor. The infant is stillborn. The mother is obviously upset and says "I want to know what happened." Although the mother had no prenatal care, she says "I am shocked that something went wrong because I had no problems with my first pregnancy; that baby is fine." The mother is sincere when she states that she did not smoke or drink alcohol during the pregnancy. The mother says, "Everything was going along just fine with this pregnancy until just few hours ago." The mother requests an autopsy on the infant. Relevant physical findings of the autopsy include the finding that the body is swollen and jaundiced; yellow deposits in several areas of the brain, especially the basal ganglia; and ascites. Relevant laboratory findings of the autopsy include severe anemia, high serum bilirubin levels, and infant's blood type O positive. Further lab tests were done on the blood of the mother and father. The mother's blood was O negative. The father's blood was O positive. Which of the following is the most likely diagnosis? (A) Oligohydramnios (B) Polyhydramnios (C) Severe preeclampsia (D) Erythroblastosis fetalis (E) Placental abruption

D. Erythroblastosis fetalis. The Rh factor is clinically important in pregnancy. If the mother is Rh-, she will produce Rh antibodies if the fetus is Rh1. This situation will not affect the first pregnancy but will affect the second pregnancy with an Rh1 fetus. In the second pregnancy with an Rh1 fetus, a hemolytic condition of red blood cells (RBCs) occurs known as Rhhemolytic disease of newborn (erythroblastosis fetalis). This causes destruction of fetal RBCs, which leads to the release of large amounts of bilirubin (a breakdown product of hemoglobin). This causes fetal brain damage due to a condition called kernicterus, which is a pathological deposition of bilirubin in the basal ganglia. Severe hemolytic disease in which the fetus is severely anemic and demonstrates total body edema (i.e., hydrops fetalis) may lead to death. In these cases, an intrauterine transfusion is indicated. Rh0(D) immune globulin (RhoGAM, MICRhoGAM) is a human immunoglobulin (IgG) preparation that contains antibodies against Rh factor and prevents a maternal antibody response to Rh1 cells that may enter the maternal bloodstream of an Rh-mother. This drug is administered to Rh- mothers within 72 hours after the birth of an Rh1 baby to prevent erythroblastosis fetalis during subsequent pregnancies.

A 65-year-old alcoholic is found to have megaloblastic anemia. He has no neurologic abnormalities. Serum and erythrocyte folate are markedly decreased, and serum vitamin B12 is normal. Which of the following is an expected characteristic of this form of anemia? (A) Decreased susceptibility to malaria (B) Hypochromic erythrocytes (C) Increased mean corpuscular hemoglobin concentration (MCHC) (D) Increased mean corpuscular volume (MCV) (E) Suppressed â-chain synthesis

D. Folate deficiency leads to megaloblastic anemia. The red cells are macrocytic (increased MCV) and normochromic (increased MCHC). Since the thickened red cells appear denser on peripheral blood smears, these cells are often erroneously thought to be hyperchromic; however, the MCHC is normal. An increased MCHC is found in hereditary spherocytosis. Hypochromic erythrocytes are typical of iron deficiency anemia, some cases of the anemia of chronic disease, and the thalassemias. Suppressed â-chain synthesis is characteristic of the â-thalassemias. Decreased susceptibility to malaria is associated with absence of the Duffy blood group antigen and with G6PD deficiency.

A nonnucleoside analog that inhibits herpesvirus DNA replication is (A) Acyclovir (B) Amantadine (C) Cytarabine (D) Foscarnet (E) Interferon

D. Foscarnet inhibits herpesvirus DNA polymerase directly.

A 68-year-old woman has fever, generalized lower abdominal pain, and bright red blood in the stools. The white blood cell count is 15,000/mL, with 85% segmented neutrophils. The most likely diagnosis is (A) acute appendicitis. (B) carcinoma of the rectum. (C) Crohn disease. (D) diverticulitis. (E) tubular adenoma in sigmoid colon.

D. Generalized lower abdominal pain, bloody stools, and signs of acute inflammation in an older patient are classic findings in diverticulitis. Appendicitis and Crohn disease occur more often in younger persons, and bloody stools would not be expected. Signs of acute inflammation would not be expected in carcinoma of the rectum or in tubular adenoma.

Both division and differentiation of B cells leading to production of plasma cells require (A) CD4 and CD8 cells (B) Interleukin (IL)-1 and IL-3 (C) Only IL-1 (D) IL-4 and IL-6

D. IL-4 and IL-6 are required for division and differentiation of B cells. IL-1 and IL-3 appear to play no role. CD8 cells are functional in cytotoxic cell production.

A 16-year-old boy is referred to a dermatologist by an emergency room physician. The patient has had an intractable, severe, itching, burning, pruritic rash of the hands and lower extremities that has lasted several hours. Earlier in the day he had hiked through a wooded area filled with brush but was unaware of any direct contact with plants or other possible irritants. A skin biopsy revealed many infiltrating T cells and macrophages, suggesting an immune hypersensitivity reaction. Which type of reaction is most likely? (A) Type I (immediate or anaphylactic) hypersensitivity (B) Type II (antibody-mediated or cytotoxic) hypersensitivity (C) Type III (immune complex) hypersensitivity (D) Type IV (cell-mediated) hypersensitivity

D. In spite of the somewhat atypical presentation, the histologic findings are typical of contact dermatitis, which is a classic example of T cell-mediated (type IV) hypersensitivity.

An 18-year-old college freshman living in a university residence hall develops severe headache, neck stiffness, and fever. When he cannot be aroused, he is transported to the hospital where a Gram stain of a centrifuged cerebral spinal fluid shows Gram-negative bacteria. A latex particle agglutination test confirms the etiological cause of the bacterial meningitis. Which causative agent is most likely? (A) Escherichia coli (B) Haemophilus influenzae (C) Listeria monocytogenes (D) Neisseria meningitidis (E) Streptococcus pneumoniae

D. In the specific patient (young adult, new stresses, crowded living conditions such as residence halls or potentially, college bars), your first suspicion even before the Gram stain and LPA tests should be Neisseria meningitidis, which was confirmed by the Gram stain. E. coli would be unlikely unless he had a CNS shunt or was a neonate. H. influenzae would be unlikely in anyone except a 3 month old to 5 year old child who had not been vaccinated as a child.

Which bacterial gene transfer process would be inhibited by free extracellular exonucleases? (A) Conjugation (B) Generalized transduction (C) Specialized transduction (D) Transformation (E) Transposition

D. In transformation, the DNA is extracellular before it is picked up by the competent cells; during this period, the DNA is subject to the extracellular exonucleases. Because the DNA in generalized and specialized transductions is protected extracellularly by the virus capsid, it is not subject to extracellular exonucleases. In conjunction, the DNA is never outside of a cell. Transposition is a mechanism of inserting a transposon into another molecule of DNA and has no extracellular transport mechanism associated with it.

A patient who is diabetic and been in the hospital twice in the last year presents with a boil on his neck. You drain it, but because of his hospitalizations realize that there is a good chance that it could be a drug resistant strain and order direct Gram stain cultures and susceptibility testing and anaerobic culture as well. The preliminary report comes back with Gramnegative cocci in clusters, which puzzles you, so you call the lab supervisor who realizes the stain was done by a new lab person. It turns out to be Staphylococcus aureus. What mistake in the Gram stain did the new person most likely make to produce pink cocci with no hint of purple? (A) She left the safranin on too long (B) She decolorized a bit too long (C) She forgot the decolorization completely (D) She forgot the Gram's iodine (E) She forgot the safranin

D. It is most likely that she forgot to put the iodine on so the dye does not form the large complex, and the uncomplexed crystal violet will decolorize even Gram-positive organisms. It was also a sharp physician who realized that there are no Gram-negative cocci arranged in clusters and that abscesses are either Staph. or rod-shaped anaerobes or mixed infections.

The Pap smear of a female college soccer player contains koilocytotic cells. The disease associated with the appearance of the cells could most likely have been prevented by previous administration of a (A) Inactivated virus vaccine (B) Inactivated whole cell vaccine (C) Live attenuated virus vaccine (D) Recombinant vaccine containing virus-like particles (E) Toxoid vaccine

D. Koilocytoic cells have enlarged nuclei and cytoplasmic vacuoles are produced from HPV-infected epithelial cells. The Gardasil vaccine employed to prevent HPV genital disease is a recombinant quadrivalent vaccine containing viruslike particles of types 6, 11, 16, and 18 HPV.

A 35-year-old intravenous drug abuser in a methadone maintenance program is admitted to the hospital for a work-up of suspected pulmonary tuberculosis. While in the hospital, he complains of diarrhea and cramping. After stool studies return with a negative result, you decide to begin an antidiarrheal. Which of the following is a good choice for this patient? (A) Kaolin (B) Codeine (C) Diphenoxylate (D) Loperamide (E) Propantheline

D. Loperamide would be a good choice in this patient as it effectively controls diarrhea. Both codeine and diphenoxylate are opioids with abuse potential, especially in patients with abusive histories. Diphenoxylate is available in combination with atropine to reduce the potential for abuse. Anticholinergics such as propantheline prevent cramping but have little effect on diarrhea. Kaolin is good for absorbing toxins from the intestines.

A man brings his 3-year-old son into the office, complaining that his son is having "bad stomach pains" and talks about him "running a fever" and "being thirsty all the time." He remarks that his son has not had a bowel movement lately. Relevant physical exam findings include painless rectal bleeding, dark-red stools, and abdominal distention. Relevant laboratory findings include radiograph showing a remnant of the vitelline duct that was estimated to be about 2 feet from the ileocecal valve and a biopsy showing ectopic gastric and pancreatic mucosal tissue. Which of the following is the most likely diagnosis? (A) Volvulus (B) Intussusception (C) Foreign-body obstruction (D) Meckel diverticulum (E) Biliary atresia

D. Meckel diverticulum. Meckel diverticulum (ileal diverticulum) occurs when a remnant of the vitelline duct persists, thereby forming an outpouching located on the antimesenteric border of the ileum. The outpouching may connect to the umbilicus via a fibrous cord or fistula. A Meckel diverticulum is usually located about 30 cm proximal to the ileocecal valve in infants and varies in length from 2 to 15 cm. Heterotopic gastric or pancreatic mucosa may be present, which leads to ulceration, perforation, or gastrointestinal bleeding, especially if a large number of parietal cells are present. It is associated clinically with symptoms resembling appendicitis and bright-red or dark-red stools (i.e., bloody).

A 70-year-old man has a 2-cm red plaque on his scalp. On microscopy the tumor is comprised of small, round, blue cells with scant cytoplasm and granular chromatin. Immunostaining with cytokeratin 20 shows perinuclear dot-like positivity and synaptophysin is also positive. The diagnosis is (A) Malignant melanoma. (B) Squamous cell carcinoma. (C) Metastatic small cell carcinoma of the lung. (D) Merkel cell carcinoma. (E) Basal cell carcinoma.

D. Merkel cell carcinoma is a neuroendocrine tumor of the skin which most commonly arises in the head and neck of the elderly. It is comprised of small round blue cells that express neuroendocrine markers and show perinuclear staining for cytokeratin 20. Although the morphology and synaptophysin staining could be seen in a small cell carcinoma of the lung, the cytokeratin 20 staining would not be expected.

A new class of pathogen appears, causing pneumonia. It appears to be some sort of budding yeastlike organism with chitin in the cell wall, but the nucleus is not membrane bound and the protein synthetic equipment appears to be procaryotic. Which of the following antimicrobial agents is most likely to be effective? (A) Metronidazole (B) Penicillins (C) Third generation cephalosporins (D) Doxycycline

D. Metronidazole (choice A) is not likely to work, as the agent is causing pneumonia, so it is probably aerobic or a facultative anaerobe. If the cell wall does not include peptidoglycan, b-lactam drugs (choices B and C) are likely to be ineffective. However, since the ribosomes are procaryotic, doxycycline has the best chance as long as it can penetrate the pathogens cell wall and cytoplasmic membrane.

A 30-year-old woman presents with ptosis and severe generalized muscle weakness. Administration of edrophonium (an anticholinesterase) results in a rapid and dramatic recovery of muscle strength. Which of the following is the most likely diagnosis? (A) Alzheimer disease (B) Huntington disease (C) Idiopathic Parkinson disease (D) Myasthenia gravis (E) Wernicke-Korsakoff syndrome

D. Myasthenia gravis is an autoimmune disorder caused by autoantibodies to acetylcholine receptors of the neuromuscular junction. Dramatic improvement in muscle strength with an anticholinesterase agent, such as edrophonium, is a useful diagnostic sign.

What is the first event for a patient who ultimately develops Neisseria meningitidis meningitis? (A) Crossing the blood-brain barrier (B) Meningococcemia (C) Skin lesions (D) Upper respiratory colonization (E) Waterhouse-Friderichsen syndrome (F) Traumatic implantation directly into the brain

D. Neisseria meningitidis is an immunoglobulin A protease producer with a capsule; these virulence factors play a role in the upper respiratory colonization that is followed by invasion of the bloodstream and that precedes meningitis. Skin lesions develop from overproduction of outer membrane, which is excreted without being incorporated, causing endotoxin shock and petechiae that progress to frank purpura. Waterhouse-Friderichsen syndrome is late and occurs when the adrenal glands are involved.

In preparation for a stem cell transplant in a patient with leukemia, the oncologist considers the differential susceptibility of various tissues and cells to high-dose chemotherapy and lethal irradiation. So-called permanent cells are the least vulnerable to injury, but also have the least capacity to regenerate. Which of the following is a permanent cell? (A) Bronchial epithelial cell (B) Gastric mucosal cell (C) Hepatocyte (D) Hippocampal neuron (E) Renal tubular cell

D. Neurons are permanent cells, and the classic teaching has been that neurons do not proliferate during adult life (recent evidence casts some doubt on this concept). Bronchial epithelial cells, gastric mucosal cells, and skin epithelial cells are labile cells. Hepatocytes and renal tubular cells are stable cells. Both labile and stable cells are capable of proliferation and regeneration.

After a football injury, a 16-year-old boy survives a ruptured spleen, hemorrhagic shock, and profound hypotension; however, a period of severe oliguria follows, so the boy undergoes a kidney biopsy. Which one of the following renal tubular changes found represents irreversible cellular injury? (A) Fatty change (B) Formation of cell blebs (C) Formation of myelin figures (D) Nuclear pyknosis (E) Swelling of organelles

D. Nuclear pyknosis, along with karyorrhexis and karyolysis, is a sign of necrosis and is, of course, irreversible. Fatty change, formation of cell blebs or myelin figures, and swelling of the cell or of organelles are all reversible changes.

A 5 year old girl is brought to your office by her mother who reports the child had a flulike disease for the past week and now has developed a rash. Examination of the rash on the face and trunk shows general redness consistent with a slapped cheek. A single-stranded DNA virus that could be responsible is (A) Coxsackie A virus (B) Echovirus (C) JC virus (D) Parvovirus

D. Only one virus family has single-stranded DNA as its genetic material, and that is the parvovirus family. Parvovirus B19 can cause a disease called erythema infectiosum, which produces signs and symptoms consistent with those described.

Corynebacterium diphtheriae is isolated from a patient with pharyngitis. What is the best predictor that the strain is pathogenic? (A) Those producing the blackest colonies on tellurite medium. (B) Those with a plasmid with tox+ genes. (C) Those with chromosomal inv+ genes. (D) Those lysogenized by corynebacteriophage-b

D. Only toxin-producing strains of Corynebacterium diphtheriae cause diphtheria. The genes directing the production of the toxin are located on molecules of corynebacteriophage- b DNA, which may infect and lysogenize C. diphtheriae, causing production of the toxin. Neither plasmids nor the chromosome contains the genes to direct the synthesis of the toxin. Repressor molecules for the tox+ gene are on the chromosome, however. Both toxigenic and nontoxigenic strains of C. diphtheriae will be gray to black on tellurite medium.

A 23-year-old woman consults an obstetrician because she is hoping to become pregnant but is concerned about possible consequences of rubella infection. She received all of her childhood immunizations, but now she has been found to be negative for antibodies to rubella. The obstetrician administers a rubella vaccination and advises her to return for an anti-rubella titer prior to becoming pregnant. Which of the following is true regarding congenital rubella infection? (A) Associated fetal defects are limited to the cardiovascular system. (B) The fetus is most vulnerable during the third trimester of pregnancy. (C) The majority of cases of congenital heart disease are caused by rubella or other intrauterine infections. (D) Patent ductus arteriosus and septal defects are the most frequent congenital cardiac abnormalities associated with rubella infection. (E) A predominant IgG antibody response indicates recent primary infection.

D. Patent ductus arteriosus and septal defects are the most frequent congenital cardiac abnormalities associated with congenital rubella infection; however, defects are not limited to the cardiovascular system, and congenital infection can also lead to deafness and mental retardation. In addition, congenital rubella infection, along with other congenital intrauterine infections, accounts for only a small proportion of cases of congenital heart disease, the majority being of unknown cause. The most severe consequences occur as a result of infection during the first trimester of pregnancy. As with all infections, an IgM antibody response indicates recent primary infection.

N-acetylmuramic acid is located in (A) Lipopolysaccharide (B) Lipoprotein (C) Outer membrane (D) Peptidoglycan (E) Teichoic acid

D. Peptidoglycan (mucopeptide and murein) is a complex cell-wall polymer containing N-acetylglucosamine and N-acetylmuramic acid and associated peptides.

Which of the following bacterial compounds is important in colonization? (A) Diaminopimelic acid (B) Calcium dipicolinate (C) ''O'' antigens (D) Pilin (E) Porins

D. Pilin is the main protein component of the pili involved in bacterial adherence.

A 50-year-old chronic alcoholic with jaundice and ascites secondary to known cirrhosis becomes disoriented and confused. Asterixis (flapping tremor) can be demonstrated. Which of the following features of his disease is most closely related to the change in mental status? (A) Hypoalbuminemia (B) Increased hepatic lymph formation (C) Increased portal venous pressure (D) Portal-systemic venous shunting (E) Renal retention of sodium and water

D. Portal-systemic venous shunting leads to encephalopathy in end-stage cirrhosis. It also leads to esophageal varices, rectal hemorrhoids, and distention of periumbilical venous collaterals. Hypoalbuminemia, increased hepatic lymph formation, increased portal venous pressure, and renal retention of sodium and water all contribute to the development of ascites, but have little to do with encephalopathy.

A 32-year-old woman being treated for an acute exacerbation of lupus erythematosus complains of pain on eating. What is the most likely causative agent? (A) Oxytocin (B) Androlone (C) Vasopressin (D) Prednisone (E) Clomiphene

D. Prednisone, a steroid commonly used to treat exacerbations of lupus erythematosus, can cause peptic ulcer disease due to the inhibition of the prostaglandins that normally protect the mucosa.

An 18-year-old African-American army recruit with a history of G-6-PDH deficiency is to be stationed in Somalia. During his tour of duty he develops a cyclic fever, malaise, and weakness. A thin blood smear shows malarial organisms within red blood cells. Which antimalarial is likely to exacerbate the hemolysis, given his enzyme deficiency? (A) Chloroquine (B) Pyrimethamine (C) Doxycycline (D) Primaquine (E) Sulfasalazine

D. Primaquine is associated with intravascular hemolysis or methemoglobinuria in G-6-PDH deficiency patients, as it causes oxidative damage to hemoglobin. Chloroquine and pyrimethamine do not cause hemolysis, although they are often used with sulfa drugs, which can cause hemolysis in such patients. Chloroquine rarely causes hemolysis, and doxycycline is not known to cause problems in G-6-PDH deficiency.

A 28-year-old man was evaluated for progressive weakness, weight loss, and anorexia. He was found to be hypotensive, and had generalized hyperpigmentation involving exposed surfaces of the skin, lips, and buccal mucosa. Which of the following laboratory findings is expected in this patient? (A) Increased serum sodium (B) Decreased serum potassium (C) Increased serum glucose (D) Decreased plasma cortisol not corrected by administration of adrenocorticotropic hormone (ACTH) (E) Increased urinary 17-ketosteroids

D. Primary adrenocortical deficiency (Addison disease), as distinguished from adrenal cortical insufficiency secondary to pituitary hypofunction, is indicated by the presence of pigmentation. Also, decreased plasma cortisol in Addison disease is unresponsive to ACTH administration. Progressive weakness and hypotension are strongly suggestive of adrenocortical deficiency regardless of cause, and other expected findings include decreased serum sodium, increased serum potassium, decreased serum glucose, and decreased urinary 17-ketosteroids.

A 60-year-old woman presents with fever, chills, dysuria, hematuria, and pain. She reports a long-standing history of use of phenacetin, acetaminophen, nonsteroidal anti-inflammatory drugs (NSAIDs), and aspirin. Abuse of these medications is associated with necrosis of which of the following sites? (A) Basal ganglia (B) Hilar lymph nodes (C) Myocardium (D) Renal papillae (E) Splenic arterioles

D. Renal papillary necrosis is a well-known complication of chronic analgesic nephritis, which is caused by long-term abuse of phenacetin or its metabolite, acetaminophen, most often in combination with aspirin or a NSAID. Another major cause of renal papillary necrosis is diabetes mellitus. Phenacetin abuse is also associated with a markedly increased incidence of transitional cell carcinoma of the renal pelvis.

A patient with sickle cell anemia is most likely to have repeated septicemias and possible osteomyelitis with which agent? (A) Candida albicans (B) Nontypeable Haemophilus influenzae (C) Mycobacterium avium-intracellulare (D) Salmonella enteritidis (E) Staphylococcus aureus

D. Sickle cell anemia patients have problems with septicemias with encapsulated organisms, such as pneumococcus and Klebsiella. Of those listed, only Salmonella enteritidis has a prominent capsule and it is noted for causing repeated infections in sickle cell carriers. Staphylococcus aureus is only rarely encapsulated.

A 65-year-old man presents with recurrent fever and painless cervical and supraclavicular lymphadenopathy. Biopsy and further studies reveal that the patient has Hodgkin disease at a stage that is usually associated with a very poor prognosis. Staging is based on which of the following? (A) Cell of origin (B) Degree of anaplasia (C) Degree of differentiation of tumor cells (D) Distribution and extent of disease (E) Number of mitotic figures

D. Staging is based on clinical evaluation of the distribution and extent of the disease process and is contrasted with grading, which is based on histopathologic evaluation of a malignant neoplasm.

A 92-year-old male presents with a cough of several months duration and coughing up brownish ''stuff.'' He is 50600; tall and his weight is down from 150 to 140 lbs having lost 10 lbs. in the past 2 months without trying. His tuberculin skin test has been greater than 10 mm the three times you have tested him in the 25 years he has been your patient. The chest radiograph is consistent with reactivational TB. You order Mycobacterium tuberculosis testing and susceptibilities and start him on standard therapy. Luminescent real time PCR confirms M. tuberculosis. What has been the standard medium used to grow M. tuberculosis and what is the standard clue to TB in examination cases? (A) Blood agar (B) Buffered charcoal yeast extract agar (C) Chocolate agar (D) Lowenstein-Jensen medium (E) Tellurite-containing medium such as Regan-Lowe

D. Standard cultures may still be set up on Lowenstein-Jensen agar. Palmitic acid-containing broths (some with anti-TB drugs) have replaced solid agars, and growth detection systems now allow more rapid diagnosis. (The mycobacteria have limited numbers of ribosomal genes slowing growth.) Choice B (BCYE) is a clue to legionellosis; chocolate agar is required to grow either Neisseria spp. or Haemophilus influenzae from sterile sites like CSF. Thayer-Martin is a chocolate agar with antibiotics used to culture Neisseria spp. from any mucosal surface preventing overgrowth of the normal flora. Tellurite (choice E) is a clue to C. diphtheriae.

A 60-year-old man presents with chronic headache and personality changes since he had been in a car accident a month ago, during which he had suffered a whiplash neck injury. A CT scan of the head demonstrates a hyperdense crescentic region along the inner skull. Which of the following is the most likely diagnosis? (A) Bruising of the brain substance of the cerebral hemisphere (B) Enlargement of the cerebral ventricles (C) Epidural hematoma (D) Subdural hematoma (E) Subarachnoid hemorrhage

D. Subdural hematoma can be acute (e.g., due to moderate head trauma) or chronic (e.g., due to whiplash). Subdural hematomas are caused by venous bleeding, most often from laceration of the bridging veins, which join the cerebral vessels to the venous sinuses within the dura. The classic CT scan finding is formation of a "crescent" along the inner skull, which is due to subdural venous bleeding.

A 14-year-old girl has symptoms that include a low-grade fever, chills, headache, muscle aches, and malaise. A physical examination indicates an enlarged spleen. A Monospot test is negative. The laboratory finding that would confirm a diagnosis would be (A) 10 times or greater macrophages in the blood (B) Heterophile antibodies (C) Koilocytotic cells in pharyngeal scrapings (D) ''Owl's eye'' cells in urine

D. Symptoms are consistent with a mononucleosis disease that is usually caused by EBV or CMV. The lack of a positive Monospot test for heterophile antibodies and presence of ''owl's eye'' cells would indicate CMV.

A 54-year-old man presents with multiple recurrent peptic ulcers of the duodenum and the jejunum. Gastric acid secretion is refractory to proton pump inhibitors. These findings suggest an underlying (A) adenocarcinoma of the pancreas. (B) adenoma of the adrenal medulla. (C) carcinoid of the jejunum. (D) islet cell tumor of the pancreas. (E) pheochromocytoma.

D. The Zollinger-Ellison syndrome, characterized by markedly increased gastric acid production and intractable peptic ulcer, is caused by hypersecretion of a gastrin-producing islet cell tumor (gastrinoma).

A 70-year-old man is found to have a papillomatous neoplasm of the bladder. Even though the lesion is extremely well- differentiated, the pathologist makes a diagnosis of transitional cell carcinoma grade I (urothelial neoplasm of low malignant potential). The distinction of "malignant" from "benign" in this instance was most likely based on which of the following known general characteristics of this type of lesion? (A) Appearance of oncofetal antigens (B) Chromosomal aneuploidy (C) Loss of contact inhibition in tissue culture (D) Clinical behavior (E) Ultrastructural alterations

D. The clinical behavior of neoplasms is the underlying basis for all other indicators that distinguish malignant from benign lesions. For example, some extremely well-differentiated, otherwise benign-appearing lesions are known to metastasize and, thus, are classified as malignant.

While being investigated for long-standing hypertension, a 55-year-old woman is found to have the following serum laboratory test values: normal creatinine, total protein, albumin, and globulin; increased calcium and alkaline phosphatase; and decreased phosphorus. These findings suggest the presence of (A) carcinoma metastatic to bone. (B) excessive dietary calcium intake. (C) multiple myeloma. (D) parathyroid adenoma. (E) sarcoidosis.

D. The combination of increased serum calcium and alkaline phosphatase along with decreased serum phosphorus is most consistent with primary hyperparathyroidism. The most frequent cause of this endocrine abnormality is a parathyroid adenoma. Decreased phosphorus would not be an expected finding in metastatic carcinoma. The normal serum proteins mitigate against multiple myeloma and sarcoidosis. Additionally, the alkaline phosphatase is usually normal in multiple myeloma. Hypercalcemia from increased intake of calcium (as in the milk-alkali syndrome) is usually unaccompanied by significant changes in phosphorus or alkaline phosphatase.

A compromised patient is admitted in respiratory distress. She had signs of focal central nervous system (CNS) lesions early in the day and is now in a comatose state. The CNS and pulmonary biopsies show dichotomouslybranching (at an acute angle), septate hyphae. What is the most likely underlying condition? (A) CD4+ cell count less than 200 (B) Ketoacidotic diabetes (C) Multiple myeloma (D) Severe neutropenia (E) Sickle cell disease

D. The disease is invasive aspergillosis, which is found primarily in patients with neutrophil counts less than 500/mm3.

Which of the following phrases best describes a prophage? (A) Phage that lacks receptors (B) Phage attached to the cell wall that has released its DNA (C) Newly assembled intracellular phage particle (D) Intracellular temperate phage DNA

D. The integrated intracellular form of the DNA of a temperate phage is called a prophage.

A 6 month old infant has had watery diarrhea for 6 days; he vomited a couple of times. The stools have no blood and no pus. He is dehydrated. He has not been outside of Cincinnati. Two other toddlers who visited for a day also are sick. What is most likely causing this? (A) Clostridium botulinum toxin (B) Giardia lamblia (C) Norwalk agent (D) Rotavirus (E) Salmonella enteritidis (F) Staphylococcus aureus enterotoxin

D. The lack of flaccid paralysis leaves out botulinum toxin, and staphylococcal toxin is also unlikely as it is a short, self-resolving illness. The most likely is rotavirus due to his age and the length of the disease.

An asymptomatic 35-year-old male Catholic missionary has returned to the United States following 12 years in rural Mexico. His routine physical examination shows elevated transaminases. His medical history in Mexico was unremarkable except for an auto accident requiring a blood transfusion of 3 pints of blood. What definitive lab test should be performed to determine what effective treatment should be prescribed? (A) Blood culture on MacConkey media (B) Differential cytology of the blood (C) Electron microscopy for Dane particles (D) Molecular genotyping of circulating virus (E) Serology for levels of IgG HEV

D. The man was infected with HCV during his blood transfusion. Molecular genotyping of circulating virions is necessary since effective treatment with a-interferon and ribavirin occurs in only two genotypes (2 and 3) of the six genotypes.

After a trip to Peru to adopt a 6 month old baby, a 32-year-old woman and her new baby both develop profuse, watery diarrhea with flecks of mucus. Both are hospitalized because of the severity and rapidity of the dehydration, but neither one is febrile. What is the most likely causative agent? (A) Campylobacter jejuni (B) Escherichia coli O157 (C) Salmonella typhi (D) Vibrio cholerae (E) Shigella dysenteriae

D. The mention of Peru should raise suspicion of cholera. (Rotavirus would have to be ruled out for the baby, but it is not one of the choices.) The loss of fluids and electrolytes is rapid and most severe with Vibrio cholerae. The lack of fever and lack of pus and blood in the stools suggests that the causative agent is not Campylobacter, Salmonella, or Shigella. The lack of blood also decreases the chance of Escherichia coli O157. If untreated, cholera may lead to dehydration, hemoconcentration, and hypovolemic shock.

Cytologic findings consistent with carcinoma of the endometrium are found during diagnostic evaluation of a 65-year-old woman with uterine bleeding. Which of the following is associated with this condition? (A) Arsenic exposure (B) BRCA mutations (C) Endometriosis (D) Prolonged exposure to estrogens (E) Sexual promiscuity

D. The most important factor in the pathogenesis of endometrial carcinoma appears to be prolonged estrogen stimulation, such as that associated with estrogen therapy or estrogen-secreting tumors. Obesity and conditions associated with it, such as diabetes mellitus or hypertension, may contribute to hyperestrinism because estrone can be synthesized in peripheral fat cells. Arsenic exposure is associated with carcinomas of the lung and skin and with hepatic angiosarcoma. BRCA mutations are associated with breast and ovarian cancer. Endometriosis is not a neoplasm, although interestingly its incidence is increased in patients with certain gynecologic malignancies, most notably clear cell carcinomas of the ovary and endometrium. Sexual promiscuity is a risk factor for cervical cancer, not endometrial cancer.

Parents bring in a 5 month old who suddenly has lost developmental ground and is no longer able to hold his head up or sit up. In taking the health history, they did note constipation a couple of days before. The EEG confirms a pattern consistent with botulism. After assuring them that with monitoring and support the child will survive, the parents become terribly concerned that the loss of development might be permanent. What can you tell them about their child's prognosis? (A) Elevated cerebrospinal fluid pressure occurs in over half of the children, resulting in some permanent neurologic defects (B) Almost all of the children have some permanent neurologic defects (C) There is some permanent muscle weakness following recovery that, like polio, may worsen late in life (D) Neurological recovery, although slow, is expected to be complete in all children

D. The neurotoxin involved in infant botulism, unlike bacterial meningitis, does not cause elevated cerebrospinal fluid pressure, so recovery should be complete. The use of the recombinant human antitoxin shortens hospitalization and recovery time. However, it still requires regrowth of nerve endings.

A 6 month old child has had watery diarrhea for 6 days. The stools have no blood and no pus. The causative agent has doublestranded RNA as genetic material. Which of the following is the most likely causative agent? (A) Bacillus cereus (B) Giardia lamblia (C) Norwalk agent (D) Rotavirus (E) Salmonella enteritidis (F) Staphylococcus aureus

D. The patient's age, symptoms, and nucleic acid indicate that rotavirus is the primary suspect.

IgA is largely responsible for protection of the respiratory and intestinal mucosal tract. Effective protection is dependent on the action of secretory piece which (A) Facilitates the passage of IgA out of the plasma cell (B) Facilitates the formation of the IgA dimer (C) Is released by IgA causing an inflammatory response (D) Is a poly Ig transport receptor on mucosal epithelial cells

D. The secretory piece is part of a polyIg transport receptor facilitating transfer of IgA to mucosal epithelial cells.

In January a 4 month old girl is seen by her family physician. Her mother noted that she has been coughing and has had a slight fever for 3 days and recently developed difficulty breathing that is characterized by wheezing. She has a runny nose composed of clear fluid with some cells present. X-rays show her lungs are clear of infection. Examination of the nasal discharge is likely to show what entity to confirm diagnosis? (A) Eosinophilic cytoplasmic inclusions in cells (B) Gram-positive cocci (C) Gram-negative coccobacilli (D) Multinucleated giant cells (E) Secretory IgA heterophile antibody

D. The signs and symptoms of the little girl's infection are consistent with a bronchiolitis. The most common cause of bronchiolitis in infants is respiratory syncytial virus.

A 7 year old develops watery diarrhea, which becomes bloody, frightening him. He is afebrile but becoming very ill. Macroscopic examination of the feces shows no areas of pus; microscopically there was no excess of PMNs over that expected from peripheral blood. If his disease is a result of a trip to a Wisconsin dairy farm and petting cattle, what is causing the bleeding? (A) A toxin binding to Gs, shutting off protein syntheses (B) Invasion of Mcells with shallow ulceration resulting from actin C (C) Polymerization, ''jetting'' the bacterium laterally (D) A toxin nicking the eukaryotic ribosome, shutting down protein synthesis and damaging the mucosa (E) Invasion of intestinal cells with transit to the bloodstream and reinvasion

D. The symptoms are only characteristic of enterohemorrhagic E. coli. The toxin kills intestinal cells by shutting off protein synthesis, and it kills cells that cause erosion of the intestinal wall. HUS may also occur.

A 22-year-old female student complains of excessive fatigue for the past 2 months. Polyarthralgia and vasculitis is noted on examination. Lab tests reveal multiple autoantibodies including antidouble stranded DNA. The most logical diagnosis is (A) Hashimoto's thyroiditis (B) Multiple myeloma (C) Sj€ogren's syndrome (D) Lupus erythematosus

D. These signs and symptoms are characteristic of lupus erythematosus, with antibodies to double-stranded DNA being selective for SLE. Although autoantibodies are present in Hashimoto's thyroiditis, anti-dsDNA is absent.

Which of the following would be useful in treating nocturnal enuresis? (A) Mannitol (B) Indomethacin (C) Furosemide (D) Vasopressin (E) Probenecid

D. Vasopressin can be tried in cases of recalcitrant nocturnal enuresis. Mannitol is most commonly used in the management of cerebral edema (choice A). Indomethacin can occasionally be used as an antidiuretic agent in diabetic patients (choice B). Furosemide is used in congestive heart failure (choice C). Probenecid is used in the treatment of gout (choice E).

A father brings his 4-year-old daughter into the clinic. He says he noticed "a lump on her lower right side" and that "it has gotten bigger over time." Relevant physical exam findings include a large, palpable mass on the right flank and no evidence of a urinary tract infection (UTI). Relevant laboratory findings include normal catecholamine levels and normal androgen levels, and genetic testing reveals a deletion of a tumor suppression gene on chromosome 11. Which of the following is the most likely diagnosis? (A) Neuroblastoma (B) Pheochromocytoma (C) Congenital adrenal hyperplasia (D) Wilms tumor (E) Childhood polycystic kidney disease

D. Wilms tumor (WT). WT is the most common renal malignancy of childhood. WT presents as a large, solitary, well-circumscribed mass that on cut section is soft, homogeneous, and tangray in color. WT is interesting histologically, in that this tumor tends to recapitulate different stages of embryological formation of the kidney, so that three classic histological areas are described: a stromal area; a blastemal area of tightly packed embryonic cells; and a tubular area. Neuroblastoma is ruled out because there was no mention of an increase in urine vanillylmandelic acid (VMA) and metanephrine levels.

A 36-year-old female drug abuser has an abscess on her jaw where she lost a tooth from being struck in the face 2 weeks ago. She did not get medical help. You lance the lesion, express some lumpy material with difficulty, and send a swab of the organism to the lab for Gram stain, culture, and susceptibilities. The predominant organism in the Gram stain is a filamentous bacterium that is Gram positive, yet nothing grows in the lab. What is the most likely explanation for why it did not grow? (A) It is an obligate aerobe (B) It is microaerophilic (C) It is a facultative anaerobe (D) It is an obligate anaerobe (E) The lab did not grow it on the correct medium

D. Your organism is most likely an anaerobe that did not tolerate transport aerobically. Unless you ask for specimens to be collected anaerobically or at least ask the lab to culture an abscess, it will not be cultured anaerobically especially when you send it in on a swab. The clinical case suggests an organism from the gingival crevices and not Staphylococcus aureus as the cause. Needle biopsy with the specimen sent in the syringe and anaerobic culture should have been done instead. It is most likely Actinomyces. It may take debridement by an oral surgeon and antibiotics for several weeks.

Which of the following statements characterize g2j2 antibody? (A) It contains a J chain (B) It contains a secretory piece (C) It is the initial antibody synthesized after antigen (D) It contains a hypervariable region (E) Each of the above statements are characteristic of this antibody

D. g2j2 antibody describes an IgG antibody. It does not require a J chain, as do IgM and IgA, nor does it need a secretory piece, as does IgA. IgM is the initial antibody synthesized after antigen stimulation. All antibodies possess a hypervariable region.

A patient presents with paranasal swelling, hemorrhagic exudates in the eyes and nares, and mental lethargy. Nonseptate hyphae are found invading the tissues. Rhinocerebral Mucor infection (zygomycosis) is diagnosed. What is the most likely underlying condition? (A) C5-C8 deficiencies (B) Epstein-Barr virus infection (C) Hepatitis A infection (D) Hepatitis B infection (E) Ketoacidotic diabetes (F) Severe neutropenia

E . Ketoacidotic diabetes is a major predisposing condition for zygomycosis, although lymphoma and leukemia also predispose the patient to zygomycosis.

A 19-year-old college freshman has a sore throat, sore and enlarging cervical lymph nodes, and a fever and is greatly fatigued. A diagnosis of infectious mononucleosis is made. Which of the following factors is present? (A) Delta hemagglutinin (B) E1A protein (C) Large T antigen (D) TAT protein (E) VCA protein (F) Matrix protein

E The VCA protein, or viral capsid antigen, is the main component of the Epstein-Barr virus (EBV) capsid. EBV is the causative agent of infectious mononucleosis.

A key difference in the mechanism of excitation-contraction coupling between the muscle of the pharynx and the muscle of the wall of the small intestine is that (A) slow waves are present in the pharynx, but not in the small intestine (B) adenosine triphosphate (ATP) is used for contraction in the pharynx, but not in the small intestine (C) intracellular [Ca2+] is increased after excitation in the pharynx, but not in the small intestine (D) action potentials depolarize the muscle of the small intestine, but not of the pharynx (E) Ca2+ binds to troponin C in the pharynx, but not in the small intestine, to initiate contraction

E [Chapter 1, VI B; VII B; Table 1.3]. The pharynx is skeletal muscle, and the small intestine is unitary smooth muscle. The difference between smooth and skeletal muscle is the mechanism by which Ca2+ initiates contraction. In smooth muscle, Ca2+ binds to calmodulin, and in skeletal muscle, Ca2+ binds to troponin C. Both types of muscle are excited to contract by action potentials. Slow waves are present in smooth muscle but not skeletal muscle. Both smooth and skeletal muscle require an increase in intracellular [Ca2+] as the important linkage between excitation (the action potential) and contraction, and both consume adenosine triphosphate (ATP) during contraction.

A receptor potential in the pacinian corpuscle (A) is all-or-none (B) has a stereotypical size and shape (C) is the action potential of this sensory receptor (D) if hyperpolarizing, increases the likelihood of action potential occurrence (E) if depolarizing, brings the membrane potential closer to threshold

E [Chapter 2, II A 4; Figure 2.2]. Receptor potentials in sensory receptors (such as the pacinian corpuscle) are not action potentials and therefore do not have the stereotypical size and shape or the all-or-none feature of the action potential. Instead, they are graded potentials that vary in size depending on the stimulus intensity. A hyperpolarizing receptor potential would take the membrane potential away from threshold and decrease the likelihood of action potential occurrence. A depolarizing receptor potential would bring the membrane potential toward threshold and increase the likelihood of action potential occurrence.

At the beginning of which phase of the cardiac cycle does the second heart sound occur? (A) Atrial systole (B) Isovolumetric ventricular contraction (C) Rapid ventricular ejection (D) Reduced ventricular ejection (E) Isovolumetric ventricular relaxation (F) Rapid ventricular filling (G) Reduced ventricular filling

E [Chapter 3, V; Figure 3.15]. Closure of the aortic and pulmonic valves creates the second heart sound. The closure of these valves corresponds to the end of ventricular ejection and the beginning of ventricular relaxation.

Which of the following conditions causes hypoventilation? (A) Strenuous exercise (B) Ascent to high altitude (C) Anemia (D) Diabetic ketoacidosis (E) Chronic obstructive pulmonary disease (COPD)

E [Chapter 4, IX A, B; Chapter 5 IX D]. Chronic obstructive pulmonary disease (COPD) causes hypoventilation. Strenuous exercise increases the ventilation rate to provide additional oxygen to the exercising muscle. Ascent to high altitude and anemia cause hypoxemia, which subsequently causes hyperventilation by stimulating peripheral chemoreceptors. The respiratory compensation for diabetic ketoacidosis is hyperventilation.

Arterial [HCO3 -] of 18 mEq/L, Pco2 of 34 mm Hg, and increased urinary HCO3 - excretion would be observed in a (A) patient with chronic diabetic ketoacidosis (B) patient with chronic renal failure (C) patient with chronic emphysema and bronchitis (D) patient who hyperventilates on a commuter flight (E) patient who is taking a carbonic anhydrase inhibitor for glaucoma (F) patient with a pyloric obstruction who vomits for 5 days (G) healthy person

E [Chapter 5, IX D 1; Table 5.9]. The blood values are consistent with metabolic acidosis (calculate pH = 7.34). Treatment with a carbonic anhydrase inhibitor causes metabolic acidosis because it increases HCO3 - excretion.

A 54-year-old man with a lung tumor has high circulating levels of antidiuretic hormone (ADH), a serum osmolarity of 260 mOsm/L, and a negative free-water clearance CH O 2 ( ). Which diagnosis is correct? (A) Primary polydipsia (B) Central diabetes insipidus (C) Nephrogenic diabetes insipidus (D) Water deprivation (E) Syndrome of inappropriate antidiuretic hormone (SIADH)

E [Chapter 5, VII D 4]. A negative value for free-water clearance CH O 2 ( ) means that "free water" (generated in the diluting segments of the thick ascending limb and early distal tubule) is reabsorbed by the collecting ducts. A negative CH O 2 is consistent with high circulating levels of antidiuretic hormone (ADH). Because ADH levels are high at a time when the serum is very dilute, ADH has been secreted "inappropriately" by the lung tumor.

Which of the following hormones is converted to its active form in target tissues by the action of 5á-reductase? (A) Adrenocorticotropic hormone (ACTH) (B) Aldosterone (C) Estradiol (D) Prolactin (E) Testosterone

E [Chapter 7, IX A; Figure 7.16]. Testosterone is converted to its active form, dihydrotestosterone, in some target tissues by the action of 5á-reductase.

Inhibition of which enzyme in the steroid hormone synthetic pathway reduces the size of the prostate? (A) Aldosterone synthase (B) Aromatase (C) Cholesterol desmolase (D) 17,20-Lyase (E) 5á-Reductase

E [Chapter 7, IX A]. 5á-Reductase catalyzes the conversion of testosterone to dihydrotestosterone. Dihydrotestosterone is the active androgen in several male accessory sex tissues (e.g., prostate).

Levels of which of the following hormones are high during the first trimester of pregnancy and decline during the second and third trimesters? (A) Adrenocorticotropic hormone (ACTH) (B) Estradiol (C) Follicle-stimulating hormone (FSH) (D) Gonadotropin-releasing hormone (GnRH) (E) Human chorionic gonadotropin (HCG) (F) Oxytocin (G) Prolactin (H) Testosterone

E [Chapter 7, X F 2; Figure 7.20]. During the first trimester of pregnancy, the placenta produces human chorionic gonadotropin (HCG), which stimulates estrogen and progesterone production by the corpus luteum. Peak levels of HCG occur at about the 9th gestational week and then decline. At the time of the decline in HCG, the placenta assumes the responsibility for steroidogenesis for the remainder of the pregnancy.

A mother is holding her newborn baby in the hospital bed just a few hours after giving birth. The mother becomes alarmed when her baby begins to have a difficult time breathing and rings for help. You arrive at the bedside and observe that the baby is in severe respiratory distress. A quick physical exam reveals that the baby has a barrel-shaped chest, ascaphoid-shaped abdomen, and absence of breath sounds on the left side, and the heartbeat is displaced to the right side. A chest radiograph reveals air/fluid containing bowel in the left-side hemithorax, no visible aerated lung on the left side, contralateral displacement of the heart and other mediastinal structures, compression of the contralateral lung, and reduced size of the abdomen. Which of the following is the most likely diagnosis? (A) Esophageal hiatal hernia (B) Hypertrophic pyloric stenosis (C) Tracheoesophageal fistula (D) Respiratory distress syndrome (E) Congenital diaphragmatic hernia

E. A congenital diaphragmatic hernia is a herniation of abdominal contents into the pleural cavity caused by a failure of the pleuroperitoneal membrane to develop or fuse with other components of the diaphragm. Affected neonates usually present in the first few hours of life with respiratory distress that may be mild or so severe as to be incompatible with life.

An autoimmune disease characterized by a triad of thrombocytopenia, eczema, and recurrent infections is (A) Myasthenia gravis (B) Severe combined immunodeficiency disease (SCID) (C) Pernicious anemia (D) Congenital agammaglobulinemia (E) Wiskott-Aldrich syndrome

E. A primary defect in the short arm of the X chromosome results in thrombocytopenia, eczema, and recurrent infections. Depressed CMI and serum Ig levels along with a poor response to capsular polysaccharide antigens complete the syndrome.

A 39-year-old man comes to your office complaining of "heartburn after trying to eat" and not being able to swallow anything. He states, "I have tried everything from water to steaks; it doesn't matter what I eat, I always have trouble swallowing it down." Relevant physical exam findings include dysphagia and normal thyroid on palpation. Relevant laboratory findings include barium swallow radiograph showing a dilated esophagus with an area of distal stenosis (almost looks like a "bird's beak") and normal T4 levels. Which of the following is the most likely diagnosis? (A) Esophageal atresia (B) Thyroid tumor (C) Esophageal stenosis (D) Reflux esophagitis (E) Achalasia

E. Achalasia. Achalasia occurs due to the loss of ganglion cells in the myenteric (Auerbach) plexus and is characterized by the failure to relax the lower esophageal sphincter, which will cause progressive dysphagia and difficulty in swallowing. The "bird's beak" appearance on the radiograph is a telltale sign of achalasia. Another telltale sign is that patients have a dysphagia involving both solids and liquids. The physical and lab findings exclude both thyroid disease and masses. Although reflux esophagitis would present with heartburn, it is limited to dysphagia of solids only, not solids and liquid.

Soon after drug administration, the patient in the above question begins making odd faces with spastic movements of his neck. Which of the following should be administered to treat these dystonic reactions? (A) Fluphenazine (B) Bromocriptine (C) Dantrolene (D) Prolactin (E) Benztropine

E. Acute dystonias are a complication of antipsychotics that work primarily through dopamine D2 receptors and therefore have a high incidence of extrapyramidal effects. Haloperidol and agents such as fluphenazine are the most likely offenders. Such reactions are best managed with an anticholinergic agent such as benztropine. Another complication of haloperidol is the neuroleptic malignant syndrome, which is treated with a dopamine agonist receptor and dantrolene. Hyperprolactinemia with galactorrhea is common with agents that block dopamine's actions, as dopamine normally represses prolactin release.

A Peace Corps volunteer recently returned from rural Africa develops symptoms of liver damage and a blocked bile duct after general anesthesia for a knee replacement. What is the nature of the most likely causative agent? (A) Cestode (B) Dimorphic fungus (C) Filamentous fungus (D) Fluke (E) Nematode (F) Protozoa

E. Adult Ascaris lumbricoides is a roundworm that maintains its position in the gastrointestinal tract by continual movement ''upstream,'' not by attaching. It is noted for migration into the bile duct, gallbladder, and liver, producing severe tissue damage. This process is often exacerbated by fever, antibiotics, and anesthetics.

Which of the following drugs inhibits xanthine oxidase? (A) Colchicine (B) Indomethacin (C) Probenecid (D) Clofibrate (E) Allopurinol

E. Allopurinol is a xanthine oxidase inhibitor and is most commonly used in the treatment of gout. It is not used for acute attacks, rather for the prevention of recurrent episodes. Colchicine (choice A) may be used for an acute episode, as well as in long-term therapy; however, it has a high incidence of side effects. Indomethacin (choice B) is useful for symptomatic treatment of gout. Probenecid (choice C) is also useful for prophylaxis of gout; however, it is not a xanthine oxidase inhibitor. This agent increases the secretion of uric acid. Clofibrate (choice D) is used in the treatment of hypercholesterolemia.

A 36-year-old woman experiences a severe headache. Investigation including computed tomographic angiography (CTA) reveals a saccular outpouching of the anterior cerebral artery. Which of the following is a major characteristic of this type of vascular change? (A) Occurs anywhere within branches of internal carotid artery (B) Infrequent site of subarachnoid hemorrhage (C) Middle meningeal artery most frequent location (D) Usually a complication of severe atherosclerosis (E) Often associated with polycystic kidney

E. An increased incidence of berry aneurysm is associated with adult polycystic kidney. Berry aneurysm occurs at sites of discontinuity of the arterial media, most frequently at bifurcations of vessels of the circle of Willis. The most common locations are the junction of the anterior cerebral and anterior communicating arteries, the bifurcation of the middle cerebral artery, the junction of the internal carotid and posterior communicating arteries, and the junction of the basilar and posterior cerebral arteries. Berry aneurysm is the most frequent cause of subarachnoid hemorrhage, and there is no association with atherosclerosis.

Your resident asks you what the mechanism of action of tPA is. What is your answer? (A) It inhibits platelet aggregation (B) It increases antithrombin activity (C) It impairs inhibits fibrin polymerization (D) It blocks GPIIa/IIIb (E) It activates plasminogen bound to fibrin

E. As the name suggests, tPA activates plasminogen bound to plasmin, thereby acting as a thrombolytic. Choice A refers to clopidogrel and ticlopidine. Choice B refers to heparin and its analogues. Choice C refers to dextran. Finally, choice D refers to abciximab.

A 57-year-old man with a strong family history of Parkinson's disease sees a neurologist for an evaluation. On examination, the neurologist notes a slight pill-rolling tremor and subtle gait abnormalities. He begins the treatment with levodopa, along with the addition of carbidopa. How does carbidopa work in this setting? (A) Restores dopamine levels in the substantia nigra (B) Inhibits monoamine oxidase (MAO) (C) Inhibits catechol-O-methyltransferase (COMT) (D) Functions as a dopamine agonist (E) Inhibits the metabolism of levodopa outside the central nervous system (CNS)

E. Carbidopa, unlike levodopa, does not penetrate the central nervous system (CNS); it does inhibit levodopa's metabolism in the gi tract, allowing lower doses of levodopa and decreased side effects. Levodopa is a precursor to dopamine and can help restore levels of dopamine in the substantia nigra. Monoamine oxidase inhibitors should be used with caution along with levodopa, as this can lead to a hypertensive crisis. Bromocriptine is a dopamine agonist used in the treatment of Parkinson's disease. Catechol-O-methyltransferase (COMT) inhibitors are yet another class of agents used in the treatment of Parkinson's disease.

A 37-year-old alcoholic is recovering in the hospital from pneumonia due to Haemophilus influenzae. His treatment included the use of intravenous antibiotics. The nurse calls you to evaluate the patient as he returned from a visit from his "buddy" outside. The nurse tells you he smells like alcohol and he is flush, warm, and uncomfortable. You suspect a disulfiram-like reaction. Which antibiotic was he likely to be treated with? (A) Vancomycin (B) Bacitracin (C) Chloramphenicol (D) Isoniazid (E) Cefamandole

E. Cefamandole, a cephalosporin, is known to precipitate a disulfiram-like reaction. Bacitracin is not used intravenously, only topically. Chloramphenicol is associated with bone marrow suppression. Vancomycin can be associated with flush on infusion. Isoniazid is an antituberculoid antibiotic.

A Brazilian who left poverty and moved to Los Angeles 10 years before died suddenly of heart failure. Autopsy showed cardiomyopathy due to Chaga's disease. Which organism transferred the trypanosomes? (A) Lice—genus Pediculus (B) Mites (C) Mosquitoes—genus Aedes (D) Mosquitoes—genus Anopheles (E) Reduviid bugs (F) Sandflies

E. Chagas disease (South American trypanosomiasis) is transmitted by reduviid bugs that defecate as they bite. The trypanosome is actually in the feces, which is scratched into the bite site. Reduviid bugs are also called kissing, assassin, or cone-nosed bugs.

A 30-year-old patient is undergoing chemotherapy for Hodgkin's disease and develops a fever, prompting an admission by his oncologist. He is found to have a severely decreased white blood cell count, and therapy is started with several antibiotics for febrile neutropenia. Assuming his regimen contains imipenem, which of the following must also be administered? (A) Probenecid (B) Clavulanic acid (C) Sulbactam (D) Cycloserine (E) Cilastatin

E. Cilastatin must be given with imipenem. It is an inhibitor of renal dehydropeptidase, which normally would degrade imipenem. Probenecid increases penicillin concentrations by blocking their excretion by the kidney. Both clavulanic acid and sulbactam are penicillinase inhibitors used to increase the spectrum against penicillinaseproducing species. Cycloserine is a second-line agent for gram-negative organisms and tuberculosis.

A 45-year-old man had mental degeneration after a prolonged but inapparent infection. At autopsy, a subacute spongiform encephalopathy is found. What is the nature of the most likely causative agent? (A) Acid-fast organism (B) Dimorphic fungus (C) DNA virus (D) Viroid (E) Prion

E. Creutzfeldt-Jakob disease is an unconventional slow virus, or prion disease.

A father brings his 12-year-old son and tells you that "his son is feeling weakness in his legs and is beginning to fall a lot." The father says "he can't run as good as he used to." He also says it's gotten so bad that when "he is sitting down he has to put his hands on his thighs just to stand up." Relevant physical exam findings include rapidly progressive muscle weakness with frequent falls; muscle wasting in the legs and pelvis and progressing to shoulders and neck; and pseudohypertrophy of calf muscles. Relevant laboratory findings include no sign of myoglobulinuria, highly elevated creatine phosphokinase (CPK), and electromyography (EMG) showing weakness due to muscle tissue destruction and not nerve damage, and genetic testing reveals a mutation on chromosome Xp21. Which of the following is the most likely diagnosis? (A) Achondroplasia (B) Myasthenia gravis (C) McArdle disease (D) Polymyositis (E) Duchenne muscular dystrophy

E. Duchenne muscular dystrophy. Duchenne muscular dystrophy (DMD). DMD is an X-linked recessive disorder caused by a mutation in the gene for dystrophin on the short arm of chromosome X (Xp21). X-linked recessive inheritance means that males who inherit only one defective copy of the DMD gene from the mother have the disease. Dystrophin anchors the cytoskeleton (actin) of skeletal muscle cells to the extracellular matrix through a transmembrane protein (á-dystroglycan and â-dystroglycan) and stabilizes the cell membrane. A mutation of the DMD gene destroys the ability of dystrophin to anchor actin to the extracellular matrix. The characteristic dysfunction in DMD is progressive muscle weakness and wasting. Death occurs as a result of cardiac or respiratory failure, usually in the late teens or 20s. The description of how the boy arose from a seated position is called the Gower maneuver, which is classically seen in DMD. Becker muscular dystrophy normally begins around the third decade of life, whereas DMD can begin much earlier. McArdle disease is excluded because there was no sign of myoglobinuria, which would be a result of muscle glycogen phosphorylase deficiency.

A 45-year-old man with a history remarkable for both asthma and angina now has a kidney stone stuck in his right ureter. The urologist needs to perform cystoscopy, but the anesthesiologist is concerned about using a â-blocker during surgery to control the patient's blood pressure in light of his history of asthma. Ultimately, it is decided to use an ultra-short acting Beta-blocker and closely monitor both his blood pressure and respiratory status. Which of the following is the best to use in this situation? (A) Atenolol (B) Norepinephrine (C) Albuterol (D) Pseudoephedrine (E) Esmolol

E. Esmolol is an ultra-short acting beta1 antagonist that is relatively specific for the heart; however, the short half-life of this drug should allow the anesthesiologist to fine tune the delivery and readily reverse the effects should there be problems with respiration. Atenolol is a much longer acting agent that would not provide such control. Norepinephrine would actually adversely affect the patient's angina, as it is stimulatory to the heart. Albuterol is a beta agonist used in the treatment of asthma. Pseudoephedrine is an over-the-counter alpha agonist used in cold formula preparations.

A patient presents with an infected foot from stepping on a nail 2 days ago. The nail penetrated through the sole of his tennis shoe. He sprayed some antiseptic on it and did not seek medical help because he had his last tetanus booster about 1 year ago and he thought that would take care of the wound. The foot is quite inflamed around the wound with a little blue-green pus on the bandage. The organism isolated is a Gram-negative rod that is oxidase positive and does not ferment any carbohydrates. Which is it most likely? (A) Clostridium tetani (B) Escherichia coli (C) Klebsiella pneumoniae (D) Proteus vulgaris (E) Pseudomonas aeruginosa

E. From both the genus description and the epidemiology, it is most likely Pseudomonas aeruginosa. The rubber soles of tennis shoes generally clean off nails as they enter, so it is the flora of the inside of the tennis shoe, frequently Pseudomonas aeruginosa, causing these infections.

A 31-year-old woman comes into the office complaining of "running a fever," being nauseated, and losing weight—"about 15 pounds or so"—over the last month. She tells you that she had a miscarriage about 2 months ago and "all of a sudden these other problems come up." She added that she said that the doctors said she had "preeclampsia" during her first trimester of that pregnancy. She said that she was supposed to come back in, but she didn't because she "felt depressed about losing the baby." She remarks that she hasn't had any changes in her diet and remarked that she "thought she would have gained weight with all the food she was eating." Relevant physical exam findings include normal thyroid gland on palpation, no coughing of blood, and no diarrhea. Relevant laboratory findings include elevated hCG levels and normal thyroxine (T4) and thyroid-stimulating hormone (TSH) levels. Which of the following is the most likely diagnosis? (A) Achalasia (B) Hyperthyroidism (C) Pelvic inflammatory disease (D) Hydatidiform mole (E) Gestational trophoblastic neoplasia (or choriocarcinoma)

E. Gestational trophoblastic neoplasia (GTN; or choriocarcinoma). GTN is a malignant tumor of the trophoblast that may occur following a normal or ectopic pregnancy, abortion, or a hydatidiform mole. With a high degree of suspicion, elevated hCG levels are diagnostic. Nonmetastatic GTN (i.e., confined to the uterus) is the most common form of the neoplasia, and treatment is highly successful. However, the prognosis of metastatic GTN is poor if it spreads to the liver or brain. See Chapter 3, IV.D.

An Rh-positive neonate born to an Rh-negative mother has severe anemia and rapidly progressive jaundice. An older sibling, now also known to be Rh-positive, was delivered by a neighbor at home. The mother is unaware of any prior special testing or therapy related to anemia, jaundice, or gRh problems.h Which of the following mechanisms underlies the clinical scenario described here? (A) Activation of sensitized CD4ðq T cells with release of cytokines (B) gGraft-versus-neonateh response of immunologically incompetent newborn (C) Immunoglobulin (Ig)E-mediated degranulation of mast cells and basophils (D) Immune complex deposition with secondary inflammatory reaction (E) Interaction of IgG antibodies with intrinsic cell-surface antigens

E. Hemolytic disease of the newborn is a type II hypersensitivity reaction mediated by antibodies directed against intrinsic cell surface antigens (with complement- induced cell lysis). Modern management requires administration of IgG anti-Rh antibodies to the mother at the birth of a first Rh-positive child in order to avoid maternal sensitization. Another type of type II reaction is exemplified by the reaction of antibodies with cell-surface receptors, as occurs in Graves disease. Activation of sensitized CD4ðq T cells with release of cytokines is descriptive of type IV cell-mediated reactions, such as the delayed-type cell-mediated hypersensitivity exemplified by poison ivy dermatitis. gGraftversus- neonateh is a fictitious condition, but transfusion-mediated transplant rejection is an ever-present danger when immunodeficient persons receive transfusions that contain immunocompetent precursor cells. IgE-mediated degranulation of mast cells and basophils underlies type I hypersensitivity reaction, such as allergic asthma, hay fever, or anaphylaxis. Immune complex deposition with secondary inflammation occurs in generalized immune complex disorders, such as SLE.

A child from Appalachia presents with anemia, and a hookworm infection is diagnosed. This infection can be prevented by (A) Not swimming in contaminated water (B) Not using human excrement as vegetable fertilizer (C) Heating all canned foods to 60C for 10 minutes (D) Avoiding cat litter or taking proper care when changing litter (E) Wearing shoes outside in endemic regions

E. Hookworm filariform larvae may grow in the soil in endemic regions and can penetrate skin. Wearing shoes has been shown to greatly reduce the transmission of hookworm.

A 35-year-old man whose father had died of Huntington disease has the onset of neurologic abnormalities that had been predicted earlier by genetic analysis. Changes in the distance between the heads of the two caudate nuclei by magnetic resonance imaging are consistent with atrophy of the caudate nucleus and putamen. Which of the following is an important characteristic of this disorder? (A) Copper deposition in lenticular nuclei (B) Early childhood onset most common (C) Sphingomyelinase deficiency (D) Substantia nigra depigmentation (E) Trinucleotide repeat expansion

E. Huntington disease results from an expansion of the CAG trinucleotide repeat within the huntingtin gene, which can be detected by restriction length polymorphism (RFLP) analysis. Unlike many autosomal dominant disorders, it manifests later in life, in the fourth to fifth decade of life. Deposition of copper in the lenticular nuclei occurs in Wilson disease. A deficiency of sphingomyelinase results in some forms of Niemann-Pick disease. Loss of dopamine-producing neurons in the substantia nigra occurs in Parkinson disease.

A concerned couple brings their 3-weekold son into your office, stating that they think something is wrong with his genital area. They noticed that his testicles appeared to be swollen when they were changing his diaper a week ago. They said that his scrotum felt like a "water-filled balloon." Neither parent could recall any traumatic episode with their son, saying that they have been very protective of him. Relevant physical exam findings include an enlarged, nontender scrotum, testicles not immediately palpable, and no herniated bulge, and flashlight test through the enlarged area showed illumination. Relevant laboratory findings include absence of blood on fluid collection. Which of the following is the most likely diagnosis? (A) Hypospadias (B) Hematocele (C) Congenial inguinal hernia (D) Cryptorchidism (E) Hydrocele of the testes

E. Hydrocele of the testes. Hydrocele of the testes occurs when a small patency of the processus vaginalis remains, so that peritoneal fluid can flow into the processus vaginalis, which results in a fluid-filled cyst near the testes. Peritoneal fluid drains from the abdomen through the tunica vaginalis. The fluid accumulates in the scrotum, becomes trapped, and causes the scrotum to enlarge. A hydrocele is usually harmless and in most cases resolves within a few months after birth. A hydrocele is normally treated only when there is discomfort or when the testicular blood supply is threatened. A hematocele could have also been considered, but a hematocele is typically due to trauma, and blood would have been seen on fluid collection. Inginual hernias usually accompany hydroceles, but there was no bulge detected on physical examination.

A 21-year-old basketball player died suddenly during a game. Autopsy revealed hypertrophy of the left ventricular wall, especially of the ventricular septum. Histologically, the myocardial fibers were arranged in a disorganized pattern. Which of the following best characterizes this disorder? (A) Can be a manifestation of primary amyloidosis (B) Can be a result of myocarditis (C) Is often associated with alcohol abuse (D) Is often associated with coronary artery disease (E) Often demonstrates autosomal dominant inheritance

E. Hypertrophic cardiomyopathy, a condition that is usually inherited as an autosomal dominant disorder, is often associated with sudden death in young athletes. The ventricular septum is especially involved, with protrusion into the left ventricular cavity (asymmetric septal hypertrophy), sometimes leading to left ventricular outflow tract obstruction.

A 10-month-old girl presents with recurrent pulmonary infections, steatorrhea, and failure to thrive. Measurement of which substance is the most appropriate procedure in this patient? (A) Erythrocyte glucose-6-phosphate dehydrogenase (G6PD) (B) Serum ceruloplasmin (C) Serum â-lipoprotein (D) Serum phenylalanine (E) Sweat chloride

E. In a pediatric patient, the combination of recurrent pulmonary infections and steatorrhea (presumably due to pancreatic insufficiency) is strongly suggestive of cystic fibrosis. This disorder is characterized by a generalized defect in the reabsorption of anions, leading to increased sweat chloride concentration, an important diagnostic indicator.

What characteristic of the influenza A virus allows generic reassortment? (A) Poor editing by the RNA-dependent RNA polymerase (B) Presence of the rec-A gene product (C) Defective chaperone proteins (D) Poor editing function of the reverse transcriptase (E) Segmented genome

E. Influenza A virus, which causes a localized respiratory infection, has a segmented genome composed of eight pieces of negative-sense, single-stranded RNA, which can ''reassort'' when two different strains infect the same cell.

In the following pairs of organisms, which two are easiest to distinguish from each other by Gram stain? (A) Bacillus and Clostridium (B) Salmonella and Shigella (C) Haemophilus and Escherichia (D) Corynebacterium and Lactobacillus (E) Listeria and Proteus

E. Listeria is a Gram-positive rod, whereas Proteus is a Gram-negative rod. Clostridium, Lactobacillus, Corynebacterium, and Bacillus are all Gram-positive rods, whereas Haemophilus, Escherichia, Salmonella, and Shigella are all Gram-negative rods.

A 32-year-old man complains of a persistent, dry cough for several days with a mild fever and fatigue. The family physician suspects a diagnosis of "walking pneumonia" on clinical grounds, presumably due to Mycoplasma pneumoniae. Which of the following groups of antibiotics would be effective? (A) Penicillins (B) Cephalosporins (C) Vancomycin (D) Chloramphenicol (E) Macrolides

E. Macrolides such as azithromycin or clarithromycin are the agents of choice for the treatment of mycoplasmal diseases. As mycoplasmas have no cell wall, drugs such as penicillins, cephalosporins, or vancomycin are ineffective. Chloramphenicol is relatively toxic and reserved for select infections.

A neurosurgeon decides to start a patient on a diuretic that works by altering the diffusion of water relative to sodium (an osmotic diuretic) that is helpful in reducing cerebral edema. Which agent did the physician likely prescribe? (A) Furosemide (B) Hydrochlorothiazide (C) Spironolactone (D) Acetazolamide (E) Mannitol

E. Mannitol is an osmotic diuretic frequently used in the management of cerebral edema caused by various insults. This agent works by altering the diffusion of water relative to sodium by "binding" the water, with a resultant reduction of sodium reabsorption. Furosemide and hydrochlorothiazide act by directly altering reabsorption of sodium in various parts of the nephron (choices A and B). Spironolactone (choice C) antagonizes mineralocorticoid receptor. Acetazolamide inhibits carbonic anhydrase (choice D).

Which of the following agents is FDA approved for first-line treatment of diabetic neuropathy? (A) Phenytoin (B) Carbamazepine (C) Acetazolamide (D) Valproic acid (E) Pregabalin

E. Many antiseizure drugs find applications for other diseases. Pregabalin is approved for the treatment of diabetic nephropathy, an unfortunate consequence in this patient's presentation. Phenytoin is also used for the treatment of arrhythmias. Carbamazepine is used in the management of trigeminal neuralgia. It is occasionally used as a third-line agent to treat diabetic nephropathy. Acetazolamide, sometimes used as a treatment for absence seizure control, is used in the treatment of glaucoma. Valproic acid can be used in the prophylaxis of migraine headaches.

A 56-year-old female with severe rheumatoid arthritis returns to see her rheumatologist. She had been referred to a gastroenterologist, who had found multiple gastric ulcers on esophogastroduodenoscopy. She is reluctant to give up the use of NSAIDS and afraid of the potential cardiovascular toxicity of COX-2 inhibitor. At this point, what would be reasonable for the rheumatologist to prescribe? (A) Omeprazole (B) Lansoprazole (C) Nizatidine (D) Metronidazole (E) Misoprostol

E. Misoprostol is approved for use in patients taking NSAIDS, both to decrease acid production and to increase bicarbonate and mucous production. Both omeprazole and lansoprazole are proton inhibitors that would not increase the protective mucous and bicarbonate. Nizatidine is an H2-blocker that would also do nothing to increase the production of protective prostaglandins. Metronidazole is an antibiotic used to treat Helicobacter pylori.

A 26-year-old woman presents with high fever, painful and frequent urination, and left flank pain. The isolate is a Gram-negative, rodshaped, facultative anaerobe that is oxidase negative and ferments glucose and lactose. From these data what genus is most likely? (A) Neisseria (B) Proteus (C) Vibrio (D) Campylobacter (E) Escherichia

E. Neisseria are diplococci, vibrios are comma shaped, and campylobacteria are spiral shaped. Neisseria and vibrios are all oxidase positive. Because campylobacter is grown under unique conditions and temperature, it is identified by just growing. It has both a poorly functioning oxidase and catalase. Proteus and Escherichia are rod shaped and both are oxidase negative. Escherichia is the only one of the two that is a lactose fermenter.

A renal biopsy taken from a 23-year-old woman with nephrotic syndrome prominently features glomerular immune complex deposits. Of the following disorders that affect the glomerulus, which is suggested by the findings? (A) Amyloidosis (B) Diabetic nephropathy (C) IgA nephropathy (D) Minimal change disease (lipoid nephrosis) (E) Membranous glomerulonephritis

E. Of the choices listed, only membranous glomerulonephritis is an immune complex disease.

A 45-year-old man comes in complaining of chest and abdominal pain. He also says that his "blood pressure rises every so often" even when he is relaxing at home and that "it's been happening more and more." He says he exercises often and tries to stay in shape because he has a family history of obesity. Relevant physical exam findings include profuse sweating, hypertension, abdominal discomfort, and lungs clear on auscultation. Relevant laboratory findings include radiograph negative for a pulmonary embolism, hyperglycemia, increased urinary vanillylmandelic acid (VMA) and metanephrine levels, and inability to suppress catecholamines with clonidine. Which of the following is the most likely diagnosis? (A) Angina (B) Pneumothorax (C) Myocardial infarction (D) Neuroblastoma (E) Pheochromocytoma

E. Pheochromocytoma (PH). PH is a relatively rare neoplasm that contains both epinephrine and norepinephrine. PH occurs mainly in adults 40 to 60 years old and is generally found in the region of the adrenal gland, but it may be found in extrasuprarenal sites. PH is associated with persistent or paroxysmal hypertension, anxiety, tremor, profuse sweating, pallor, chest pain, and abdominal pain. Laboratory findings include increased urine VMA and metanephrine levels, inability to suppress catecholamines with clonidine, and hyperglycemia. PH is treated by surgery or phenoxybenzamine (an á-adrenergic antagonist).

Pilocarpine is what type of pharmacologic agent? (A) Indirect muscarinic agonist (B) á2-Adrenergic agonist (C) Carbonic anhydrase inhibitor (D) â-Adrenergic antagonist (E) Direct-acting muscarinic agonist

E. Pilocarpine is a direct-acting muscarinic agonist used in the management of acute narrow-angle glaucoma, often with an indirect-acting muscarinic agonist like physostigmine. Carbonic anhydrase inhibiters (e.g., acetazolamide), â-adrenoreceptor agonists, and even á2-adrenoreceptor agonists can be used in the treatment of glaucoma.

After being picked up by the police, a runaway adolescent girl became increasingly somnolent, lapsing into a deep coma 72 hours later. Her respirations were rapid and deep, and she appeared to be severely dehydrated. Laboratory studies revealed a marked reduction in serum bicarbonate and a significant anion gap, as well as neutrophilic leukocytosis. The most likely additional laboratory abnormality is (A) a blood urea nitrogen (BUN):creatinine ratio of less than 1:10. (B) decreased serum cortisol. (C) decreased serum thyroxine (T4). (D) increased CSF protein with no parallel increase in cell count. (E) increased serum glucose.

E. Progressive somnolence leading to metabolic acidosis (low bicarbonate with significant anion gap), coma, and severe dehydration, often with prerenal azotemia, are all strongly suggestive of diabetic ketoacidosis. Expected findings in this condition include increased serum and urine glucose and ketones.

A 16-year-old girl with short stature, rounded face, and shortening of the fourth and fifth metacarpals and metatarsals has hypocalcemia and hyperphosphatemia. She has multiple calcifications involving the basal ganglia, vasculature, and other sites. PTH varies from normal to increased, and administration of PTH does not result in phosphaturia. These findings are characteristic of which of the following? (A) Primary hyperparathyroidism (B) Secondary hyperparathyroidism (C) Tertiary hyperparathyroidism (D) Hypoparathyroidism (E) Pseudohypoparathyroidism

E. Pseudohypoparathyroidism is characterized by renal end-organ unresponsiveness to PTH and by shortened fourth and fifth metacarpals and metatarsals, short stature, and other skeletal abnormalities. These abnormalities are due to mutations in GNAS1, a G protein that mediates receptiveness to PTH and other hormones. Selective imprinting results in maternal inheritance of the end-organ unresponsiveness. The clinical findings mimic those of hypoparathyroidism, but PTH is most often normal or elevated.

A young mother brings in her 3-year-old son because of "a white spot in his right eye" that she first noticed in a photograph taken 2 weeks ago. She also tells you that "he seems to be always squinting with his right eye." She remembers hearing about distant family member with the same sort of spot who eventually went blind. Relevant physical exam findings include leukocoria (whitish spots in the pupillary area behind the lens), strabismus (squinting; deviation of the eye that the patient cannot overcome), poor vision in the right eye, and curious family history. Relevant laboratory findings include CT scan showing a solid intraocular tumor with intratumoral calcifications, and genetic testing reveals a deletion on chromosome 13q. Which of the following is the most likely diagnosis? (A) Congenital cataract (B) Congenital glaucoma (C) Retinitis pigmentosa (D) Papilledema (E) Retinoblastoma

E. Retinoblastoma. Retinoblastoma (RB) is a tumor of the retina that occurs in childhood and develops from precursor cells in the immature retina. The RB gene is located on chromosome 13q and encodes for RB protein, which binds to a gene regulatory protein and causes suppression of the cell cycle, that is, the RB gene is a tumor-suppressor gene (also called an anti-oncogene). A mutation in the RB gene will encode an abnormal RB protein such that there is no suppression of the cell cycle. This leads to the formation of RB. Hereditary RB causes multiple tumors in both eyes. Nonhereditary RB causes one tumor in one eye.

The parents of an 18-month-old girl appear at the emergency department of a hospital with their daughter who had an abrupt onset of vomiting and incidences of watery diarrhea. The symptoms began 2 days previously at her day care center. She refuses to eat or drink and is very lethargic. What property of the infectious agent causing these symptoms allowed the preparation of a vaccine that could have prevented them? (A) Ability to survive as a temperature-sensitive mutant (B) Identification of an enterotoxin (C) Loss of colonization factors (D) Manipulation of ''O'' antigens (E) Segmented genetic material

E. Rotaviruses are the most frequent cause of diarrheas in infants. Rotateq and Rotarix vaccines are available for prevention.

A 35-year-old man presents with a painless lump in his right testicle. Scrotal ultrasound reveals a homogeneous intratesticular mass. Histologic examination of the testicular mass tissue demonstrates morphology that closely resembles that of dysgerminoma of the ovary in women. Which of the following is the most likely diagnosis? (A) Androblastoma (Sertoli cell tumor) (B) Endodermal sinus tumor (C) Mature teratoma (D) Mixed germ cell tumor (E) Seminoma

E. Seminoma, a germ cell tumor of the testis with peak incidence in the mid-30s age group, is analogous to (and histologically closely resembles) dysgerminoma of the ovary.

A 49-year-old man has a recent diagnosis of small cell carcinoma of the lung. Which of the following is an important characteristic of this form of lung cancer? (A) Ectodermal origin (B) Frequent peripheral location (C) Less association with cigarette smoking than other forms of lung cancer (D) Paraneoplastic hyperparathyroidism (E) Poorly amenable to surgery

E. Small cell carcinoma of the lung is almost always metastatic at the time of initial diagnosis and is thus poorly amenable to surgery. Despite morphologic differences, it is thought that all lung carcinomas, including small cell carcinoma, share a common endodermal origin. The location is most often central rather than peripheral, and there is a marked association with cigarette smoking. Paraneoplastic syndromes include inappropriate secretion of ACTH and ADH. Secretion of a protein with PTH-like activity is an association of squamous cell lung carcinoma.

A neonate develops meningitis at 7 days of age. Her mother is 16 years of age, single, has had multiple sexual partners without barrier protection, and lives in the United States. The baby was born 23 hours after the mother's amniotic sac ruptured. What is the most likely causative agent? (A) Escherichia coli (B) Haemophilus influenzae (C) Listeria monocytogenes (D) Neisseria meningitidis (E) Streptococcus agalactiae

E. Streptococcus agalactiae (group B streptococci) is the most common cause of neonatal meningitis. It is most prevalent in young women who have had multiple partners and is most likely to infect the baby during a protracted delivery. Escherichia coli is the second most common cause of neonatal meningitis. Listeria is a less frequent cause of neonatal meningitis and other severe diseases in newborns. Haemophilus influenzae and Neisseria meningitidis rarely cause neonatal meningitis.

An alcoholic presents complaining of chest pain, fever, shaking chills, cough, and myalgia. She was very cold two nights ago and says she has felt ''poorly'' ever since. Her cough is producing rust-colored, odorless, mucoid sputum. Her temperature on admission is 40C. Her white blood cell count is 16,000 cells/mm3 and is predominantly neutrophils with an overall left shift. An a-hemolytic, lancet-shaped, Grampositive diplococcus is isolated on blood agar. What is the most likely causative agent? (A) Legionella pneumophila (B) Klebsiella pneumoniae (C) Mycoplasma pneumoniae (D) Neisseria meningitidis (E) Streptococcus pneumoniae

E. Streptococcus pneumoniae is the most common causative agent of pneumonia in alcoholics. Klebsiella pneumoniae is less common but even more deadly because of the high incidence of abscesses. (Almost all of the patients who have pneumonia caused by K. pneumoniae suffer from chronic lung disease or alcoholism.) If foul-smelling sputum had been present, then anaerobes would most likely be involved. Legionella and Klebsiella are both Gram-negative rods. Neisseria meningitidis is a Gram-negative diplococcus. Neither Legionella nor Mycoplasma would have grown on blood agar.

A 14-year-old girl dies after an illness characterized by progressive motor and mental deterioration. At autopsy, there is profound cortical atrophy, loss of white matter, and ventricular enlargement. Special studies indicate participation of a defective measles virus. Which of the following is the diagnosis? (A) Creutzfeldt-Jakob disease (B) Guillain-Barré syndrome (C) Kuru (D) Progressive multifocal leukoencephalopathy (E) Subacute sclerosing panencephalitis

E. Subacute sclerosing panencephalitis, one of the slow virus infections, is thought to be caused by persistent infection with a defective measles virus. The virus lacks the M component, a protein required for extracellular spread of the virus. This deficiency is thought to explain the slow nature of the infection.

An arthropod vector is involved in infection by (A) Respiratory syncytial virus (B) Parvovirus (C) Reovirus (D) Parainfluenza virus (E) Bunyavirus (F) Arenavirus

E. The California and LaCrosse viruses, which have mosquito vectors, are both bunyaviruses.

A woman who is an intravenous drug abuser and who has been a prostitute for the past 20 years now has an aortitis. She has no mucosal ulcerations or exanthems. Her VDRL (Venereal Disease Research Laboratory) test is negative and her fluorescent treponemal antibody absorption test is positive. What is the most likely diagnosis? (A) Early primary syphilis (B) Lyme disease (C) Secondary syphilis (D) Latent syphilis (E) Tertiary syphilis

E. The FTA-abs test detects specific antitreponemal antibody. The FTA-abs remains positive for life with or without antibiotic therapy. The VDRL test detects less specific reaginic antibodies, which decline with successful treatment, but also sometimes decline without treatment in tertiary syphilis. Therefore, based on the serologic data, this patient could have very early primary syphilis or untreated tertiary syphilis. The symptoms are consistent with tertiary syphilis.

A 32-year-old woman was evaluated for severe watery diarrhea. Diagnostic testing revealed achlorhydria and reduced concentrations of serum potassium. Imaging studies revealed the presence of a pancreatic tumor. Special stains will most likely reveal the pancreatic tumor to be which of the following? (A) Alpha-cell tumor (glucagonoma) (B) Beta-cell tumor (insulinoma) (C) Gastrinoma (D) Somatostatinoma (E) VIPoma

E. The VIPoma is an islet cell tumor of the pancreas that is associated with Watery Diarrhea, Hypokalemia, and Achlorhydria (WDHA syndrome or Verner-Morrison syndrome), all caused by the secretion of vasoactive intestinal peptide (VIP) by the tumor.

Prior to the birth of a stillborn infant, decreased amniotic fluid (oligohydramnios) for gestational age was demonstrated on ultrasound examination. Which of the following autopsy findings of the stillborn is most consistent with this maternal-fetal abnormality? (A) Infection with Toxoplasma gondii, plus microcephaly, hydrocephaly, chorioretinitis, and microphthalmia (B) Infection with rubella virus, along with microcephaly and heart malformations (C) Infection with CMV, plus microcephaly, hepatitis, and intracranial calcifications (D) Microcephaly, facial anomalies, and a number of central nervous system (CNS) anomalies, as well as a history of major maternal alcohol abuse (E) Bilateral renal agenesis and hypoplasia of one lung

E. The absence of both fetal kidneys leads to a decreased volume of amniotic fluid (oligohydramnios) because fetal urine is a major source of amniotic fluid. This leads to a group of secondary abnormalities, often including distorted facies and unilateral pulmonary hypoplasia. This sequence of events is referred to as the Potter progression. The other choices refer to a group of related syndromes often grouped together as the TORCH complex (produced by a group of organisms: Toxoplasma, Other etiologies, Rubella virus, Cytomegalovirus, or Herpes simplex virus; the "Other" designation includes diverse viruses and the treponemal spirochete of syphilis). These infections are associated with a similar group of fetal abnormalities, including microcephaly, brain lesions with focal calcifications, and ocular and cardiac anomalies. Microcephaly is also a common feature of the fetal alcohol syndrome.

A 30-year-old African-American woman presents with bilateral hilar lymphadenopathy and reticular densities in both lung fields. Which of the following is a defining characteristic of the disorder suggested by these findings? (A) Abnormalities restricted to lung and hilar lymph nodes (B) Hypocalcemia (C) Impaired synthesis of immunoglobulins (D) Marked hyperreactivity to tuberculin (E) Noncaseating granulomas

E. The clinical picture strongly suggests a diagnosis of sarcoidosis. The granulomas of sarcoidosis are characteristically noncaseating. Sarcoidosis is a multisystem disorder. Common findings in this highly variable disorder include anergy to tuberculin, hypercalcemia, and broad-based polyclonal hypergammaglobulinemia.

A 22-year-old woman presents with vaginal itching and erythema as well as a discharge which is thick and white. External erythema is also present with discrete pinpoint lesions off the edge. The discharge pH is 4.7 (normal). Amine test is negative as is the genetic probe test for Neisseria. What is the most likely cause? (A) Bacterial vaginosis (B) Overgrowth of Gardnerella vaginalis (C) Trichomonas vaginalis (D) Chlamydia trachomatis (E) Candida albicans

E. The discharge is characteristic of Candida overgrowth. Note that no foul odor is mentioned, and the negative amine test ''rules down'' bacterial vaginosis and Trichomonas. Both yeast vaginitis and yeast diaper rash have the described satellite lesions outside the area of major erythema. Normal pH range is 4.5 to 5 and yeast vaginitis is usually in that range.

A 60-year-old man presents with progressive bone deformity and pain, progressive hearing loss, and increasing skull size. Workup revealed generalized increased bone density with cortical thickening, normal serum calcium and phosphorus, and markedly elevated serum alkaline phosphatase. Biopsy revealed a characteristic "mosaic" pattern. Which one of the following tumors is a known complication of the disorder suggested by this scenario? (A) Ewing sarcoma (B) Giant cell tumor (C) Metastatic duct carcinoma of the breast (D) Multiple enchondromas (E) Osteosarcoma

E. The history is suggestive of Paget disease of bone, which is also marked by bone pain, anterolateral bowing of long bones, and sometimes high-output heart failure. This disorder is complicated by osteosarcoma in about 1% of cases. Paget disease of bone should not be confused with Paget disease of the breast, which is closely associated with an underlying ductal carcinoma.

Your patient, who has been hospitalized in a coma for 2 weeks, has developed erythema around his catheter. You order the catheter be pulled and the tip cultured and that a new line be started on the opposite side of the body. The report comes back that it is Staphylococcus epidermidis, which does not surprise you, as it is noted for the production of a loose network of polysaccharide sticking to catheters. What is this layer on the cell called? (A) Peptidoglycan (B) Pili (C) Teichoic acids (D) Biofilm (E) Glycocalyx

E. The layer on the cell could be called either a glycocalyx (choice C) or capsule (not a choice). The layer on the catheter is a biofilm, but the question asked for the name of the layer on the cell.

A 55-year-old woman who died suddenly at home was found at autopsy to have suffered a rupture of the left ventricle. Which one of the following myocardial changes is the most frequent cause of this catastrophic event? (A) Abscess formation and tissue destruction due to infective endocarditis (B) Fatty change due to interaction of diphtheria exotoxin and carnitine (C) Inflammation associated with Aschoff bodies (D) Inflammation due to coxsackie B infection (E) Necrosis due to coronary artery obstruction

E. The most frequent cause of cardiac rupture is MI. This complication, which often results in hemopericardium and cardiac tamponade, occurs with peak incidence within 4 to 10 days after infarction.

A 12-year-old boy presents with smoky brown-colored urine, oliguria, azotemia, and hypertension. After further tests, the patient is diagnosed with a nephritic syndrome. Which of the following is the most likely diagnosis? (A) Renal amyloidosis (B) Diabetic nephropathy (C) Membranous glomerulonephritis (D) Minimal change disease (E) Poststreptococcal glomerulonephritis

E. The prototype of the nephritic syndrome is poststreptococcal glomerulonephritis.

A 6-month-old boy with hypertension is found to have a very large tumor arising in the left adrenal gland. Microscopically, the tumor consists of sheets of "small round blue cells" with minimal cytoplasm. Some of the cells contain pink cytoplasm and nucleoli, suggesting differentiation toward ganglion cells, and neurosecretory granules are visualized by electron microscopy. Urinary VMA is markedly increased. Which of the following is a major characteristic of this neoplasm? (A) Characteristic gene deletions on chromosome 5 (B) Highly differentiated form of malignant ganglioneuroma (C) Posterior cranial fossa origin with metastasis to both adrenals (D) Serotonin production (E) Spontaneous differentiation in some instances

E. The scenario is typical of neuroblastoma, the most frequently occurring tumor in infants less than 1 year of age. The tumor may occasionally undergo spontaneous differentiation to a benign ganglioneuroma. Marked amplification of N-myc is characteristic, and greater amplification is a negative prognostic indicator. Most neuroblastomas are peripheral, and the most frequent site of origin is the adrenal medulla or adjacent tissues. CNS neuroblastomas are less common, and most often involve the cerebral hemispheres. Origin in the posterior cranial fossa is rare. Catecholamine production is characteristic.

A patient with a chronic cough is given a tuberculin skin test, which when properly read has a zone of induration greater than 15 mm. What does this mean? (A) Active infection with Mycobacterium tuberculosis (B) Active infection with any of the nontuberculous mycobacteria (C) Anergy (D) Antibody titer to M. tuberculosis (E) Previous infection with M. tuberculosis (F) Vaccination with bacillus Calmette-Guerin (BCG) vaccine only

E. The tuberculin test indicates previous infection with Mycobacterium tuberculosis or Mycobacterium bovis from between several weeks to 5 years. It does not provide proof of current active infection with M. tuberculosis. The tuberculin test detects cell-mediated immunity and not antibody. If a person is infected with a nontuberculous strain of mycobacteria, usually the skin test is smaller. Specific skin tests for some of these nontuberculous strains of mycobacteria are available. Vaccination with bacillus Calmette-Guerin (BCG) vaccine should not result in this large a zone of induration.

Intracellular survival and replication are the major virulence factors rather than exotoxin production for which of the following organisms? (A) Cholera (B) Diphtheria (C) Gastroenteritis caused by enterotoxigenic Escherichia coli (D) Pertussis (E) Plague

E. The virulence of Yersinia pestis does not depend on exotoxins. Instead, it depends on a variety of other factors, the most important of which is its ability to proliferate intracellularly. Associated with this ability and virulence are Ca2+ dependence; V andWantigens; Yersinia outer membrane proteins; F1 envelope antigen; coagulase, pesticin, and fibrinolysin production; and pigment absorption.

Adverse effects seen with high blood levels of theophylline include (A) Seizures (B) Arrhythmias (C) Nervousness (D) Nausea and vomiting (E) All of the above

E. Theophylline is associated with all of the reactions listed. They usually occur at elevated blood levels, generally accepted as greater than 20 ìg/dL. However, adverse drug reactions may occur at any blood level.

A 25-year-old woman experiences the sudden onset of fever, chills, right flank pain, and right-sided costovertebral angle tenderness. Her urinary sediment contains numerous gram-negative bacilli. Which of the following additional urinary findings would help establish the likely diagnosis? (A) Broad waxy casts (B) Decreased protein (C) Decreased volume (D) Red cell casts (E) White cell casts

E. This is a classic case of acute pyelonephritis, an acute infection of the renal parenchyma. White cell casts in the urine are pathognomonic of acute pyelonephritis. Although microscopic hematuria is a frequent finding in acute pyelonephritis and other urinary tract infections, red cell casts are not seen since the glomeruli tend to be spared in renal infection. Red cell casts are a specific indicator of glomerular inflammation.

Two physicians traveling in Central America envelope what they self-diagnose as ET Escherichia coli. What is responsible for the fluid and electrolyte disruption? (A) Adherence causing palisade layers of the bacterium on the surface of the small intestine (B) Exotoxin that inhibits protein synthesis by nicking 60S ribosomal subunits (C) Entrance through intestinalMcells and migration through the tissue through actin polymerization/''jetting'' (D) Exotoxin that inhibits protein synthesis by blocking elongation factor 2 (E) Exotoxin that causes an increase in cyclic adenosine monophosphate (F) Exotoxin that inhibits protein synthesis by nicking 60S ribosomal subunits

E. Traveler's diarrhea is most frequently caused by enterotoxigenic strains of Escherichia coli that produce the heat-labile (LT) and heat-stable (ST) toxin. LT is an exotoxin that causes an increase in cyclic adenosine monophosphate. It is not invasive, making choice C incorrect. Choice B is incorrect, it is the mechanism of Shiga toxin and the O157 shigalike toxin also known as verotoxin.

Which of the individuals listed below is the most likely to develop a serious disease from West Nile virus? (A) A 4 month old hospitalized with a serious RSV infection (B) A 3 year old with croup (C) A 21-year-old college student returning from a vacation in Costa Rica (D) A 24-year-old tour guide in the four corners of the southwestern United States (E) A 55-year-old camper returning from a 2-week wilderness experience in Minnesota

E. West Nile virus is an arbovirus transferred to humans by a mosquito vector. The most serious consequences following infection occurs in those individuals over 50 years of age who can develop a life-threatening encephalitis.

In the diagnostic workup of a 42-yearold man with chronic malabsorption and diarrhea, bacillary forms within periodic acid Schiff (PAS)-positive macrophages in the lamina propria of the small intestinal mucosa were demonstrated by electron microscopy. This finding is characteristic of (A) celiac disease. (B) Crohn disease. (C) disaccharidase deficiency. (D) tropical sprue. (E) Whipple disease.

E. Whipple disease, a systemic illness almost always involving the small intestine, is characterized morphologically by distinctive PAS-positive macrophages within affected organs. On electron microscopy, the PAS-positive material is seen to consist of numerous bacillary forms of the gram-positive actinomycete Tropheryma whippelii. The disorder responds to a number of antibacterial agents, but without therapy the course is usually progressive and fatal.

A 25-year-old male who has recently started doing intravenous drugs presents to the emergency department with fever and evidence of mild tricuspid valve insufficiency. As far as he knows, he had no previous damage to his heart. What is the most likely causative agent? (A) Candida albicans (B) Coxsackie virus (C) Enterococcus faecalis (D) Pseudomonas aeruginosa (E) Staphylococcus aureus

E. With tricuspid valve insufficiency in IVDA, the most common cause is Staphylococcus aureus, which generally presents as an acute infection.

An 80-year-old woman is referred to your cardiac practice with no fever but worsening exertional fatigue. She has had a heart murmur since she was young. It appears to have worsened recently. At her 80th birthday party 4 weeks ago, two of her great nieces were sick, and she thinks she picked up their sore throat. She was sick with fever for about a week 3 weeks ago but did not have enough energy to go to the doctor. If you did standard bacterial blood cultures, what would the most likely outcome be? (A) Growth of Streptococcus pyogenes (B) Growth of Viridans streptococci (C) Growth of Staphylococcus aureus (D) Growth of enterococci (E) No growth

E. Without dental work or gastrointestinal or genitourinary manipulation and with the pharyngitis, the most likely explanation is that your patient had rheumatic heart disease made worse recently by an untreated strep throat. Because it is the antibodies (and, perhaps, immune cells) circulating and not bacteria, the blood cultures should be sterile. Antistreptolysin O titer should be positive.

In a laboratory exercise, a 23-yearold female pathology student was found to have a prolonged bleeding time and a prolonged APPT. Her platelet count was normal. These findings are strongly suggestive of (A) Christmas disease. (B) classic hemophilia. (C) congenital afibrinogenemia. (D) Glanzmann thrombasthenia. (E) von Willebrand disease.

E. von Willebrand disease (congenital deficiency of von Willebrand factor [vWF]) is characterized by defective platelet adhesion, resulting in a prolonged bleeding time even though the platelets are qualitatively and quantitatively normal. The APTT is also prolonged because of a secondary deficiency of factor VIII. Factor VIII normally circulates in a complex with vWF and is unstable when vWF is deficient.

Arterial pH of 7.54, arterial [HCO3] of 48 mEq/L, hypokalemia, and hypoventilation would be observed in a (A) patient with chronic diabetic ketoacidosis (B) patient with chronic renal failure (C) patient with chronic emphysema and bronchitis (D) patient who hyperventilates on a commuter flight (E) patient who is taking a carbonic anhydrase inhibitor for glaucoma (F) patient with a pyloric obstruction who vomits for 5 days (G) healthy person

F [Chapter 5, IX D 2; Table 5.9]. The blood values and history of vomiting are consistent with metabolic alkalosis. Hypoventilation is the respiratory compensation for metabolic alkalosis. Hypokalemia results from the loss of gastric K+ and from hyperaldosteronism (resulting in increased renal K+ secretion) secondary to volume contraction.

An otherwise healthy 23-year-old woman presents with urinary urgency and frequency along with pain on micturition. What is the most likely causative agent? (A) Bacteroides species (B) Clostridium botulinum (C) Clostridium difficile (D) Clostridium perfringens (E) Clostridium tetani (F) Escherichia coli

F. Escherichia coli is a common causative agent of urinary tract infection. The strict anaerobes do not cause urinary tract infections.

A functional virogene is missing in (A) Mouse mammary tumor virus (B) Rous sarcoma virus (C) Polyomavirus (D) Human T lymphotrophic virus (E) Hepatitis B virus (F) Harvey sarcoma virus

F. Harvey sarcoma virus, like most viruses with high oncogenic potential, is a defective virus that lacks at least one functional virogene.

A 3 year old child develops acute glomerulonephritis following impetigo. The bacterium is a catalase-negative, Gram-positive coccus that has M12 surface protein. What is the most likely causative agent? (A) Enterococcus faecalis (B) Staphylococcus aureus (C) Staphylococcus epidermidis (D) Streptococcus agalactiae (E) Streptococcus pneumoniae (F) Streptococcus pyogenes

F. Streptococcus pyogenes is a group A streptococci that has an Mprotein on its outer cell walls that interferes with phagocytosis in the immunologically naive individual. M12 strains are often nephritogenic.

An oncogene that codes for a guaninenucleotide- binding protein is found in (A) Mouse mammary tumor virus (B) Rous sarcoma virus (C) Polyomavirus (D) Human T lymphotrophic virus (E) Hepatitis B virus (F) Harvey sarcoma virus

F. The ras gene of the Harvey sarcoma virus codes for a guanine-nucleotide- binding protein, which has biologic activity that causes cellular transformation.

A patient presents with explosive, watery, noninflammatory diarrhea along with headache, abdominal cramps, nausea, vomiting, and fever. Symptoms began the day after eating raw oysters in August. What is the most likely causative agent? (A) Giardia lamblia (B) Norwalk agent (C) Rotavirus (D) Salmonella enteritidis (E) Staphylococcus aureus (F) Vibrio parahaemolyticus

F. Vibrio parahaemolyticus is the most likely causative agent. It is found in raw oysters from contaminated oyster beds, usually in late summer.

If heart rate increases, which phase of the cardiac cycle is decreased? (A) Atrial systole (B) Isovolumetric ventricular contraction (C) Rapid ventricular ejection (D) Reduced ventricular ejection (E) Isovolumetric ventricular relaxation (F) Rapid ventricular filling (G) Reduced ventricular filling

G [Chapter 3, V; Figure 3.15]. When heart rate increases, the time between ventricular contractions (for refilling of the ventricles with blood) decreases. Because most ventricular filling occurs during the "reduced" phase, this phase is the most compromised by an increase in heart rate.


संबंधित स्टडी सेट्स

2016-2020 Amateur Extra FCC Element 4 Question Pool

View Set

Leading causes of death for people 15-24 years old

View Set

Chapter 28: Assessment of the Child (Data Collection)

View Set

Chapter 49 disorders of musculoskeletal function

View Set

Biology chapter 5 mastering practice

View Set